Location via proxy:   [ UP ]  
[Report a bug]   [Manage cookies]                

07-0146 in Class Manual GMAT V9.1

Download as pdf or txt
Download as pdf or txt
You are on page 1of 236

In Class Manual

for the GMAT


®

Version 9.1

GMAT is a registered trademark of the Graduate Management Admission Council.

DO NOT DISTRIBUTE
All rights reserved. No part of this manual may be reproduced for distribution
to a third party in any form or by any means, electronic or mechanical, including
photocopying, recording, or any information retrieval system, without the prior
consent of the publisher, The Princeton Review.

This Manual is for the exclusive use of Princeton Review course students and is
not legal for resale.

GMAT is a registered trademark of the Graduate Management Admission Council.

The Princeton Review is not affiliated with Princeton University or the Graduate
Management Admission Council.

Permission to reprint this material does not constitute review or endorsement


by ACT or the Graduate Management Admission Council of this publication as
a whole or of any other sample questions or testing information it may contain.

Copyright © 2012 by The Princeton Review, Inc. All Rights Reserved.

866.TPR.PREP / www.PrincetonReview.com

DO NOT DISTRIBUTE
Acknowledgments

Special thanks to John Fulmer, Oliver Pope, and Curtis Retherford.

Thanks to the following for their many contributions to this manual:

Shruti Ananthpadmanabha, Monisha Banerjee, Andrew Brody, Tim Christy,


Marty Cinke, Becky Robinson, Morrison Giffen, Peter Hanink, Cheryl Kreisher,
Mike Matera, Nicole Pirnie, Mindy Myers, Becky Robinson, Liz Rutzel,
Jake Schiff, Rebecca Scott, Allison Scovone, Ruchi Tomar, Vikram Vishwanath,
Dominique Walsh, Shadna Wise, Elia Zashin, and the staff and students of
The Princeton Review.

DO NOT DISTRIBUTE
| iii
DO NOT DISTRIBUTE
Table of Contents

Lesson 1 1
Verbal Introduction 1
Sentence Correction 1 2
The Two Keys to Sentence Correction Success............................ 2
The Format........................................................................................ 3
Basic Approach................................................................................. 3
Subject-Verb Agreement.................................................................. 4
Verb Tense.......................................................................................... 6
Pronouns............................................................................................ 8
Back Up Plans................................................................................. 10
Red Pencil Fever............................................................................. 11
Math 1 12
Math Introduction.......................................................................... 12
Data Sufficiency.............................................................................. 15
Algebra vs. Arithmetic................................................................... 18
Plugging In...................................................................................... 19
Hidden Plug Ins............................................................................. 22
Plugging In The Answers (PITA)................................................. 23
Math Practice 27
Math Practice Answers and Explanations 31

Lesson 2 33
Sentence Correction 2 33
Misplaced Modifiers...................................................................... 34
Parallel Construction..................................................................... 37
Comparisons................................................................................... 40
Idioms.............................................................................................. 42
Math 2 44
Pieces of the Puzzle........................................................................ 44
Yes-No Data Sufficiency................................................................ 46
Keep Plugging Away..................................................................... 49
Math Practice 52
Math Practice Answers and Explanations 54

DO NOT DISTRIBUTE
| v
Lesson 3 57
Math 3 57
Inequalities and Absolute Values................................................. 57
Roots and Exponents..................................................................... 58
Quadratics....................................................................................... 61
Functions and Sequences.............................................................. 62
Harder Manipulations................................................................... 65
Algebra and Data Sufficiency....................................................... 66
Integrated Reasoning 1 68
Table Analysis................................................................................. 68
Graphics Interpretation................................................................. 71
Multi-Source Reasoning................................................................ 72
Two-Part Analysis.......................................................................... 75
Math Practice 79
Math Practice Answers and Explanations 81

Lesson 4 85
Critical Reasoning 1 85
Basic Approach............................................................................... 85
Assumption Questions.................................................................. 86
The Negation Test........................................................................... 89
Weaken Questions.......................................................................... 90
Strengthen Questions..................................................................... 92
Math 4 94
Ratios................................................................................................ 94
Averages.......................................................................................... 96
Rates................................................................................................. 98
Percent Change............................................................................. 100
Probability..................................................................................... 101
Math Practice 106
Math Practice Answers and Explanations 110

Lesson 5 113
Reading Comprehension 1 113
The Basic Approach..................................................................... 113
Finding the Main Idea: Key Sentences...................................... 114
Finding the Main Idea: Follow the Author............................... 116
Finding the Main Idea: Change or Problem............................. 118
General Questions........................................................................ 119
Math 5 121
Geometry: The Basic Approach.................................................. 121
Data Sufficiency Figures.............................................................. 123
The Basic Approach and Geometry Data Sufficiency............. 124
Plugging In for Geometry........................................................... 125
Combining Concepts................................................................... 127
Shaded Regions............................................................................ 129
Coordinate Geometry.................................................................. 132
Math Practice 137
Math Practice Answers and Explanations 141

DO NOT DISTRIBUTE
vi |
Lesson 6 145
Reading Comprehension 2 145
The Basic Approach..................................................................... 145
Specific Questions and POE........................................................ 147
Critical Reasoning 2 152
Inference Questions..................................................................... 152
Integrated Reasoning 2 157
Integrated Reasoning and Inferences........................................ 157
Integrated Reasoning and Arguments...................................... 159
Plugging In for Two-Part Analysis............................................ 162
Plugging In The Answers for Two Part Analysis.................... 164

Lesson 7 167
Critical Reasoning 3 167
Common Argument Patterns..................................................... 167
Math 6 172
More Problem Solving Plugging In........................................... 172
More Data Sufficiency Plugging In............................................ 174
Number Theory Topics................................................................ 175
Math Practice 179
Math Practice Answers and Explanations 180

Lesson 8 183
Critical Reasoning 4 183
Identify the Reasoning Questions.............................................. 183
Resolve/Explain Questions........................................................ 186
Minor Question Types................................................................. 188
Sentence Correction 3 191
Redundancy.................................................................................. 191
Clauses and Connectors.............................................................. 192
Grammar and Meaning............................................................... 192
Math 7 196
Simultaneous Equations.............................................................. 196
Simultaneous Equations Nuances............................................. 197
Sets.................................................................................................. 199
Groups........................................................................................... 199
Venn Diagrams............................................................................. 200
Permutations and Combinations............................................... 201
Math Practice 205
Math Practice Answers and Explanations 209

Lesson 9 213
Verbal Review 213
Sentence Correction..................................................................... 213
Critical Reasoning........................................................................ 217
Reading Comprehension............................................................. 220
Math 8 223
POE Review.................................................................................. 223
Data Sufficiency Review.............................................................. 226

DO NOT DISTRIBUTE
| vii
DO NOT DISTRIBUTE
viii |
Lesson 1

11

Lesson 1

Verbal Introduction
Here are the basic facts for the verbal section:

• 36 questions in 65 minutes
• Approximately 13 Sentence Correction, 10 Critical Reasoning, and 13
Reading Comprehension questions.
• Verbal scores range from roughly 6 to 51 (0–60 scale).
• The verbal score is a factor in the 200 to 800 overall scale.
• The verbal section is computer adaptive.

A Systematic Approach
The credited responses to verbal questions may often seem subjective and arbitrary.
Fortunately, that’s not the real story.

Verbal questions follow definite rules. Learn the rules and you’ll
be able to pick the credited response.

The Importance of POE

The phrasing of the credited response is GMAC’s job. Your job is


to find good reasons to eliminate bad answers.

The multiple-choice format of the Verbal section actually works to your benefit
because you can use Process of Elimination (POE).

DO NOT DISTRIBUTE
| 1

GMAT InClass Manual Lesson 1.indd 1 7/2/18 6:47 PM


GMAT IN CLASS MANUAL

11

Sentence Correction 1
While it may seem that GMAT Sentence Correction questions test hundreds of
different grammatical rules, there really are only a handful of errors that get tested
consistently.

The Two Keys to Sentence Correction Success


There are two keys to doing well on sentence correction.

Know the Commonly Tested Errors

Here are the most commonly tested grammatical errors. We’ll show you how to spot
Look for these same
errors as reasons to and correct these errors.
eliminate incorrect
answer choices.

The Big Six


The most common types of errors are:
• Verb Tense
• Pronouns
• Idioms
• Misplaced Modifiers
• Parallel Construction
• Subject-Verb Agreement

Know the Characteristics of a Good Answer

Good answers have four characteristics. Eliminate answer choices that lack one of
these properties.

Characteristics of a Good Answer


• Fixes the error
• Makes no new errors
• Doesn’t change the meaning
• Is concise versus other answers

DO NOT DISTRIBUTE
2 |
Lesson 1

11

The Format
Each sentence correction question presents a sentence with some or all of the
sentence underlined.

1. The Texas longhorn, a breed of cattle descended


from herds brought by the Spanish conquistadors, Answer (A) always repeats
are symbols of the Wild West era of American the underlined portion of
history. the sentence.

are symbols of the Wild West era


are symbols of the Wild West
are symbolizing the Wild West era
is symbolic in the Wild West
is a symbol of the Wild West era

Basic Approach
Spotting grammatical errors is important. But, once you spot an error, you need
a systematic approach to get to the credited response.

Can you ID one of the common errors in the underlined portion?


Yes No
Eliminate Choice (A) Can you ID an error from
the answer choices?
• 2/3 split See how 2 answers start
• Differences in answers with is and 3 answers start
Eliminate any answers that
with are in the question
obviously repeat the error. Yes No above? That’s called a 2/3
split.

POE with the remaining Starting at (B), use POE.


answer choices. • New errors.
• New errors. • Changes to the meaning
• Changes to the meaning of the sentence.
of the sentence.

Two left? Choose the more concise answer.

DO NOT DISTRIBUTE
| 3
GMAT IN CLASS MANUAL

11

Subject-Verb Agreement
While the rule for subject-verb agreement is simple, spotting errors is not always
so easy.

The Rule: A singular subject requires a singular verb. Plural


subjects require plural verbs.

Let’s try a few questions.

Verb:________________________________ Singular/Plural

Subject: ________________________________ Singular/Plural


Isolate the verb and its
subject and check for 1. Edward Abbey’s books, especially Desert Solitaire,
agreement.
in which Abbey recounts his adventures in the Utah
desert, employs wit to convey its author’s love of the
American West.
employs wit to convey its author’s love of the
American West
employs wit conveying their author’s love for the
American West
employs wit to convey the love of the American
West by the author
employ wit to convey their author’s love of the
American West
employ wit to convey its author’s love of the
American West

DO NOT DISTRIBUTE
4 |
Lesson 1

11
Verb:________________________________ Singular/Plural

Subject:_____________________________ Singular/Plural

2. Federally imposed restrictions on the amounts


that savings banks can pay small savers has
created difficulties for them as they are competing
with institutions offering unregulated investment
vehicles such as money market certificates.
has created difficulties for them as they are
competing with
has created a difficult circumstance for
savings banks competing against
have created a difficult circumstance when
they compete against
have created difficulties for savings banks
competing with
have created difficulties for them to
compete against

To spot subject-verb agreement errors, look for:

To fix subject-verb agreement errors:

DO NOT DISTRIBUTE
| 5
GMAT IN CLASS MANUAL

11

Verb Tense
Verb tense errors are usually easy to correct once you decide which tense to use.

The Rule: Choose the correct tense to match the clues in


the sentence.

Verbs:______________________________________________

Which action happens first?___________________________

1. Until Jackie Robinson made his debut in a Brooklyn


Dodgers uniform in 1947, African Americans were
prohibited to play for any Major League baseball team.
were prohibited to play
have been prohibited from playing
have been prohibited to play
had been prohibited to play
had been prohibited from playing

DO NOT DISTRIBUTE
6 |
Lesson 1

11
Verb:______________________________________________

Time Clue:_________________________________________

2. So far this year, twenty elected legislators had


opted not to run for reelection in the aftermath of
the Bathwater scandal.
had opted not to run for reelection
had opted to not run for reelection
have opted for not running for reelection
have opted not to run for reelection
have opted for not another run at reelection

To spot verb tense errors, look for:

To fix verb tense errors:

DO NOT DISTRIBUTE
| 7
GMAT IN CLASS MANUAL

11

Pronouns
Because pronouns are small words, you must read carefully to spot pronoun errors.

The Rules: Singular pronouns replace singular nouns, and


plural pronouns replace plural nouns. Also, a pronoun must
unambiguously refer to one noun.

Isolate the pronouns and the Pronoun:______________________________ Singular/Plural


nouns to which they refer.

Refers to:_____________________________ Singular/Plural

1. Each of the dogs now in the animal shelter had been


neglected by their former owner before they were
abandoned.
had been neglected by their former owner
before they were abandoned
was neglected by its former owner before it was
abandoned
was neglected by their former owner before they
were abandoned
had been neglected by its former owner before
it was abandoned
was abandoned, but before that they had been
neglected by their former owner

DO NOT DISTRIBUTE
8 |
Lesson 1

11
Pronoun:______________________________

Refers to:_____________________________

2.
Although aspirin irritates the stomach, it can be
avoided if the aspirin tablet is given a coating that will
not dissolve until the tablet reaches the intestine.
Although aspirin irritates the stomach, it
The irritation of the stomach caused by aspirin
The fact that aspirin causes irritation of the stomach
Aspirin causes stomach irritation, although it
Aspirin irritates the stomach, which

To spot pronoun errors, look for:

To fix pronoun errors:

DO NOT DISTRIBUTE
| 9
GMAT IN CLASS MANUAL

11

Back Up Plans
If you can’t identify an error in the underlined portion, try using the answer choices.

Can you ID one of the common errors in the underlined portion?


Yes No
Eliminate Choice (A) Can you ID an error from
the answer choices?
• 2/3 split
Eliminate any answers that • Differences in answers
obviously repeat the error. Yes No

POE with the remaining Starting at (B), use POE.


answer choices. • New errors.
• New errors. • Changes to the meaning
• Changes to the meaning of the sentence.
of the sentence.

Two left? Choose the more concise answer.

1.
Attempts to maintain the current level of funding for
museums, though impressive, has not resulted in the
How do the differences in the continuation of financial backing for the coming year.
answer choices help you to
find the error? Attempts to maintain the current level of funding
for museums, though impressive,
The attempt to maintain the current level of
funding for museums, though impressive,
Maintaining the current level of funding, though
an impressive attempt,
The impressive attempts to maintain the current
level of museum funding
Attempts to maintain the level of funding for
museums currently, though impressive,

DO NOT DISTRIBUTE
10 |
Lesson 1

11
2. Approximately seventy-five percent of the global
freshwater supply is stored in glaciers, which cover
roughly ten percent of land area.
is stored in glaciers, which cover roughly ten
percent of land area Even when you don’t find an
is stored in glaciers, which covers roughly error by checking a rule, you
ten percent of land area may be able to eliminate
is stored in glaciers, that covers roughly ten some answer choices.
percent of land area
are stored in glaciers, which cover roughly
ten percent of land area
are stored in glaciers, which covers roughly
ten percent of land area

Red Pencil Fever


Don’t eliminate something just because you don’t like the way it sounds. Often, the
correct answer won’t sound very good, and you don’t want to eliminate it hastily.

1. Depending on which scholar you consult, either


Daniel Defoe’s Robinson Crusoe, Henry Fielding’s
Joseph Andrews, or Samuel Richardson’s Pamela
is believed to have been the first English novel ever
written.
is believed to have been the first English
novel ever written
is believed as being the first English novel
ever written
are the English novels believed to be the first written
are the English novels which were believed
as the first written
are the first English novels ever believed to
be written

DO NOT DISTRIBUTE
| 11
GMAT IN CLASS MANUAL

MATH 1
Here are the basic facts for the math section:

• 31 questions in 62 minutes
• Approximately 16 Problem Solving and 15 Data Sufficiency
• Math scores range from roughly 6 to 51 (0–60 scale).
• The math score is a factor in the 200 to 800 overall scale.
• The math section is computer adaptive.

Math Introduction
Smart strategies can help you guess wisely on difficult questions or when time is short.

Trap Answers
Determine the trap answers for these questions:

1. The original price of an article was reduced by 25 percent. During


a special sale the new price was decreased by 10 percent. By
approximately what percent would the price now have to be
increased in order to restore the price of the article to its
original amount?
32.5%
35%
48%
65%
67.5%

2. A rectangular wooden crate has inside dimensions 3 meters


by 4 meters by 12 meters. What is the length, in meters, of the
longest, straight, inflexible rod of negligible diameter that can be
placed completely within the crate?
12
12.6
13
19
24

DO NOT DISTRIBUTE
12 |

GMAT InClass Manual Lesson 1.indd 12 7/2/18 6:47 PM


Lesson 1

11
3. The average (arithmetic mean) of x, y, and z is 50. What is the
sum of (4x + y), (3y + z), and (3z) ?
150
200
600
800
It cannot be determined from the information given.

Common Trap Answers Include


• Too Obvious
• Numbers in the Problem
• Simple Manipulations
• Partial Answers

Number Savvy
Number Savvy is your ability to do calculations in smart, time efficient ways.
Practice looking for savvy
ways to do calculations as
you do problems.
( −2.8 )(1.8 ) − (1.4 )( 2.4 )
=
4. 105

−0.008
−0.08
−0.8
0.8
8.0

DO NOT DISTRIBUTE
| 13
GMAT IN CLASS MANUAL

11

Ballparking
Ballparking means knowing the approximate size of the answer.

Use Ballparking to Eliminate Answers When


• You are stuck
• Running out of time

5. Paul drives from his apartment to his parents’ house and back along the
same route. On the trip to his parents’ house, he travels at an average
speed of 60 miles per hour. On the return trip, Paul drives at an average
speed of 80 miles per hour. Which of the following is the closest
approximation of Paul’s average speed, in miles per hour, for the round trip?
60.0
68.6
70.0
71.4
80.0

X Y

W Z

6. In the diagram above, the circle with center O is inscribed within square
WXYZ. If the square has area 400, what is the area of the circle?
20π
50π
75π
100π
400π

DO NOT DISTRIBUTE
14 |
Lesson 1

11

Data Sufficiency
Many test takers feel that data sufficiency questions are much harder than problem
solving questions. Once you are familiar with the format, however, data sufficiency
questions won’t seem so bad.

What the Answer Choices Mean


The first step is to understand the answer choices.

Answer A

1. What is the value of x ?


(1) x + 7 = 12 (A) 1 2
(2) x is an integer

Answer B

2. What is the value of x ?


(1) x is an integer (B) 1 2
(2) x + 7 = 12

Answer C

3. What is the value of x ?


(1) x2 = 25 (C) 1 2
(2) x ≥ 0

Answer D

4. What is the value of x ?


(1) x + 7 = 12 (D) 1 2
(2) x = 25

Answer E

5. What is the value of x ?


(1) x ≥ 0 (E) 1 2
(2) x is an integer

Basic Approach—AD/BCE
Based on statement (1),
which answers are possible?
6. What is the value of x ? A B C D E
(1) x + 7 = 12
(2)

DO NOT DISTRIBUTE
| 15
GMAT IN CLASS MANUAL

11

AD/BCE in Action

1. What is the value of x ?

(1) If x is subtracted from 60, the result is 20.

(2) If x is divided by z, the result is 20.

2. If x + y + z = 180, what is the value of x ?

(1) y = 75

(A) 1 2 (2) y + z = 141

(B) 1 2

(C) 1 2

(D) 1 2

(E) 1 2 x
3. If x and y are positive integers and =2, what is the value of x ?
y
(1) 1< y < 5

(2) y is odd.

3x
4. If = z, what is the value of x ?
2y
(1) yz = 30

(2) y =10 and z = 3

5. What is the value of x ?

(1) x + y = 20

(2) z – 3x = 45

DO NOT DISTRIBUTE
16 |
Lesson 1

sufficient insufficient

sufficient insufficient sufficient insufficient

sufficient insufficient

DO NOT DISTRIBUTE
| 17
GMAT IN CLASS MANUAL

11

Algebra vs. Arithmetic


Which of the following questions would you rather answer?

1. Max has a 10 dollar bill. He goes into a candy store and buys 3
pieces of candy that cost 50 cents each. How much change, in
dollars, does Max receive?

$9.50
$8.50
$7.00
$1.50
$0.50

2. Max has x dollars. He goes into a candy store and buys y pieces
of candy that cost z cents each. How much change, in dollars,
does Max receive?

x – yz

yz – x
x − yz

100
100x – yz
yz
x−
100

DO NOT DISTRIBUTE
18 |
Lesson 1

11

Plugging In
There is a way to turn algebra problems into arithmetic problems. It’s called
Plugging In. Just follow these steps:

Step 1: Assign a number to each variable in the problem.


Step 2: Work the problem step-by-step using the numbers you
chose. You should end up with a numerical answer (with no
variables left over) that answers the question in the problem.
Circle the answer. This is your target.
Step 3: Plug the number(s) you assigned to the variable(s) into the
answer choices. Choose the answer choice that matches
your target. Be sure to check all five answer choices.

3. Steven is three times as old as Jean, and Jean is two years


older than Ken. If Ken is k years old, then how old is Steven?

3k + 2
3k + 6
3k – 3
3k – 6
k −2

3

DO NOT DISTRIBUTE
| 19
GMAT IN CLASS MANUAL

11

Choose Good Numbers


Make your life easier by choosing good numbers that make the calculations easier.

4. The Amazing soft drink company interviewed c consumers for


What’s a good way to 2
a market-research study. The study found that of consumers
choose a number for c? 5
preferred Zing cola to Diet Zing cola. Of those who preferred Diet
1
Zing, preferred Caffeine Free Diet Zing. How many consumers,
6
in terms of c, did not prefer Caffeine Free Diet Zing?

c

11
c

10
7c

15
9c

10
10c

11

DO NOT DISTRIBUTE
20 |
Lesson 1

11
5. A machine working at maximum capacity can produce x radial
tires in an hour. How many tires can the machine produce if it
works continuously at maximum capacity for y minutes?

x

60 y
xy

60

60 x

y

60

xy

60xy

Avoid numbers that can make several answer choices match


your target.
• Do not use 0 or 1.
• Do not use numbers that appear in the question or in the
answer choices.
• Do not use the same number for more than one variable.

Choose numbers to make the calculations easy:

• If there are fractions in the question, choose a number that’s a


common multiple of the denominators. (Multiplying the
denominators together is an easy way to find a good number.)
• If a question involves different units, use a multiple or factor of
the conversion number.
• If a question involves percents, use 100 or a multiple of 100.

DO NOT DISTRIBUTE
| 21
GMAT IN CLASS MANUAL

11

Hidden Plug Ins


In some Plugging In problems, the variable is hidden. Once you find the variable,
you can plug into it.

1 1
1. Maggie pays of her monthly income for food, for utilities,
8 8
1 4
for student loans, and of the remainder for rent. If at the end
8 5
1
of each month Maggie puts of her remaining income into a CD
2
account, what portion of Maggie’s monthly income does she put

Most hidden plug-ins involve into the account?


fractions, percents, or ratios.
1

8
1

10
7

80
1

16
1

20

2. If 20 percent of the trees in a certain park are evergreens, and 40


percent of the non-evergreens are maple trees, and there are 75
percent as many oak trees as maple trees in the park, what percent
of the trees in the park are not maples, oaks, or evergreens?
10%
12%
20%
24%
25%

DO NOT DISTRIBUTE
22 |
Lesson 1

11

Plugging In The Answers (PITA)


Sometimes GMAC expects you to write and solve an equation. These problems
can also be solved by plugging in the answers.

1. Rob must make 5 payments on his student loans from


college. Each payment is twice the amount of the previous
payment. If the total amount he has to pay back is $1,550,
how much is Rob’s first payment?

$10 Work one step at a time.


$20 Write down each step as
you go.
$25
$50
$75

Follow these three steps:

Step 1: Identify what the question is asking and label the answers.
Step 2: Plug in the middle answer choice. Work one step of the
problem at a time.
Step 3: Eliminate answers that don’t match the condition in the
problem. Keep plugging in answers until you find one that
works.

DO NOT DISTRIBUTE
| 23
GMAT IN CLASS MANUAL

11

2. Mike has twice as many stamps as Jean has. After he gives Jean
6 stamps, he still has 8 more stamps than Jean does. How many
stamps did Mike have originally?

28
32
36
38
40

Choose Good Numbers


Sometimes it makes sense to test more likely answers first.

3. Julia is twice as old as her brother Paolo, who is five times as old
as their dog Winnie. In 10 years, Julia will be four times as old as
Winnie will be then. How old is Paolo?
25
27
30
33
35

DO NOT DISTRIBUTE
24 |
Lesson 1

11
Identifying Plugging In Problems
Plugging In is a great technique, and you should be on the lookout for
opportunities to Plug In.

Plug In
To recognize a basic Plug In problem, look for:

Hidden Plug In
To recognize a Hidden Plugging In problem, look for:

Plugging In The Answers (PITA)


To recognize a Plugging In The Answers problem, look for:

DO NOT DISTRIBUTE
| 25
GMAT IN CLASS MANUAL

11
Homework Review
Use this chart to note any questions you have from the reading or examples in the homework.

Page # Question # What que stion do you have ?

DO NOT DISTRIBUTE
26 |
Lesson 1

Practice
Plugging In
1. If k years from now George will be l years old, 5. If x = 3t – 1 and y = 12t2, what is y, in terms of x ?
how old was George m years ago?
l–k–m (x + 1)2
l + (k – m)
l+k–m 4(x + 1)2
k – (l + m) 3( x +1)2

k + (m – l) 4
2. If 2x = 4y = z, what is x – y, in terms of z ? 4( x +1)2

3
z
(x –1)2
4
z

2 x2 +2 x − 8
6. =
x2 − 6 x + 8
z
1

2z
–1

4z 1
x
3. A group of x people are in a room. One-third of 3
the people leave the room, and an additional 2 x+4
people enter the room. In terms of x, how many
x −4
people are in the room now?
x +8
 
 x − 1 + 2 x −8
 
 2
7. Copper pipe costs x cents per foot in 8-foot
2
x +2 lengths, and x + y cents per foot in shorter
3 lengths. What is the least possible price, in cents,
for 51 feet of pipe, in terms of x and y ?
3x + 2
51(x + y)
51x
3x + 6 48x + 3y
48(x + y)
x+2 51x + 3y

x+y x−y
4. What must be added to to obtain ?
2 2
–y
–x
2y
2x
y–x

DO NOT DISTRIBUTE
| 27
GMAT MANUAL

Hidden Plug Ins


1. Twenty-five percent of the residents of City Y 3. A parking garage has places for a certain
are 65 years old or older. Twenty percent of the 1
number of cars. If of the places are left
residents of City Y who are under the age of 5
65 are age 18 or younger. What percent of the 2
empty, and of the places are used by
residents of City Y are between the ages of 18 5
and 65 ? compact cars, non-compact cars take up what
15%
fraction of the filled spaces in the garage?
25%
45% 1
60%
3
75%
2
3
2. In an election, of the voters voted for 5
7 3 1
Candidate A, and of the remaining voters
5 2
voted for Candidate B. Of the voters who voted 3

1 5
for neither Candidate A nor Candidate B,
2 4
voted for Candidate C. What fraction of all the
5
votes were cast for Candidate C ?
4. Fred and Bobbie are book collectors, and Fred
1
has twice as many books as Bobbie does.
70 One-fourth of Fred’s books are signed by the
4 authors, and three-fifths of Bobbie’s books are
signed by the authors. If Fred and Bobbie
35
combine their collections, what fraction of the
9 books are signed by the authors?

35
11
13
30
35
9 29

60
10
23

30
17

20

It cannot be determined from the


information given.

DO NOT DISTRIBUTE
28 |
Lesson 1

5. Computer retailer A is selling a computer at a 6. At a certain university, 3 out of every 5 students


20 percent discount from its suggested retail live in an on-campus dormitory. If one out of
price. Computer retailer B promises to match every 10 students who do not live in an
this price and then discount it a further on-campus dormitory lives in a university-owned
10 percent. Computer retailer B’s final price is apartment, what fractional part of the student
what percent of the suggested retail price? body does not live in either an on-campus
dormitory or a university-owned apartment?
75%
72% 7
70%
10
68%
1
28%
2
9

25
3

10
7

25

DO NOT DISTRIBUTE
| 29
GMAT MANUAL

Plugging In The Answers

1. If 2x2 – 2x – 12 = 0 and y2 – 5y + 6 = 0 when 5. Judy is 26 years old and Diane is 5 years old. In
x = –y, then what is the value of x ? how many years will Judy be twice as old
as Diane?
–3
16
–2
19
0
21
2
24
3
26
2. Several people rented a car for $30. If there had
6. Pat has a pocket full of quarters, dimes, and
been one more person in the group, it would
nickels. He takes 6 coins out of his pocket that
have cost each person $1 less. How many
total $0.70. If there are only two denominations
people were in the group originally?
of coins among the 6 coins in Pat’s hand, how
5 many nickels is he holding?
6 2
10 3
12 4
15 5
3. This year, half of the clients of a certain 6
consulting firm ended up paying the firm exactly
7. On any given Sunday, 75 percent of the people
$22,000 each, while the other half ended up
in a city who own TV sets turn them on. Thirty
paying the firm exactly $33,000 each. If the firm
percent of the people who turn their TV sets on
received a total of $275,000 from its clients, how
watch football. If 9,900 people watch football
many clients does the consulting firm have?
on Sunday, how many people in the city own
9 TV sets?
10 75,000
11 44,000
12 30,000
13 24,000
4. A certain bakery produces only chocolate and 20,000
vanilla cupcakes. If the bakery sells 160
cupcakes per day, and 26 more chocolate 8. Fredrico earns a salary of $350 in a certain week
cupcakes than vanilla cupcakes are sold per day,
how many chocolate cupcakes does the bakery at an appliance store. In addition, he receives
sell per day? 2
a bonus equal to of the amount of his total
5
54
sales that exceed $1,500. If he earns $770, what
67
82 were Fredrico’s total sales for the week?
93 $1,600
106 $1,750
$1,925
$2,225
$2,550

DO NOT DISTRIBUTE
30 |
Lesson 1

Answers and Explanations

Plugging In Hidden Plug Ins


1. A Plug in k = 3, l = 10, and m = 5. If George will be 1. D Plug in 100 for the number of residents. There are
10 in 3 years, he’s 7 now. Thus he was 2 (target 25 people who are 65 or older, and 15 who are 18
answer) five years ago. or younger. There are 60 people in between.

2. A Plug in. Be sure to choose numbers that make


2. B Use the denominators to find the ideal number
the equation in the question true. Let x = 6,
y = 3, and z = 12. In that case, x – y = 6 – 3 = 3 to plug in for the number of voters: 70. From the
(target). Only choice (A) works.
data given, 30 choose Candidate A and 24 vote
3. B Plug in. The calculations will be easier if you make for Candidate B. Of the remaining 16, 8 choose
x a multiple of 3; let x = 12. If a third of the people
leave, that leaves 8; if 2 come in, your new total is Candidate C. The fraction of voters that vote for
10 (target answer). 8 4
C is , which reduces to .
70 35
x+y
4. A If you plug in x = 3 and y = 5, then = 4 and 3. C If there are 25 places in the garage, then there are
2
x−y
= −1 . You have to add –5 (target answer) 5 empty places and 10 of the places have compact
2
to 4 in order to get –1. cars in them. If you picked (B), you may have
misread the question; the question involves the
5. D Plug in for t first because both x and y are based
on the value of t. If t = 2, then x = 5 and y = 48. fraction of non-compact cars in the filled spaces.
The question wants the value of y, so 48 is your
target answer. There are 10 non-compact cars and 20 filled
spaces, so the fraction is 1 .
9+ 6 −8 2
6. D Plug in x = 3, so the fraction becomes ,
7 9 − 18 + 8 4. A Get rid of (E), which is a trap answer. Give Fred
which reduces to , or –7 (target answer). 20 books and Bobbie 10 books; 5 of Fred’s books
−1
are signed and 6 of Bobbie’s are signed. The total
7. E The 8-foot lengths are less expensive, so you want is 11 out of the 30 books.
to get as many as possible: six. If you get six 8-foot
lengths, that leaves 51 – 48 = 3 feet at the more 5. B Plug in $100 for the suggested retail price. Retailer
expensive rate. Let x = 2 and y = 3; the first 48 feet A sells the computer for $80, and retailer B sells it
cost $96, and the remaining 3 feet cost $5 each, for $72. This second price is a 28 percent discount
or $15. Your target answer is $111. from the original price. Choice (C), which is too
obvious, is a trap.

6. C Plug in for the number of students. Notice that


3
3 out of 5 is the same thing as . If there are 50
5
students, then 30 live in a dorm and 2 live in an
apartment. There are 18 students remaining, and
18 9
reduces to .
50 25

DO NOT DISTRIBUTE
| 31
GMAT MANUAL

Plugging In The Answers


1. B The answers represent possible values for x. Start 5. A The answer choices represent the number of years
with (C), which turns out to be incorrect because in which Judy will be twice as old as Diane. For
0 doesn’t work in the first equation. Next, plug –2 choice (C), in 21 years, Judy is 47 and Diane is
into the first equation [ 2(–2)2 – 2(–2) – 12 = 0]. 26. Judy is less than twice as old as Diane, so (C)
Now, plug 2 (remember x = –y) into the second is too big. Eliminate choices (C), (D), and (E).
equation [(2)2 – 5(2) + 6 = 0]. For choice (A): In 16 years, Judy is 42 and Diane
is 21, making Judy twice as old as Diane.
2. A The answers represent the number of people in
the original group. For choice (C): 10 people 6. C The answer choices represent the number of
pay $3 each, and 11 people pay $2.73. So, the nickels. For choice (C): If Pat has 4 nickels, the
difference is not $1. For (A): 5 people pay $6 each, other two coins must be quarters: 4 × $0.05 = $0.20,
and 6 people pay $5 each, which is $1 less. and 2 × $0.25 = $0.50.

3. B The answers represent the number of clients. Get 7. B The answers represent the number of people in the
rid of all the odd answers because the first step of city who own TV sets. For choice (C), if 30,000
the problem divides the clients into two halves. own sets, then 22,500 of those sets are turned on
Only (B) and (D) are left; try either one. For and 6,750 are tuned to football. Since 6,750 is less
choice (B): 10 clients means that 5 pay $22,000, than the 9,900 who are supposed to be watching
for a total of $110,000, and the other 5 pay football, choice (C) is too small. Eliminate choices
$33,000, for a total of $165,000. Since the total (C), (D), and (E). For choice (B), if 44,000 people
received by the firm is $275,000, choice (B) is own sets, then 33,000 of those sets are turned on
correct. and 9,900 have the football game on.

4. D The answers represent the number of chocolate 8. E The answer choices represent Fredrico’s total
cupcakes sold by the bakery each day. For choice sales for the week. If you noted that Fredrico’s
(C), if the bakery sells 82 chocolate cupcakes, it bonus is only paid on the portion of his sales
sells 82 – 26 = 56 vanilla cupcakes. The bakery that exceed $1,500, you probably realized that
sells a total of 82 + 56 = 138 cupcakes, which choices (A), (B), and (C) are all too small.
is less than the 160 cupcakes the bakery is Between choices (D) and (E), choice (E) is a little
supposed to sell. Eliminate choices (A), (B), and easier to work with so start there. For (E), if the
(C). For choice (D), if the bakery sells 93 chocolate total sales are $2,550, then the bonus is paid on
cupcakes, it sells 67 vanilla cupcakes and a total $2,550 – $1,500 = $1,050. The bonus amount is
of 93 + 67 = 160 cupcakes. $420, which gets added to Fredrico’s base salary of
$350 to make his total earnings for the week $770.

DO NOT DISTRIBUTE
32 |
Lesson 2

Lesson 2
22

Sentence Correction 2
The previous lesson covered three of the six most commonly tested errors. This
lesson focuses on the remaining three: misplaced modifiers, parallelism, and idioms.

Review
1. Each team of business students are responsible for
creating a marketable business plan.
are responsible for creating a marketable
business plan
are responsible to create a marketable
business plan
is responsible for creating a business plan
that is able to be marketed
is responsible for creating a marketable
business plan
is responsible to create a marketable
business plan

Can you ID one of the common errors in the underlined portion?


Yes No
Eliminate Choice (A) Can you ID an error from
the answer choices?
• 2/3 split
Eliminate any answers that • Differences in answers
obviously repeat the error. Yes No

POE with the remaining Starting at (B), use POE.


answer choices. • New errors.
• New errors. • Changes to the meaning
• Changes to the meaning of the sentence.
of the sentence.

Two left? Choose the more concise answer.

DO NOT DISTRIBUTE
| 33
GMAT IN CLASS MANUAL

22
Misplaced Modifiers
You learned about modifiers in the pre-class assignment. Let’s look at how they will
be tested on the GMAT.

The Rule: A word or phrase that describes something should go


right next to the thing it modifies.

Modifying Phrase:______________________________________________

Modifies?_____________________________________________________

1. Eaten in the Mediterranean countries, northern


The modifier is usually an
introductory phrase that Europeans viewed the tomato with suspicion, for they
starts with a participle. assumed it had poisonous properties because of its
relationship to deadly nightshade.
northern Europeans viewed the tomato with
suspicion, for they
northern Europeans were suspicious of the
tomato, and they
the tomato was viewed with suspicion by
northern Europeans, who
the tomato was suspicious to northern
Europeans, and it was
the tomato was viewed with suspicion by
northern Europeans, it being

DO NOT DISTRIBUTE
34 |
Lesson 2

22
Sometimes the introductory phrase is underlined. In this case, you can fix the
error in two ways:

• Rewrite the phrase so it describes the subject after the comma.


• Change the phrase into a clause. Since a clause has its own
subject, there will be no confusion about what it describes.

Modifying Phrase:____________________________________________

Modifies?____________________________________________________

2.
Sold over the counter at the turn of the century, Always think in terms of
what the phrase modifies
the government now prohibits the sale of cocaine
grammatically.
derivatives.
Sold over the counter at the turn of the century
While sold over the counter at the turn of the century
Being sold over the counter at the turn of the century
Although they were selling them over the counter at
the turn of the century
Although they were sold over the counter at the turn
of the century

DO NOT DISTRIBUTE
| 35
GMAT IN CLASS MANUAL

22
Modifying Phrase:____________________________________________

Modifies?____________________________________________________

3.
Controlling most inroads to business ventures in
Europe, economists argue that the U.S., with its
diminished economic leverage there, now has reason
to fear the European Common Market.
Controlling most inroads to business ventures
in Europe, economists argue that the U.S.,
with its diminished economic leverage there,
now has reason to fear the European Common
Market.
Controlling most inroads to business ventures in
Europe, the diminished economic leverage of
the U.S. there is, according to economists, one
reason to fear the European Common Market.
Because it controls most inroads to business
ventures in Europe, a place where the
U.S. have diminished economic leverage,
economists argue that they now have reason
to fear the European Common Market.
Because it controls most inroads to business
ventures in Europe, economists argue that
the U.S.’s diminished economic leverage is a
reason for the U.S. to fear the actions of the
European Common Market.
Economists argue that the U.S., with its
diminished economic leverage in Europe, now
has reason to fear the actions of the European
Common Market, which controls most inroads
to business ventures in Europe.

To spot misplaced modifier errors, look for:

To fix misplaced modifier errors:

DO NOT DISTRIBUTE
36 |
Lesson 2

22
Parallel Construction
Parallel construction means that words in a list share the same part of speech or that
phrases in a list have the same structure.

The Rule: In a list of two or more things, each item


in the list must use the same grammatical structure.

List Item 1:_____________________ Part of Speech:_______________


Lists are marked by
Conjunction: conjunctions. The most
common conjuctions used on
the GMAT to test parallelism
are and, but and
List Item 2:_____________________ Part of Speech:_______________ not only,,, but also.

1. The two main goals of the Eisenhower presidency


were a reduction of taxes and to increase military
strength.
a reduction of taxes and to increase military strength
to reduce taxes and an increase in military strength
to reduce taxes and to increase military strength
a reduction and an increase in taxes and military strength
taxes being reduced and military strength being increased

DO NOT DISTRIBUTE
| 37
GMAT IN CLASS MANUAL

22
List Item 1:_____________________ Part of Speech:_______________

Conjunction:

List Item 2:_____________________ Part of Speech: _______________

2. The Department of Education has concluded that if children


learn to read and did math from an early age, they will
require less remedial work in later school years.
did math from an early age, they will require less
remedial work in later school years
did math from an early age, they require in later
school years less remedial work
do math from an early age, they require less
remedial work in later school years
do math from an early age, they will be required in
later school years to do less remedial work
did math from an early age, less remedial work will
be required in later school years

DO NOT DISTRIBUTE
38 |
Lesson 2

22
List Item 1:____________________ Part of Speech:______________

List Item 2:____________________ Part of Speech:______________

Conjunction:

List Item 3:___________________ Part of Speech:_____________

3. In response to losing a significant part of its market


share to a competitor, the soft drink company
has cut costs by withholding executive bonuses,
changed their advertising agency, and have
redesigned the company logo.
changed their advertising agency, and have
changing their advertising agency, as well as
has changed its advertising agency, and
and changed its advertising agency, and has
changed its advertising agency, and

To spot parallel construction errors, look for:

To fix parallel construction errors:

DO NOT DISTRIBUTE
| 39
GMAT IN CLASS MANUAL

22 Comparisons
Just remember “apples to apples and oranges to oranges.”

The Rule: Only similar items can be compared.

Item 1 Comparison Word Item 2

1. The rules of written English are more stringent than


spoken English.
Look for comparison words. than spoken English
Common comparison words as spoken English
used on the GMAT include: than those of spoken English
Like as those of spoken English
Unlike so than those of spoken English
Similar to
Compared to
More...than
Less...than
Different...from
As

DO NOT DISTRIBUTE
40 |
Lesson 2

22

Item 1 Comparison Word Item 2

2. Scientists now suspect that imperial mammoths


of the Pleistocene period were wiped out not by
human predation, as were mammoths of earlier
periods, but by a sudden and extreme change in
climate.
as were mammoths of earlier periods
like that in an earlier period
as in earlier periods
as were earlier periods
like the mammoths in an earlier period

To spot comparison errors, look for:

To fix comparison errors:

DO NOT DISTRIBUTE
| 41
GMAT IN CLASS MANUAL

22
Idioms
As you learned from the pre-class assignment, idioms are fixed expressions. Knowing
your idioms can help you to find errors in the underlined portion and to eliminate
answer choices.

The Rule: Know your idioms!

1. The establishment of land trusts in New England


has been praised not because of it restricting
The use of improper idioms
is a good way to identify an overdevelopment but because it allows the inheritance
incorrect answer choice. by future generations of the property.
not because of it restricting overdevelopment
but because it allows the inheritance by future
generations of
because it not only is restricting
overdevelopment and also allows the
inheritance by future generations of
because it does not only restrict
overdevelopment, it also allows future
generations for inheriting
because of not only the restrictions on
overdevelopment but also because future
generations can inherit
not only because it restricts overdevelopment
but also because it allows future generations
to inherit

DO NOT DISTRIBUTE
42 |
Lesson 2

22
2. Dermatologists are trained to distinguish an
irritation, which results from prolonged exposure
to the offending substance, from a true allergic
reaction, which manifests itself immediately upon
contact with the allergen.
Errors rarely occur in
an irritation, which results from prolonged appositive phrases. Try
exposure to the offending substance, from a reading the sentence without
true allergic reaction, which manifests itself the appositive to spot an
error.
immediately upon contact with the allergen
an irritation, resulting from prolonged
exposure to the offending substance, and
a true allergic reaction, manifesting itself
immediately upon contact with the allergen
between an irritation, which results from
prolonged exposure to the offending
substance, or a true allergic reaction,
which manifests itself immediately upon
contact with the allergen
between an irritation, resulting from
prolonged exposure to the offending
substance, from a true allergic reaction,
manifesting itself immediately upon
contact with the allergen
a true allergic reaction, which manifests itself
immediately upon contact with the allergen,
and an irritation, which results from prolonged
exposure to the offending substance

To spot idiom errors, look for:

To fix idiom errors:

DO NOT DISTRIBUTE
| 43
GMAT IN CLASS MANUAL

22
Math 2

Pieces of the Puzzle


When you start a data sufficiency question, always take stock of what you
know from the question stem. This method is particularly helpful for value data
sufficiency questions.

What Do You Know?_________________________________________

What Do You Need?__________________________________________

1. Each week, Jesse is paid x dollars per hour for the first 40 hours
and 1.5x dollars for each additional hour he works that week. How
much did he earn last week?
(1) Last week, Jesse worked a total of 48 hours.
(2) Jesse earns $36 more when he works a total of 42 hours in a week
than when he works 40 hours.

What Do You Know?_________________________________________

What Do You Need?__________________________________________

2. At a certain bakery, cherry pies cost $15. How many cherry pies
did the bakery sell on Wednesday?
(1) On Tuesday, the bakery’s cherry pie sales totaled $225.
(2) On Wednesday, the bakery’s cherry pie sales were $75 more than
Tuesday’s cherry pie sales.

DO NOT DISTRIBUTE
44 |
Lesson 2

22
What Do You Know?_________________________________________

What Do You Need?__________________________________________

3. A certain bakery sells only cherry pies and apple pies, which
cost 20% more than cherry pies. If the bakery’s pie sales on
Wednesday totaled $111, how many cherry pies were sold?
(1) Cherry pies cost $15 each.
(2) The bakery sold a total of 7 pies on Wednesday.

What Do You Know?_________________________________________

What Do You Need?__________________________________________

4. How many square tiles, each with a side of length n,


are needed to completely cover a rectangular floor with
dimensions l and m ?
(1) n is a factor of both l and m.
(2) n is one-half of l and one-third of m.

DO NOT DISTRIBUTE
| 45
GMAT IN CLASS MANUAL

22
Yes-No Data Sufficiency
You’ve already seen data sufficiency problems that ask about a value. Other data
sufficiency problems ask questions that require a “yes” or “no” answer.

What’s different about the 1. Does x = 10 ?


way this question is phrased?
(1) Ten percent of x is 1.
(2) One-fifth of x is an even prime number.

2. Has Brand X been purchased by more than 50 percent of the


5,000 people in Moab?
(1) Brand Y has been purchased by 75 percent of the people in Moab.
(2) Brand X has been purchased by exactly 3,000 people in Moab.

Has the answer changed?


Why? 2. Has Brand X been purchased by more than 50 percent of the
5,000 people in Moab?
(1) Brand Y has been purchased by 75 percent of the people in Moab.
(2) Brand X has been purchased by exactly 2,000 people in Moab.

Now, has the answer


changed? Why?
2. Has Brand X been purchased by more than 50 percent of the
5,000 people in Moab?
(1) Brand Y has been purchased by 75 percent of the people in Moab.
(2) Brand X has been purchased by between 2,000 and 3,000 people
in Moab.

DO NOT DISTRIBUTE
46 |
Lesson 2

22
Plugging In for Yes-No Data Sufficiency
When Yes-No data sufficiency questions involve variables, Plugging In can help
you determine whether a statement is sufficient.

3. Is x + 7 an odd integer?

(1) 2x + 3 is an odd integer.

(2) x + 11 is an even integer.

Pick numbers based on the statement.


No

Do the numbers satisfy the statement?


Yes
Answer the question with the numbers.

Pick new numbers based on the statement.


Try a different type of number.
No

Do the numbers satisfy the statement?


Yes
Answer the question with the numbers.
Same Different
Answers? Answers?
Statement Statement
is is
Sufficient Insufficient

DO NOT DISTRIBUTE
| 47
GMAT IN CLASS MANUAL

22
4. Is x a positive number?

(1) x + 6 > 2

(2) x2 > 25

5. If x is an integer such that 9 < x < 100, is x prime?

(1) Both the tens digit and the units digit of x are prime.

(2) x + 6 is prime.

DO NOT DISTRIBUTE
48 |
Lesson 2

22
Keep Plugging Away
Some problem solving questions ask which answer choice must be true. These
problems can be solved using Plugging In.
What’s a good type of
1. If a and b are distinct positive integers, and a is even, then number to try the second
which of the following must also be even? time you plug in?

2(a + b) – 3
(a – b) + 2
a+b–1
a–b
ab – 2

For must-be questions:


• Plug in numbers for the variables. Eliminate choices that
are false.
• Plug in again using the most different type of numbers
allowed by the conditions in the problem.
• Keep plugging away until only one choice remains.

2. If p and q are integers, such that p < 0 < q, and s is a


nonnegative integer, which of the following must be true?

p2 < q2

p+q=0

sp < sq

sp ≠ sq
p
<s
q

DO NOT DISTRIBUTE
| 49
GMAT IN CLASS MANUAL

22
3. If x, y, and z are nonzero integers and x > yz, which of the
following must be true?
x
I. >z P
y
‘Must Be’ questions often
test one of these differences x
II. >y
even vs odd z
integers vs fractions x
III. >1
positive vs negative yz

None of the above


I only
III only
I and II only
I, II, and III

DO NOT DISTRIBUTE
50 |
Lesson 2

Homework Review 22
Use this chart to note any questions you have from the reading or examples in the homework.

Page # Question # What que stion do you have ?

DO NOT DISTRIBUTE
| 51
GMAT IN CLASS MANUAL

Practice

Yes-No Data Sufficiency Fundamentals Practice


1. If –2 < a < 11 and 3 < b < 12, then which of
1. Does x + y = xy ?
the following is NOT true?
(1) x is neither a positive integer nor a negative 1 < a + b < 23
integer. –14 < a – b < 8
(2) y is neither a positive integer nor a negative
–7 < b – a < 14
integer.
1 < b + a < 23
2. Does positive integer a equal 5 ? –24 < ab < 132

(1) a is not a factor of 6006. 1 2


2. If <z< , then what is the value of z ?
(2) 5 is the largest divisor of a. 3 3

3. If x is a positive integer, is x a multiple of 5 ? (1) When positive integer x is divided by 2, the


result is z.
x (2) When positive even integer y is divided by
(1) 5 is a factor of .
2 12, the result is z.
(2) 3x + 5 is a multiple of 5. 3. On the first of the year, James invested x
dollars at Proudstar bank in an account that
4. Is m3 an integer? yields 2% in interest every quarter year. At
the end of the year, during which he made no
(1) m2 is an integer. additional deposits or withdrawals, he had y
(2) m is an integer. dollars in the account. If James had invested
the same amount in an account which pays
5. Does s = t ? interest on a yearly basis, what must the
interest rate be for James to have y dollars at
(1) s = t the end of the year?
(2) s is both a factor and multiple of t.
2.04%
6. Is m a multiple of 6 ? 6.12%
8%
(1) More than 2 of the first 5 positive integer 8.25%
multiples of m are multiples of 3.
10%
(2) Fewer than 2 of the first 5 positive integer
multiples of m are multiples of 12.

7. Is the tens digit of two-digit positive integer p


divisible by 3 ?

(1) p – 5 is a multiple of 3.
(2) p – 11 is a multiple of 3.

DO NOT DISTRIBUTE
52 |
Lesson 2

4. All votes cast in a recent presidential election 6. At the same time that Rick opened an account
were for either the incumbent or the challenger. with $150 at Bank A, Mary Jane opened an
The challenger received 5.4 million votes and account at Bank B with $150. Mary Jane’s
the incumbent received 5 million. If after a account has 10% simple annual interest and
recount of the votes and the addition of Rick’s gives 12% annual interest compounded
previously uncounted absentee ballots, the quarterly. If neither Rick nor Mary Jane make
incumbent had 5.2 million votes while the any additional deposits or withdrawals, then
challenger had 5.4 million, then the percentage what percent more does Rick have in his
of the total number of votes that were for the account after a year?
challenger
150(1.03)4 −150(1.10)
decreased approximately 10%
150(1.10)
decreased approximately 1%
neither increased nor decreased 150(0.12)(2) – 150(0.10)(2)
increased approximately 1%
150(0.12)(4) − 150(0.10)
increased approximately 2%
150(0.10)

150(1.03)4 −150(1.10)
5. If x and y are greater than zero, then what is the
value of x2y ? 150(1.12)4 −150(1.10)
1 1 1 1
(1) y = + + + 150(1.10)
2 4 8 16
(2) x has exactly two distinct positive 7. If P is a set of consecutive integers, is there an
factors, one of which is even. even number of integers in set P ?

(1) The sum of the integers in set P is 0.


(2) The product of the integers in set P is 0.

8. A quarterly interest rate of 5 percent over a


12–month period is equal to an annual interest
rate of approximately
60%
33%
22%
20%
15%

DO NOT DISTRIBUTE
| 53
GMAT IN CLASS MANUAL

Answers and Explanations


Yes-No Data Sufficiency

1. E Since this is a yes-no question, you should plug So, AD. For statement (2), x could be 5 because
15 + 5 is 20, which is a multiple of 5. Since 5
in to evaluate the statements. For statement
1 is a multiple of 5, the answer to the question is
(1), x and y could both be in which case ‘yes’. The next value of x that satisfies statement
2
the answer to the question is ‘no’. However, 2 is 10, which also gives an answer of ‘yes’ to
the statement. You can quickly determine that
for statement (1), x and y could also both be the only values of x that satisfy statement (2) are
0, in which case the answer to the question is multiples of 5. Again, the answer to the
question is always ‘yes’.
‘yes’. So, BCE. For statement (2), you can use
1 4. B To answer this question, we need to know
the same values: if x and y are both then the
2 whether m is an integer. For statement (1), m
answer to the question is ‘no’ but if both x and
could be 2 because its square, 4, is an
y are 0, then the answer to the question is ‘yes’. integer. In this case, the answer to the question
Cross off B. Since you used the same values to is ‘yes’. However, m could also be 2 because
its square 2 is an integer. In this case, however,
evaluate both statements, you can also use those the answer to the question is ‘no’. So, BCE.
same values to produce both an answer of ‘yes’ For statement (2), m could be 1, 4, 9, 16, etc.
and an answer of ‘no’ when the statements are Note that only perfect squares satisfy statement
(2). These numbers always produce an answer
combined. So, eliminate (C). of ‘yes’ to the question because the cube of any
integer is also an integer. So, statement (2) is
2. B Plug in. For statement (1), a could be 8, which
is not a factor of 6006. If a = 8, then the answer sufficient.
to the question is ‘no’. However, if a = 5, which
5. B Statement (1) is insufficient because s and t
is also not a factor of 6006, then the answer to
could both be 1, which would be equal, or s
the question is ‘yes’. So, BCE. For statement (2),
could be 4 and t could be 2. So, BCE. What we
you need to remember that the greatest factor
are given in Statement (2) answers the question
(aka divisor) of any integer greater than 1 is the
because the only number that can be both a
integer itself. So, statement (1) tells you that
factor and a multiple of t is t, thus s must be
a = 5.
equal to t.
3. D For statement (1), x could be 10 because 10
6. B Start with Statement (1). Multiples of 6
divided by 5 is 5 and 5 is a factor of 5. Now,
(6, 12, 18, 24, and 30) would yield an answer of
that you’ve found a number that satisfies the
‘yes’. Multiples of 3 (3, 6, 9, 12, 15) would yield
statement, use that number to answer the
a ‘no’. Thus Statement (1) is insufficient.
question. Is 5 a multiple of 5? Yes. Next, you’d
Eliminate AD. Approach Statement (2) the
want to see if you could get an answer of ‘no’.
same way. The information we are given in
However, only numbers such as 20 and 30,
this statement doesn’t allow us to use 6 or any
which are multiples of 5 satisfy statement (1).
multiple of 6 for m, thus answering the question
These numbers always give an answer of ‘yes’ to
with a definitive ‘no!’.
the question. The first statement is sufficient.

DO NOT DISTRIBUTE
54 |
Lesson 2

7. E We need to plug in here. For Statement (1), p 4. B The simplest way to solve this question is just
could be 38, which would produce a ‘yes’, or 47, to estimate. The original total number of votes
which would produce a ‘no’. For is 10.4 million. It increases to 10.6 million after
Statement (2), p could still be 38, which would the recount, but the challenger still has the same
produce a ‘yes’, or 47, which would produce a number of votes. Since the total increased, but
‘no’. Since we were able to use the same the challenger’s number remained the same, the
numbers in each statement, we know that they challenger’s percentage of the total vote
aren’t sufficient together, either. The correct decreased, so the answer must be (A) or (B).
answer is (E). Since the change in the total number of votes
was very small (only 200,000 out of about 10
Fundamentals million, or a change of about 2 percent), the
percent of the total vote that the challenger
1. C Whenever you are asked to combine two
received must have changed only slightly, so the
inequalities, you should just do all four
answer is (B).
possible combinations of the endpoints for
whatever operation is specified. In this instance, 5. C Statement (1) only tells us about y so it’s not
unfortunately, each answer choice gives us a sufficient by itself, leaving us choices BCE.
different calculation to do. That’s not hard, just Statement (2) tells us only about x, so it’s also
very time-consuming. For example, to check to insufficient alone, thus we eliminate (B). When
see if answer (E) were true, we would have to the two statements are taken together,
Statement (1) allows us to calculate the value of
multiply to combine the given ranges. y and Statement (2) tells us that the value of x is
–2 × 3 = –6, –2 × 12 = –24, 11 × 3 = 33 and 2 (2 is the only number with exactly two
11 × 12 = 132. Take the largest and smallest factors, only one of which is an even positive),
values (132 and –24), and you can see that so the correct answer is (C).
answer (E) is true. Move on to the next answer
choice and try that one. 6. A The problem wants us to find the formula for
two different compound interests and then do
1 the percent change formula, which would just
2. D Statement (1) tells us that z must be , since
2 be time-consuming and brutal. The easier way
1 2 out is to start to eliminate choices as soon as
the only fraction between and you can
3 3 possible. If we know the compound interest
1 formula: (principle)(1 + interest rate)# of compound-
get by dividing an integer by 2 is . Statement
2 ing periods
, then we should know that we need to
(1) is sufficient; so AD. Statement (2): If y is an raise the amount to the power of the number of
periods, not multiply by the number of
even integer and z must be a fraction between periods. This eliminates (B) and (C). Second, if
1 and 2 , then y must be 6, so z must be 1 . we pay attention to how often Rick’s interest is
2 compounded, we should note that his account
3 3
compounds every quarter, which means that
his 12% yearly is actually a 3 percent quarterly
3. D Just estimate. 2 percent interest per quarter interest rate. This means that we should have
would be 8 percent annually if the account (1.03), not (1.12). Eliminate (D) and (E). (D)
didn’t earn interest on the money gained each also does not actually do the percent more for
quarter. But it does. So the correct answer which the problem asks; it only finds the
should be a little more than 8 percent. difference in their amounts after the 2 years.

DO NOT DISTRIBUTE
| 55
GMAT IN CLASS MANUAL

7. A Statement (1) lets us know that the sum of the 8. C Just ballpark questions like these. Compound
consecutive integers in the set is 0. Since we’re interest is always a little more than simple
dealing with consecutive integers here, we know interest. Simple interest at 5 percent per quarter
would be 20 percent. Compound interest would
some have to be positive and some have to be be a little more than 20 percent. The only
negative, and the positive and negative integers possible answer is (C).
have to balance out (e.g., –1, 0, 1 would be a
set that would work). The only way to have
this balance of positive and negative integers
is to have an odd number of integers, since 0
must also be included. Hence, Statement (1) is
sufficient, and the answer must be (A) or (D).
Statement (2) only lets us know that one of the
numbers in the set is 0, but sheds no light on
whether there is an odd or an even number of
integers in the set.

DO NOT DISTRIBUTE
56 |
Lesson 3

Lesson 3 33

Math 3

Inequalities and Absolute Values


Data sufficiency problems often include inequalities, absolute values or both.

There are a variety of ways to


1. Is x > y ? tackle this problem.

(1) x > y

1 1
(2) x− y >0
4 3

10 − x
2. If < −2 x, which of the following must be true?
3


I. 2 < x

II. x −5 ≥ 7

x −1
III. >1
x
I only
II only
III only
II and III
I, II, and III

DO NOT DISTRIBUTE
| 57
GMAT IN CLASS MANUAL

33 x
3. If xy ≠ 0, is >0 ?
y
What are the options for
evaluating these x y − xy
statements? (1) <0
x+y

(2) x + y = x + y

Roots and Exponents


There are six rules to know about exponents. You can solve any GMAT question using
these six rules.

412 − 411
1. =
643
4

643
1

163

16

48

64

DO NOT DISTRIBUTE
58 |
Lesson 3

2. If y = (mx)2, then y2m =


33
m2x
m4x
m4x + 1
m4x + 2
m5x

3. Which of the following pairs of numbers are reciprocals? What’s the defintion of a
reciprocal? How does that help
1
7 to solve this problem?

I. 72 and
7

II. 7−2 and 7

III. 73 and 73

I only
II only
I and II only
II and III only
I, II, and III

DO NOT DISTRIBUTE
| 59
GMAT IN CLASS MANUAL

x
m
33 4. If m > 0, what is the value of
y
?
m
(1) m = 3

(2) x = y

5. If x ≠ 0, is x –y > 0 ?

(1) x > 0

(2) y < 0

Tips for Working with Exponents


• Look for common bases.
• Try factoring if you can’t see a way to apply one of the
exponent rules.

DO NOT DISTRIBUTE
60 |
Lesson 3

Quadratics
33
Recall that a quadratic equation is any equation of the form ax2 + bx + c = 0.
A root of an equation is just another word for a solution of an equation.

1. If in the equation x2 – 2x – c = 10, x is a variable and c is a


constant, what is the value of c ?

(1) (x + 3) is a factor of the equation x2 – 2x – c = 10.

(2) 5 is a root of the equation x2 – 2x – c = 10.

( x 2 − y 2 )2
2. If xy ≠ 0, which of the following is equivalent to ? Look for the common
x 2 + 2 xy + y 2 quadratics.

x2 – y2

1

x − y2
2

x2 – 2xy + y2

x2 + 2xy + y2

DO NOT DISTRIBUTE
| 61
GMAT IN CLASS MANUAL

3. If x is an integer, is x even?
33
(1) x 2 – y2 = 0

(2) x2 + y2 = 18

Functions and Sequences


To solve function questions, you typically just need to follow the directions.

1. If [a] is the least integer greater than or equal to a, then


[–4.5] + [4.5] =
[–1.5]
[–0.5]
[0]
[0.5]
[1.5]

DO NOT DISTRIBUTE
62 |
Lesson 3

33
xy
2. If a and b are distinct positive integers and x ⊕ y = , then
x−y
−2 −3
which of the following is an expression for a ⊕ b ?

b3 − a2

a−2 + b−3

1

b − a2
3

a2 b3

b3 − a2
a2 b3

a2 − b3

DO NOT DISTRIBUTE
| 63
GMAT IN CLASS MANUAL

3. For all integers n, the function f is defined by f(n) = kan where k


33 and a are constants. What is the value of f(3) ?

(1) f(0) = 5

(2) f(2) = 20

4. A sequence of numbers satisfies the equation An = 2(An – 1) + 1. If


A4 = 10, what is the value of A1 ?
0.375
1.375
1.75
4.5
9

DO NOT DISTRIBUTE
64 |
Lesson 3

Harder Manipulations
33
For some harder algebra problems, plugging in is the tool of choice. Sometimes,
it’s better to do the algebra, however.

2
 x+y
 
 
 x−y

1
1. If x ≠ y, x ≠ 0, and y ≠ 0 and if x is replaced by and y is
1 x
replaced by everywhere in the expression above, then the
y
resulting expression is equivalent to
2
 x+y
 
 
 x−y
2
 x−y
 
 
 x+y
x2 + y2

y2 − x2

x2 − y2

y2 + x2
2
 x+y
−  

 x−y

DO NOT DISTRIBUTE
| 65
GMAT IN CLASS MANUAL

2
33 2. If n > 1, which of the following is equal to ?
Is plugging in a good n+1− n −1
option for this question?

2n

n+1+ n −1

2
n+1+ n −1

n+1− n −1

Algebra and Data Sufficiency


For some data sufficiency questions, you may need to translate the statement into an
equation.

1. If b and c are positive and str ≠ 0, is


a
s
(1+ a b+a c =) a
s
+
b
t
+
c
r
?

s b
(1) a2 =
t

s c
(2) a2 =
r

DO NOT DISTRIBUTE
66 |
Lesson 3

2. What is the maximum weight of a package that can be


measured by a certain shipping scale? 33
(1) The scale can accurately determine the weight of a 200
pound package.

(2) If 100 pounds were added to a package that weighs half What do you need to do
to evaluate the second
the scale’s measurement capacity, the weight of the package statement?
1
would increase by .
4

3. For a banquet, a caterer charges $25 per person for the first
10 diners and x dollars for each additional diner. How much
does the caterer charge for each additional diner?

(1) If 15 people attend the banquet, the average Which statement is easier?
(arithmetic mean) cost per diner is $23. How can that help?

(2) The average cost per diner when 30 people attend the banquet
is $1 less than when 20 diners attend the banquet.

DO NOT DISTRIBUTE
| 67
GMAT IN CLASS MANUAL

Integrated Reasoning I
33
Integrated Reasoning questions mostly test familiar GMAT math and verbal
concepts. Mostly, you just need to get used to the different question formats.

Table Analysis
For some questions, you may need to sort the data in the table.

The table displays information about selected breakfast cereals.

Cereal Calories per Total Fat Sodium Riboflavin


Serving (%DV) (%DV) (mg)
Fiber Max 197 4 17 1.42
Frosted Chips 367 0 19 1.50
Golden Grains 361 6 27 5.37
Malted Crunchies 401 8 31 1.42
Multi-Grain Puffs 338 3 27 0.77
Raisin Flakes 354 2 16 0.80

For each of the following statements, select Yes if the statement can be
shown to be true based on the information in the table. Otherwise, select
No.

First decide if you need to Yes No


sort, then what you need to
sort by. The cereal with the greatest number of calories
per serving also had the greatest percentage of
the recommended daily value of Sodium.

The cereal with the least amount of Riboflavin per


serving also had the third lowest percentage of
the recommended daily value of Total Fat.

If Malted Crunchies provides 83% of the


recommended daily value of Riboflavin,
Golden Grains provides less than 300% of the
recommended daily value for Riboflavin.

DO NOT DISTRIBUTE
68 |
Lesson 3

Sort By: Calories per Serving


33
Cereal Calories per Total Fat Sodium Riboflavin
Serving (%DV) (%DV) (mg)
Fiber Max 197 4 17 1.42
Multi-Grain Puffs 338 3 27 0.77
Raisin Flakes 354 2 16 0.80
Golden Grains 361 6 27 5.37
Frosted Chips 367 0 19 1.50
Malted Crunchies 401 8 31 1.42 For Table Analysis questions,
you can sort by any column.

Sort By: Total Fat (%DV)

Cereal Calories per Total Fat Sodium Riboflavin


Serving (%DV) (%DV) (mg)
Frosted Chips 367 0 19 1.50
Raisin Flakes 354 2 16 0.80
Multi-Grain Puffs 338 3 27 0.77
Fiber Max 197 4 17 1.42
Golden Grains 361 6 27 5.37
Malted Crunchies 401 8 31 1.42

Sort By: Sodium (%DV)

Cereal Calories per Total Fat Sodium Riboflavin


Serving (%DV) (%DV) (mg)
Raisin Flakes 354 2 16 0.80
Fiber Max 197 4 17 1.42
Frosted Chips 367 0 19 1.50
Golden Grains 361 6 27 5.37
Multi-Grain Puffs 338 3 27 0.77
Malted Crunchies 401 8 31 1.42

Sort By: Riboflavin (mg)

Cereal Calories per Total Fat Sodium Riboflavin


Serving (%DV) (%DV) (mg)
Multi-Grain Puffs 338 3 27 0.77
Raisin Flakes 354 2 16 0.80
Malted Crunchies 401 8 31 1.42
Fiber Max 197 4 17 1.42
Frosted Chips 367 0 19 1.50
Golden Grains 361 6 27 5.37

DO NOT DISTRIBUTE
| 69
GMAT IN CLASS MANUAL

33 Table Analysis Basic Approach

1. Review any labels or titles that apply to the entire


table.

2. Decide whether you need to read any text blurbs


that go with the table.

3. For each question, decide whether you need to


sort the table.

4. If you decide to sort, decide which column to


sort by.

5. For calculation questions, decide whether you


need to use the calculator.

DO NOT DISTRIBUTE
70 |
Lesson 3

Graphics Interpretation
33
For graphics interpretation questions, your first step is to take the time to under-
stand the chart or graph.

100
Scores

50
A B C D E F G
Companies
The graph represents the range of the results of an entrance exam
for the potential employees of companies A, B, C, D, E, F, and G.
The highest possible score on the exam is 100 and 50 represents
the lowest score achieved on the exam by any of the potential
employees. The circles represent the average (arithmetic mean)
score accepted for employment. The squares represent the
minimum accepted score.

From each drop-down menu, select the option that creates the most
accurate statement based on the information provided.
Remember to open the drop
down boxes so you can use
1. The company with the least difference between its average POE!
and minimum accepted scores is __________.

A
C
D
G

2. The most commonly accepted minimum score is closest to


_________.

60
65
80

DO NOT DISTRIBUTE
| 71
GMAT IN CLASS MANUAL

33 Graphics Interpretation Basic Approach

1. Review the axes for the charts. Pay attention to


units.

2. Decide whether you need to read any text blurbs


that go with the chart.

3. Remember to open the drop down boxes before


you start working on the question. Review the
answers.

4. Write down a letter for each answer choice so you


can use POE.

5. For calculation questions, decide whether you


need to use the calculator.

Multi–Source Reasoning
Multi-Source Reasoning questions can include either two or three tabs with
information presented as graphs, tables, or text.

Retail Manager Sales Performance

In this store, the Music and DVD department occupies 2,500


square feet of floor space, which is 50% more floor space than
the Televisions department.

While the gross margin for the Music and DVD department may
be higher than that of the Televisions department, we also need
to look at sales per square foot.

On slow days, the sales per square foot for the Music and DVD
department are often less than $1.00 per square foot. The sales
for the Televisions department, however, are almost never less
than $5.00 per square foot.

DO NOT DISTRIBUTE
72 |
Lesson 3

Retail Manager Sales Performance


33
The chart shows some key statistics for one day’s performance
at an electronics store broken down by the store’s 6
departments.

Gross margin is the difference between the store’s sales and


the cost of the goods sold. Gross margin does not include
costs incurred to sell the goods such as payroll, transportation,
or warehousing costs.

Payroll shows the amounts paid to the employees who worked


that day but does not include additional costs such as taxes or
benefits paid by the store. The average hourly rate is shown for
each department for the employees who worked on this day.

Department Sales Gross Payroll Average


Margin (%) Hourly Rate
Camcorders $4,623 11.4% $300.72 $12.53
Computers $8,320 8.5% $439.36 $13.73
Televisions $14,256 14.1% $625.24 $14.21
Major
Appliances $9,280 12.2% $168.48 $10.53
Audio $7,879 18.5% $285.60 $9.52
Music & DVD $3,885 20.3% $252.88 $8.72

Consider each of the following statements. Does the information


supplied support the inference as stated?

Statement style questions


Yes No are common for Multi-
Source Reasoning.
Television sales for the day shown were more
than three times camcorder sales.

The store devotes less than 2,000 square feet to


selling televisions.

The Music and DVD’s sales per square foot for


the day shown were less than $1.00 per
square foot.

DO NOT DISTRIBUTE
| 73
GMAT IN CLASS MANUAL

1. If no employee can work more than 8 hours per day, what is the
33 minimum number of employees who worked in the Televisions
department on the day shown?
Standard multiple choice
questions can also be part 3
of Multi-Source Reasoning.
4
5
6
7

Multi-Source Reasoning Basic Approach

1. Review the information on each tab.

2. For each question, decide which tab or tabs the


information you need is on.

3. For statement style questions, remember that you


may not need to finish the calculation to answer
the question.

4. For questions that involve calculations, decide


whether you need to use the calculator.

DO NOT DISTRIBUTE
74 |
Lesson 3

Two-Part Analysis
33
For two-part questions, you pick two answers based on a similar situation.
For some Two-Part
Analysis questions, you
determine your answer for
1. Jason and Andrew each have $8,000 in a bank account. For each column separately.
1
each of three consecutive years, Jason withdraws of the
2
3
money in his account and Andrew withdraws of the money
4
in his account.

In the table below, select an answer that represents the fraction


of the money remaining in Jason’s account and select an
answer that represents the fraction of the money that remains in
Andrew’s account at the end of the three years. Make only two
selections, one in each column.

Jason’s Andrew’s Fraction of money


Account Account remaining
1
256
1
64
1
8
3
16

1
2
3
4

DO NOT DISTRIBUTE
| 75
GMAT IN CLASS MANUAL

2. At a certain gas station, the price of regular grade gasoline is


33 $3.50 per gallon and the price of premium grade gasoline is
20% greater than the price of regular grade gasoline. Krissi’s
car averages 22 miles per gallon on either grade of gasoline and
Krissi purchases gasoline to drive 110 miles.

In the table below, select an answer that represents how much


Krissi pays if she purchases regular grade gasoline. Then, select
an answer that represents how much she pays if she purchases
premium grade gasoline. Make only two selections, one for each
column.

Regular grade Premium Cost to drive 110


gasoline grade gasoline miles
$14.00
$17.50
$21.00
$28.00
$31.50
$77.00

DO NOT DISTRIBUTE
76 |
Lesson 3

3. Emily and Maggie collect rare books. Emily currently has 200
books in her collection and Maggie has 320. Both women 33
make monthly additions to their collections, each at her own For some Two-Part Analysis
questions, the answers are
constant monthly rate. In three months, Emily will have the
linked in some way.
same number of books in her collection as Maggie has in hers.

In the table below, select a rate of increase, in books per month,


for Emily’s book collection and a rate of increase, in books per
month, for Maggie’s collection that together are consistent with
the informaton. Make only two selections, one for each column.

Emily’s Maggie’s Monthly rate of


collection collection increase
20
50
70
80
100
120

DO NOT DISTRIBUTE
| 77
GMAT IN CLASS MANUAL

Homework Review
33 Use this chart to note any questions you have from the reading or examples in the homework.

Page # Question # What question do you have?

DO NOT DISTRIBUTE
78 |
Lesson 3

Practice
Roots and Exponents
a4 − 3 a3 − a + 3 1 2 4 8
1. If a ≠ 3 and = −126 , then what is 6. + + + =
the value of a ? a − 3
12 13 14
2 2 2 215
1

–5 210
5 1

3 212
126
15

126 215
2
25
210
2. If a and b are distinct integers greater than –1, 23

then what is the value of ab ? 216

(1) a–1 is undefined  1 1 1


(2) b = 2 48  + + 
4 2 2
2 3 2 
7. =
3. Is integer n greater than 3 ? 32
n
 1  16
(1)   > 0.0001
 100  27
61
n −1
 1  27
(2)   > 0.001
 100  61

4. If the speed of sound in air is 3.316 × 102 3
meters per second and the speed of sound
129
in water is 1,500 meters per second, then
approximately how many times faster does
sound travel in water than in air? 183
2
50(5−2 − 2−2 )
3 8. =
52
5
3
10
2
20
3
c −
 a 2
5. If a, b, and c are positive integers, is   > 1 ?
 b 21

(1) b – a = 9 50
(2) c > 1 21

50
21

25

DO NOT DISTRIBUTE
| 79
GMAT IN CLASS MANUAL

9. If a is not equal to zero, is a–3 a number greater Quadratics and More Algebra
than 1 ?
1. What is the sum of positive integers x and y ?
(1) 0 < a ≤ 2
(2) ab = a
(1) x2 + 2xy + y2 = 16
(2) x2 – y2 = 8
10. Dr. McCoy designed a space shuttle that can
theoretically travel at a maximum velocity of
8 times the speed of light. If the speed of light 2. Which of the following are roots of the
is 300 million meters per second, then which x ( x + 5)( x 2 − 4)
equation =0 ?
of the following is the theoretical maximum x +12
speed, in meters per second, of Dr. McCoy’s
shuttle? –2, 0, 5, –12
0, –5, 2, 12
2.4 × 103
–2, 0, 2, 5, –12
2.4 × 108 –5, –2, 0, 2
2.4 × 109 0, 4, 5
3.0 × 106
3.0 × 109 3. If a, b, and x are integers greater than zero,

11. Which of the following does NOT equal  then which of the following must be greater
(0.009) 3
a
than ?
(0.0003)  ? 3
a+ b

2.7 × 104 a+ x

a+ b+ x
23 × 33 × 53
a− x

0.00027 × 108 a+ b+ x

0.033 × 108 2a

2 a + 2b + x
1 1
×  a 
2
3−3 10−3  
 
 a+ b 
12. If x is greater than 0 but less than 10 and k = x9,
what is the value of integer k ? a −1

a + b −1
(1) x2 has a units digit of 1.

1 3 a−1 − 3−1 a
(2) x –2
< 4. =
50 3+ a
3− a

3a
3+ a

3a
3 a−2 +1

3a

3−a
a

1+ a

DO NOT DISTRIBUTE
80 |
Lesson 3

Answers and Explanations


Roots and Exponents
6. A The first thing you should recognize here is that
1. A If you are asked a specific question to which you
are given specific numerical answer choices, you
the intention cannot be to get you to just do the
should always Plug In The Answers. Though we
typically start with (C), here we want to start
math; to find a common denominator here would
with the choice that will be easiest to calculate,
which is (B). If we plug in (B), we get the answer
require ugly calculations. So, look for a simpler
124 = –126. Clearly this answer is too large, so
we need to try something smaller. Try (A) since
way to solve the question. The presence of the
it’s the only smaller choice. It works.
exponents and all the multiples of two should
2. A Statement (1) tells us that a=0, since only
division by 0 is undefined. Since b is distinct from
give you a hint that expressing things as powers of
a, it cannot be 0, and this statement is sufficient
to determine that ab is 0. So, AD. Statement (2)
two might be a good way to get a handle on the
tells us nothing about a, and so is insufficient;
eliminate (D).
problem. If we express the numerators as powers
3. D Plug In possible values for n. In both statements, 1 2 2 2 23
of two, then we would get   + + + .
you cannot plug in a value for n that is greater 212 213 214 215
than 3 without contradicting the statement, so
each statement is sufficient, and the answer is (D). Next, reduce each of the fractions that can be

4. C Convert the speed that’s in scientific notation reduced, and you have 112 + 112 + 112 + 112 .
2 2 2 2
into a “regular” number to avoid confusion.
3.316 × 102 = 331.6. 1,500 is about 5 times as Now adding our fractions is really easy, and
big, so the answer is (C).
we get 4 . Unfortunately, they aren’t done
a 212
5. A Statement (1) tells us that is a fraction less
b making us work. Since our answer doesn’t show
than one, and any fraction less than one, when up in the answer choices, we have to reduce again:

raised to a positive power, will remain a fraction 4 22 1


12
= 12
= 10 .
2 2 2
less than one. Statement (1) is sufficient. So, AD.
7. B This question calls for straight math, covering
Statement (2) does not tell us anything about
fractions and exponents. The easiest way to start
a a a
c
; if is an integer greater than one, then   is to re-express 48 as powers of 2 and 3 and then
b b b 
a
will also be greater than one, but if is a fraction distribute. Since 48 = 3 × 16, we can write 48
c
b
a 
less than one, then   will not be greater than as 3 × 24. Then we can multiply that by each of
b 
one. Statement (2) is insufficient; eliminate (D). the fractions in the parentheses and then reduce:

DO NOT DISTRIBUTE
| 81
GMAT IN CLASS MANUAL

3 × 24 3 × 24 24 3 × 24 9. E This question begs for a little translation and


= 3, = , and = 3 × 2 2 = 12.
24 32 3 22 simplification; it is another way to say 13 . For
1 a
Now add these all together: 3
to be greater than 1, a must be a positive
a
fraction less than 1. Statement (1) does not resolve
24 16 61
3 + + 12 = 15 + = . N o w , y o u ’ r e whether a is a fraction or not. So, BCE. Statement
3 3 3
(2) only tells you that b is 1; it tells us nothing
dividing the whole thing by 32, which is the same as
about a. Eliminate (B). When we look at the
1 61 1 61
multiplying by 2 or 1 . × = . statements together, we know nothing more about
3 9 3 9 27
a than we knew in Statement (1), so together they
8. C This question tests basic math in a somewhat are still not sufficient.

complex manner; it combines exponent, fraction, 10. C Given the fact that the answers here are in
scientific notation, we ought to do our
and distribution rules. First, we should probably calculations in scientific notation as well. 300
million is 3.0 × 108. Multiply that by 8 and you
re-express the numbers with negative exponents get 24 × 108 or 2.4 × 109.

as fractions in order to multiply them by 50.


11. D The answers here indicate that you need to
Remember, a negative exponent is really just express the fraction given as scientific notation or

1 over a positive exponent (e.g. 5–2 is the same you can try to calculate the value of the fraction,
which would be much more difficult. First, let’s
as 1 ). You should now have the inside of the
52 work with the numbers inside the parentheses:
parentheses as  1 − 1  . Now distribute the 50
 25 4 0.009 can be expressed as 9 × 10–3 and 0.0003 can
and reduce, and you get 2 − 25 . The next step be expressed as 3 × 10–4. Next, raise the numbers
2
is to deal with the denominator. Dividing by 52 inside the parentheses to the exponents out-
93 ×10−9
1 side the parentheses; we now have .
is the same as multiplying by 2 . We can then 33 ×10−12
5 You need to rewrite 93 in terms of 3: 93 = 36.
distribute, which gives us 25 25 . The last

2 2 × 25 Now divide, remembering that you should
step is to reduce and then subtract the fractions. subtract exponents when you divide. This yields
2 1 4 − 25 21 33 × 103, or 27,000. Now find the answer that is not
− = = − . Don’t forget to Bowtie to
25 2 25 × 2 50
33 × 103, or 27,000.
make the subtraction easier.

DO NOT DISTRIBUTE
82 |
Lesson 3

12. C For Statement (1), simply square the numbers 2. D The roots of an equation are those values that make
between 0 and 10; the results show that x could the equation equal 0. So all we have to do is find
be either 1 or 9, thus there are at least 2 possible what values will make the equation equal 0. If you
values for k. So, the possible answers are B, C, or don’t see anything that will make the equation
E. The only values of x that work with Statement equal 0, then just plug in the answers. If you do,
(2) are 8 and 9, so Statement (2) is not sufficient, then look for your numbers in the answer choices,
but the two statements together make it clear that and eliminate anything without your numbers.
the value of x is 9. The roots are –5, –2, 0, and 2.

3. A Plug In. Let a = 2, b = 3, and x = 4. Then the


Quadratics and More Algebra
question can be read as “which of the following
1 D Start with Statement (1). If we factor the equa-
is greater than 2 .” The next step is to plug those
tion given, it yields (x + y)2 = 16, so x + y = 4 5
(note that we’re told that x and y are positive), so numbers into the answers and see which one
Statement (1) is sufficient. So, AD. Statement (2)
gives a number bigger than 2 . Remember to
can also be factored, and yields (x + y)(x – y) = 8. 5
This tells us that x + y and x – y must be factors check all the choices, even though the first one
of 8. Eight only has four factors, 1, 2, 4, 8. If we
consider each possible factor in turn, and if x and works.
y are positive integers and must equal one of these
4. A Plug in. Let a = 2, then solve the given equation.
factors, there is no way that x + y can equal 1. If
1 3 2 5
3a–1 = 3   = . 3–1a = × 2. − = , which,
x + y must equal 2, then x and y must both be 1, 1 3
but in that instance x – y would not equal 4, thus 2 2 3 2 3 6
x + y cannot be 1. If we continue to try each factor, when divided by 5 (the sum of 3 + a) is 1 . Now
6
the only factor of 8 that x + y could be is 4, thus just plug 2 into each answer choice, and find which
this statement is also sufficient.
one works.

DO NOT DISTRIBUTE
| 83
DO NOT DISTRIBUTE
Lesson 4

Lesson 4 44

Critical Reasoning 1
In the pre-class assignment, you learned how to analyze an argument by
breaking it into its component parts. Now we’ll see how to use that analysis to
work critical reasoning questions.

Basic Approach
Follow the same basic approach for all critical reasoning questions.

Step 1: Identify the question.


Begin by reading the question stem. Identify the question type before you read
the argument.

Step 2: Work the argument.


For most question types, you will start working the argument by finding the
conclusion and premises. Next, you’ll want to find the gap, assumption, or pattern.

Step 3: Predict what the answer should do.


Think about what the credited response needs to do.

Step 4: Use POE to find the answer.


It’s easier to find wrong answers than right answers. Use POE aggressively.

We’ll show you how to approach all the different types of arguments
questions that appear on the GMAT:

• Assumption • Inference

• Weaken • Resolve/Explain

• Strengthen • Evaluate the Argument

• Identify the Reasoning • Flaw

DO NOT DISTRIBUTE
| 85
GMAT IN CLASS MANUAL

Assumption Questions
Step 1: Read and identify the question.
44
Look for assumption, assume, or presupposition. Assumption questions commonly ask:

• The argument above assumes which of the following?


• Which of the following is an assumption on which the argument
depends?
• The claim made above rests on the presupposition that

Step 2: Work the argument.


Break the argument into its parts. Look for:

• Conclusion
• Premise
• Gap, Assumption, or Pattern

Step 3: Predict what the answer should do.


The correct answer provides a link between the conclusion and the premise.

Step 4: Use POE to find the answer.


Eliminate answers that:
• Are out of scope.
• Use extreme language.

DO NOT DISTRIBUTE
86 |
Lesson 4

Don’t forget to find the


1. The technological conservatism of tennis racket
conclusion and premise.
manufacturers is a reflection of the kinds of
demands they are trying to meet. The only tennis
44
players who are seriously interested in innovation
and willing to pay for it are professional players.
Therefore, innovation in tennis racket technology is
limited by what authorities will accept as standard
for purposes of competition in professional tennis.
Which of the following is an assumption made in
drawing the conclusion above?
The market for cheap, traditional rackets cannot
expand unless the market for technologically
advanced rackets also expands.
Professional tennis rackets are likely to
improve more as a result of technological
innovations developed in small workshops
than as a result of technological
innovations developed in major
manufacturing concerns.
Professional tennis players do not generate
a strong demand for innovations that fall
outside what is officially recognized as
standard for the purposes of competition.
The technological conservatism of tennis
racket manufacturers results primarily from
their desire to manufacture a product that
can be sold without being altered to suit
different national markets.
The authorities who set standards for
professional tennis rackets do not keep
informed about innovative racket design.

DO NOT DISTRIBUTE
| 87
GMAT IN CLASS MANUAL

2. Computers that can be identified and traced are of no


use to thieves. So Maguda Inc. has instituted a plan
to reduce theft by etching the serial number of each
44
computer both on the frame of the monitor and on the
face of the hard drive where it will be impossible to
overlook.
Maguda Inc.’s plan assumes that
thieves do not steal computers that are of no
value to them
Maguda Inc.’s competitors also plan to etch
serial numbers on their computers
thieves are more inclined to steal computers
than any other type of office equipment
etching the serial number on the computer has
no effect on the performance of the computer
imposing stiffer jail sentences for thieves would
not decrease the number of computers stolen

DO NOT DISTRIBUTE
88 |
Lesson 4

The Negation Test


On some assumption questions, you may have a lot of difficulty spotting the
44
necessary assumption due to the way the answers are worded. In those cases, try
using the negation test.

3. A recent report determined that although only 10


percent of those in police custody in the City of
Egan volunteered to undergo a polygraph test, 40
percent of those released from custody without
being charged had volunteered for the polygraph
test. Clearly, suspects in custody who had
volunteered for the polygraph test were much less
likely to have committed their alleged crime than
those who did not volunteer.

Which of the following is an assumption on which


the argument above depends?
Some of the people in police custody who had
volunteered to undergo the polygraph test
To use the Negation Test:
were not released without being charged.
1. Negate the answer.
Suspects who volunteer for polygraph tests are not 2. Ask: Do I still believe the
more likely to know that polygraph evidence can conclusion?
only be used as a defense against charges than
those who do not volunteer.
The number of suspects released from
police custody without being charged was
greater than the number of suspects who
volunteered to undergo a polygraph test.
Some of the suspects who were not
released from police custody were
suspected of having committed more than
one crime during the time period covered
by the report.
The City of Egan has a higher crime rate than
other cities of its size.

DO NOT DISTRIBUTE
| 89
GMAT IN CLASS MANUAL

44 Weaken Questions
Weaken questions ask you to find an answer that attacks the argument’s
conclusion. To weaken an argument, you must widen the gap between the
conclusion and premise, thereby showing the assumption to be false.

Step 1: Read and identify the question.


Look for key words like weaken, undermine, or cast doubt. Weaken questions
typically ask:

• Which of the following, if true, most seriously weakens the conclusion


drawn in the passage?
• Which of the following, if true, would cast the most serious doubt on the
conclusion above?
• Which of the following indicates a flaw in the reasoning above?

Step 2: Work the argument.


Break the argument into its parts. Look for:

• Conclusion
• Premise
• Gap, Assumption, or Pattern

Step 3: Predict what the answer should do.


The correct answer attacks one of the assumptions of the argument.

Step 4: Use POE to find the answer.


Eliminate answers that:
• Are out of scope.
• Use extreme language.
• Strengthen the argument.

DO NOT DISTRIBUTE
90 |
Lesson 4

1. Certain gemstones are formed by magma, a


material found deep inside the Earth, that has 44
pushed toward the surface and solidified; others
are created when superheated water and gases
cool. When chemical action, heat, or pressure
changes the existing structure of rocks, they
may recrystallize to form gemstones such as
emeralds. A large chemical company plans to
create a new source of profits by applying certain
of the chemicals it manufactures to specifically
chosen rock samples, which would be a much less
expensive way to produce emeralds than mining
them.

Which one of the following, if true, poses the most serious


problem for the success of the company’s plan?
Artificial emeralds will not be considered as
valuable as natural emeralds.
Natural emeralds are produced by heat and
pressure much more frequently than by
chemical action.
The types of material from which emeralds are
formed are not found on the Earth’s surface.
Laboratory conditions are difficult to
replicate in a large-scale factory.
The gases that will mix with superheated
water are so toxic that workers could
suffer from a number of serious medical
conditions if the handling of such gases is
not strictly controlled.

DO NOT DISTRIBUTE
| 91
GMAT IN CLASS MANUAL

Strengthen Questions
To strengthen an argument, find the gap and then build a bridge across it,
44
making the argument stronger. Strengthen questions go a step beyond assumption
questions. For strengthen questions, add a premise that provides extra support for
the argument.

Step 1: Identify the question.


Look for key words like support, strengthen, or justify. Strengthen questions
typically ask:

• Which of the following, if true, would most support the claims above?
• Which of the following statements, if true, would most strengthen the
conclusion?

Step 2: Work the argument.


Break the argument into its parts. Look for:

• Conclusion
• Premise
• Gap, Assumption, or Pattern

Step 3: Predict what the answer should do.


The correct answer bolsters one of the assumptions of the argument.

Step 4: Use POE to find the answer.


Eliminate answers that:
• Are out of scope.
• Use extreme language.
• Weaken the argument.

DO NOT DISTRIBUTE
92 |
Lesson 4

1. The mayor of City X wishes to increase voter


turnout for the next city council election. A recent
poll of registered voters in neighboring City Y
44
showed a high level of dissatisfaction with that
city’s voting facilities. Based on this result, City X’s
mayor proposed a dramatic update of his city’s
voting facilities to increase turnout at the next
election.

Which of the following, if true, most supports the


mayor’s plan?
A majority of respondents in City Y’s poll had voted
in favor of a tax cut in the previous election.
City Y’s voter turnout rates increased as the result
of the publicity the recent poll generated.
A City X poll of registered voters revealed that
their behavior is not strongly influenced by their
perception of voting facilities.
Even the best polling methods have difficulty
capturing voter opinions accurately.
The City Y poll showed that voters who are satisfied
with voting facilities are more likely to vote.

DO NOT DISTRIBUTE
| 93
GMAT IN CLASS MANUAL

MATH 4
4 Ratios
By itself, a ratio tells you only the relative amounts of the quantities. It does not tell
you the actual amounts.
Which two tecniques
can you use to solve this 1. The present ratio of almonds to cashews in a certain can of
problem? nuts is 2 to 3. If 45 almonds and 30 cashews were to be added
to the can, the ratio of almonds to cashews would be 7 to 8.
What is the present number of nuts in the can?
30
60
90
120
150

DO NOT DISTRIBUTE
94 |

GMAT InClass Manual Lesson 4.indd 94 7/2/18 6:49 PM


Lesson 4

2. In a certain mosaic, the ratio of the number of blue tiles to the


number of red tiles is 9 to 2, and the ratio of the number of
yellow tiles to the number of blue tiles is 3 to 8. If the ratio of
4
the number of green tiles to the number of red tiles is 9 to 10,
what is the ratio of the number of yellow tiles to the number of
green tiles?

2

9
1

4
8

15
9

2
15

8

3. A certain jar contained only pennies and nickels. After 3 pennies


What do you need to make
were removed, how many nickels remained in the jar? a ratio box work?

(1) The original ratio of pennies to nickels in the jar was 7:4.

(2) After the 3 pennies were removed, the ratio of pennies to


nickels in the jar was 3:2.

DO NOT DISTRIBUTE
| 95

GMAT InClass Manual Lesson 4.indd 95 7/2/18 6:49 PM


GMAT IN CLASS MANUAL

Averages
The average is also called the arithmetic mean, or simply the mean. Use the Average
44
Pie to organize your information.

Total

# of Things Average

1. On a geography quiz, Emily and Katherine scored an average


(arithmetic mean) of 80 points. If Katherine and Julia scored an
average of 85 points, how many more points did Julia earn
than Emily?
15
10
7.5
5
It cannot be determined from the information given.

2. If two sets of numbers, P and Q, have the same number of


elements, is the average (arithmetic mean) of set Q less than
the average (arithmetic mean) of set P ?

(1) Set Q consists of consecutive even integers and set P of


consecutive odd integers.

(2) The median of set Q is greater than the average of


set P.

DO NOT DISTRIBUTE
96 |
Lesson 4

3. For any positive integer n, the sum of the first n positive


n( n + 1)
integers equals . What is the difference between the What does this problem
2 have to do with averages? 44
sum of all the multiples of 4 between 1 and 44 and the sum

of all the multiples of 3 between 1 and 33 ?


55
165
220
355
385

4. The average (arithmetic mean) of u, v, w, and 64 is 8 more


than the average (arithmetic mean) of u, v, and w. The
average of u, v, w, and 64 is what percent greater than the
average of u, v, and w ?
12.5%
25%
37.5%
50%
96%

DO NOT DISTRIBUTE
| 97
GMAT IN CLASS MANUAL

Rates
Rate problems ask you questions about work or distance. Use the Rate Pie to
44
organize your information.

Work/Distance

Time Rate

When Rob and David are


both traveling, how many 1. Rob and David live 200 miles apart. Deciding to have a picnic,
total miles do they travel in they both start driving at 9:00 a.m., traveling in a straight line
an hour? towards each other. Rob drives at a steady speed of 50 miles
per hour, and David drives at a steady speed of 30 miles per
hour. What is the time when they meet for their picnic?
11:30 a.m.
1:00 p.m.
1:30 p.m.
2:00 p.m.
3:40 p.m.

DO NOT DISTRIBUTE
98 |
Lesson 4

2. Joan can do an entire job in 12 hours. Ken can do an entire


job in 6 hours. If Joan and Ken work together for 3 hours,
how many hours will it take Ken to finish the rest of the job
44
by himself?
1
1
1
2
1
4
2
6

3. Working together without a break, Jermaine and Crystal can What do you know?
do a job in 12 hours. How long will it take Crystal to do the What do you need?
job alone?

(1) Working without a break, Jermaine takes 16 hours to do


the job alone.

(2) Working alone and without a break, Crystal takes 24 hours


to do half the job.

DO NOT DISTRIBUTE
| 99
GMAT IN CLASS MANUAL

Percent Change
You learned about percent change problems in the pre-class assignment. The
44
percent change formula is:

% Change = difference × 100


original

Increase or greater means that the original number is the ___________ number.
Decrease or less means that the original number is the ___________ number.

Use the percent change formula to solve the following questions. Make sure you
correctly identify the original number.

1. By what percent did Company A’s revenues increase from 1980


to 1985 ?

(1) In 1980, Company A’s revenues were $600,000; in 1985, its


revenues were $800,000.

(2) Each year from 1980 to 1985, Company A’s revenues


increased by $40,000.

What technique should 2. The original price of an article was reduced by 25 percent. During
you use when a question
asks for a percentage of an
a special sale the new price was decreased by 10 percent. By
unknown total? approximately what percent would the price now have to be
increased in order to restore the price of the article to its
original amount?
32.5%
35%
48%
65%
67.5%

DO NOT DISTRIBUTE
100 |
Lesson 4

Probability
As you learned in the pre-class assignment, probability questions test your
44
understanding of part-to-whole relationships.

number of outcomes you want


Probability =
number of total possible outcomes

What operation should


1. Alejandro flips a coin four times. What is the probability that
you perform to find the
he gets heads on the first 2 throws and tails on the last probability of a series of
2 throws? events?

1

16
1

8
1

4
1

2
3

4

DO NOT DISTRIBUTE
| 101
GMAT IN CLASS MANUAL

What’s different about this 2. Griffin has a bag of marbles that contains only 6 black marbles
problem? and 4 red marbles. If he removes three marbles at random
without replacing any of the marbles, what is the probability that
44
all three marbles selected are red?

2

5
1

6
3

25
8

125
1

30

DO NOT DISTRIBUTE
102 |
Lesson 4

3. Ann has a deck of 52 cards, made up of four suits, each


with cards numbered 1 through 13. If she selects a card at
random, what is the probability that she selects a 2, 7, or 9 ?
44
1

3825
1

64
3

52
3

13
3

4

Probability (A and B) = Probability (A) × Probability (B)


Probability (A or B) = Probability (A) + Probability (B) – Probability (A and B)

DO NOT DISTRIBUTE
| 103
GMAT IN CLASS MANUAL

Probability (event happens) + Probability (event does NOT happen) = 1


44


What is the significance of 4. Kevin flips a coin four times. What is the probability that he gets
the words at least one in heads on at least one of the four flips?
this problem?
1

16
1

4
3

4
13

16
15

16

5. A bowl of fruit contains only apples, oranges, and pears. If a piece


of fruit is randomly selected from the bowl, what is the probability
that the piece of fruit is an apple?

(1) Half of the fruit in the bowl is oranges.

(2) The probability of NOT selecting a pear is 70 percent.

DO NOT DISTRIBUTE
104 |
Lesson 4

Homework Review
Use this chart to note any questions you have from the reading or examples in the homework.

Page # Question # What que stion do you have ?


44

DO NOT DISTRIBUTE
| 105
GMAT IN CLASS MANUAL

Practice
Ratios, Averages and Rates
1. Frances can complete a job in 12 hours, and 5. If the average (arithmetic mean) of a, b, and c is
Joan can complete the same job in 8 hours. 2x, and the average of d and e is 3y, what is the
Frances starts the job at 9 a.m., and stops average of a, b, c, d, and e, in terms of x and y ?
working at 3 p.m. If Joan starts working at x–y
4 p.m. to complete the job, at what time is the
job finished?
x+y
6 pm
7 pm
6x + 6y
8 pm
10 pm 2 x + 3y

12 pm 5
1 6 x + 6y
2. In a mixture of alcohol and water there is
5 5
more alcohol than water. What is the ratio of
6. Working together, Wayne and his son can
alcohol to water in the mixture? shovel the entire driveway in three hours. If
Wayne can shovel three times as fast as his son
5:1 can, how many hours would it take for his son
4:1 to shovel the entire driveway on his own?
12:5 4
6:5 6
5:6 8
9
3. Susan and Iman are each typing a term paper.
12
Who finishes first?
7. If the range of the set of numbers {150, 90,
(1) Susan begins typing at 9 am and
125, 110, 170, 155, x, 100, 140} is 95, which of
finishes at 5 pm.
the following could be x ?
(2) Iman finishes typing 6 hours after she
80
starts.
85
4. In six successive games, a baseball team 95
scored 3 runs once, 6 runs twice, and 9 runs 125
three times. What was the average (arithmetic 185
mean) number of runs the team scored per
game over the six-game period? 8. Working at their individual rates, Marcus and
Latrell can build a certain brick house in 7.5 and
8 5 hours, respectively. When they work together,
7 they are paid $35 per hour. If they share their
6 pay in proportion to the amount of work each
5 does, then what is Marcus’ hourly pay for
4 building the house?
$3
$6
$7
$14
$21

DO NOT DISTRIBUTE
106 |
Lesson 4

9. Working together, Max and Mandy painted the Probability


living room in six hours. How long would it have
taken for Max to paint the room by himself? 1. Tammie has 10 cards numbered 1 through 10.
(1) Mandy paints twice as fast as Max does. If she deals two to Tarrell without replacing
any of them, what is the probability that Tarrell
(2) If Max had left when the job was half will get both a 2 and a 3 ?
1
finished, it would have taken Mandy 4
2 1
hours to finish the job by herself.
5
10. If two projectiles are launched at the same 1
moment from 1,320 miles apart and travel
directly towards each other at 480 miles per 45
hour and 510 miles per hour, respectively, how 1
many minutes will it take for them to meet?
50
40 1

44 90
80 14
88
45
90
2. A die with x sides has consecutive integers on
11. Both Robert and Alice leave from the same
location at 7:00 a.m. driving in the same its sides. If the probability of NOT getting a 4
direction, but in separate cars. Robert drives 30 36
miles per hour while Alice drives 40 miles per on either of two tosses is , how many sides
49
hour. After 6 hours, Alice’s car stops. At what
does the die have?
time will Robert’s car reach Alice’s car?
4
1 pm
5
3 pm
7
4 pm
8
8 pm
13
9 pm
3. In a drawer of shirts, 8 are blue, 6 are green,
12. Before leaving home for the town of Madison, and 4 are magenta. If Mason draws 2 shirts at
Pete checks a map which shows that Madison random, what is the probability that at least
is 5 inches from his current location, one of the shirts he draws will be blue?
Gardensquare. Pete arrives in Madison 2.5
hours later and drove at an average speed of 25
60 miles per hour. At what scale, in inches per
153
mile, is the map drawn?
28
1
153
3
5
1
17
30
1 4

10 9
12
2
17

30

DO NOT DISTRIBUTE
| 107
GMAT IN CLASS MANUAL

4. A certain consulting firm employs 8 men and 4 6. Three students, Mark, Peter, and Wanda, are all
women. In March, 3 employees are selected at
random to represent the company at a working on the same math problem. If their
convention. What is the probability that the 1 2
individual probabilities of success are , ,
representatives will NOT all be men? 4 5
3
and , respectively, then what is the
14 8
probability that at least one of the students will
55
3 get the problem correct?

8 3
41
80
55
9
2
32
3
23
54
32
55
77

80
39
5. A jar contains only nickels, dimes, and
quarters. If a coin is drawn from the jar at 40
random, what is the probability that it is either
a nickel or a quarter?
(1) The probability the coin is a nickel or a 7. James and Logan are taking batting practice.
If their individual probabilities of hitting a
1 homerun are x and y, respectively, then what
dime is .
2 is the probability that James will not hit a
1 homerun but Logan will?
(2) The probability the coin is a dime is .
5 x – xy

x – y2

y – xy

y

x
1
−y
x

DO NOT DISTRIBUTE
108 |
Lesson 4

8. Jean drew a gumball at random from a jar of 10. Scott, Jean, and Warren are all building

pink and blue gumballs. Since the gumball she wooden models for an architectural

selected was blue and she wanted a pink one, presentation at noon tomorrow. If their

she replaced it and drew another. The second individual probabilities of finishing on time
1 1
gumball also happened to be blue and she are x, , and , respectively, then what is the
3 7
replaced it as well. If the probability that she probability that Warren will finish on time but
9
drew two blue gumballs is , what is the Jean and Scott will not?
49
probability that the next one she draws will be 21x − 38

pink? 21

1 12 x

49 21
2−2x
4
21
7
3 2x −2
21
7
x
16
21
49
40

49

9. When a die that has one of six consecutive
integers on each of its sides is rolled twice,
what is the probability of getting the number 1
on both rolls?
(1) The probability of NOT getting an eight is 1.

25
(2) The probability of NOT getting a seven is
. 36

DO NOT DISTRIBUTE
| 109
GMAT IN CLASS MANUAL

Answers and Explanations


Ratios, Averages and Rates
8. D As with the vast majority of work questions, you
1. C Plug In for the number of tasks to be completed
before the job is done; let the job involve making want to start this by Plugging In for the total size
24 widgets. Thus, Frances makes 2 per hour, and of the job. If the house is 30 units (30 because it’s
Joan makes 3 per hour. Frances works for 6 hours,
so she makes 12 widgets. Joan needs 4 hours to divisible by both 7.5 and 5), then Marcus does 4
make the other 12 widgets; if she starts at 4 p.m., units per hour and Latrell does 6 units per hour.
she’ll finish at 8 p.m.
At these rates, they would be completing 10 units
2. D You have more alcohol than water, so get rid of per hour together and would finish the job in 3
(E). Plug In: If you have 5 gallons of water, you
have 1 more gallon of alcohol. That’s 6 gallons of hours. In 3 hours Marcus would have done 12
alcohol and 5 gallons of water. units ( 2 of the job) and Latrell would have done
5 3
3. E Statement (1) doesn’t tell you anything about 18 units ( of the job). Marcus should therefore
Iman, so the possible answers are B, C, or E. 2 5
get of the money.
Statement (2) doesn’t mention Susan, so 5
eliminate (B). Even (1) and (2) together don’t
tell who finishes first. You know that Iman takes 9. D Statement (1) is tricky, but it’s doable. Say that
less time than Susan does, but she might have the paint job involves painting 72 square feet.
started later. Together, Max and Mandy paint 12 square feet
4. B The team scored a total of 42 runs over the course per hour. If Mandy paints twice as fast, then she
of 6 games. So, the average is 7 runs per game. paints 8 square feet per hour and Max paints 4
square feet per hour. Thus, the job would take
5. E It’s a Plug In extravaganza! If you plug in a = 5,
Mandy 9 hours to do by herself. Remember
b = 6, c = 7, then x = 3; further, if you plug in
that you should then plug in a second number
d = 11 and e = 13, you’ll find that y = 4. The
42 to be sure that you get the same answer to the
average value of 5, 6, 7, 11, and 13 is (target
5 question. Statement (2) offers similar info. If
answer). 1
Mandy does half the job in 4 hours, then she
6. E Let the driveway have an area of 120 square feet. 2
can do the whole job in 9 hours.
Together, Wayne and his son shovel 40 square feet
per hour. You can use a Ratio Box to show that if
10. C The projectiles rocket toward each other at a
Wayne shovels three times as fast, Wayne shovels
30 square feet per hour and his son shovels 10 combined speed of 990 m.p.h. (At this point,
square feet per hour. Thus, it would take his son you know it will take more than an hour;
12 hours to shovel 120 square feet.
eliminate (A) and (B).) Now use the formula
7. E They tell us the range of the set {150, 90, 125, rt = d: If 990t = 1,320, t = 4 . Therefore, it takes
110, 170, 155, x, 100, 140} is 95. Since the 3
1 hour and 20 minutes.
present range without x is 80, x has to be either the
greatest or the least number in the set. If x is the least
number, it would be 170 – 95 = 75, but that’s not an
option. Therefore x has to be the greatest number.
90 + 95 = 185.

DO NOT DISTRIBUTE
110 |
Lesson 4

11. B Alice travels 10 miles per hour faster, so she is 3. E Remember that at least one is a clue, and
60 miles ahead after 6 hours (at 1 p.m.). Robert
travels 30 m.p.h., so he needs 2 hours to catch up. when you see that phrase, you need to find the
He’ll be there at 3 p.m. probability of getting everything except what you
12. B First, use the rate formula to calculate the want (in other words, the probability of getting
distance from Madison to Gardensquare, which
any other color except blue), and then subtract
is 150 miles. Now you know that 5 inches on the
map is equal to 150 miles in real life; thus, 1 inch that from 1. The formula for this would be
must represent 30 miles. Since the answers are in
inches per mile, we know that inches must be on 1 – (the probability of getting the other colors).
the top of any fraction.  10 9  5 12
1−  ×  =1− = .
 18 17  17 17
Probability 4. C The question here really translates to “what is the
1. B Remember probability is expressed as a fraction: probability that at least one woman will go to the
(number of possibilities that meet requirement convention?” The easiest way to determine this is
of question) divided by (the number of total
to take 1 minus the chances that all the delegates
possibilities). On the first deal, Tammie has 2  8 7 6 14 41
will be men. 1 −  × ×  =1− = .
cards that would meet the requirement and 10  12 11 10  55 55
total possibilities. On the second deal, there is only 5. B We’re looking for the chances of selecting either
a nickel or a quarter. Statement (1) gives the
1 that would meet the requirement and only 9 chances of getting a nickel or a dime, which
possibilities. Thus, we get two fractions: allows you to infer the chances of getting a
2 1 1 quarter. We still don’t know the exact chances
× = .
10 9 45 of getting a nickel, though, so it’s not sufficient.
Write down BCE. Statement (2) is sufficient
2. C Don’t do algebra here; it’s a nightmare. Plug In The because if you know that the chances of getting a
Answers, since you are given the possibilities for dime are 1 in 5, then that means the chances of
getting either a nickel or a quarter are 4 in 5.
the number of sides. Then, find the probability of
NOT getting a 4 and see if it matches what you 6. C The words at least one are key here: You need
are given. If you plug in (C), you get seven sides. to figure the odds of NONE of the students
The odds of NOT getting a number that is on one solving the problem correctly, and then subtract
 6   6  36
of those seven sides is     = . The answer
 7   7  49 that number from one. The chances of NONE
is (C).
of them answering the question correctly are
 3  3  5  9
 4   5   8  = 32 . Subtract that from 1, and you
   
get your answer: 23 .
32

DO NOT DISTRIBUTE
| 111
GMAT IN CLASS MANUAL

7. C Plug In. If we plug some fraction in for x and y, 10. C To find the probability here, we just need to
then we can work the problem with relative ease.
multiply the probability that Warren finishes
1 1
For example, if x = and y = , then James’
4 3 by the probability that Scott doesn’t finish by
3
probability of not hitting a home run is and
4
Logan’s probability of hitting a home run is 1 .   the probability that Jean doesn’t finish. Since
3
The next step would be to multiply these two they have given us the probabilities for Warren
fractions: 3 × 1 = 1 . Now plug your values for x already, we don’t need to do anything but use
4 3 4
and y into the choices to see which one gives you
that number. For Jean, if her probability of
your target answer. Remember to check all five
1
finishing is , then the probability of her not
answers. 3
2
8. B It’s all about Plugging In The Answers here. Just be finishing would be . Plug in a value for Scott:
3
1
clear on what the answers represent: the chances Let x = . So his chances of not finishing
4
3 3
of getting pink. Consider (C). If you have a would be . Now just multiply the whole mess
7 4
chance of getting pink, that means you have a 4 1
together: × 2 × 3 = 1 . Now just plug in for
1
7 7 3 4 14 4
chance of getting blue, which would give you a
x, and find the answer that matches.
16 chance of getting two blue, which is too big.
49
We need a smaller chance of getting blue, which

actually means we need a larger chance of getting

pink. The correct answer is (B).

9. B Taking a minute to translate and understand


the pieces of the puzzle needed to answer the
question reveals that to find out this probability,
all we need to know is whether the number 1 is
on the die. Statement (1) tells us that 8 is not on
the die. The six integers could be either 0–5 or
2–7, so it’s insufficient. The possible answers are
B, C, or E. Statement (2) tells us that seven is on
the die (since the probability is not 100% there
is some chance of getting a 7); therefore since all
six integers are consecutive, one cannot be on the
die, thus Statement (2) is sufficient.

DO NOT DISTRIBUTE
112 |
Lesson 5

Lesson 5
55

Reading Comprehension 1
The Basic Approach
Step 1: Work the passage.
Read for the passage’s main idea, and get a sense of its structure.

Step 2: Understand the Question Task.


Look for the question’s subject and the words that indicate its task.

Step 3: Find the Information that Addresses the Task.


Once you understand the question task, use the question’s subject to locate infor-
mation in the passage that addresses the question task.

Step 4: Use POE to find the answer.


Be skeptical of each answer choice. Try to find reasons to eliminate the answer.

DO NOT DISTRIBUTE
| 113
GMAT IN CLASS MANUAL

Finding the Main Idea: Key Sentences


There are a variety of ways that you can find the main idea of the passage. Which
tool you use often depends on the structure of the passage.
55
What is the most important Retail stores and diversified manufacturing companies
sentence of this paragraph? have operated under a set of traditional assumptions
that warrant challenge. The most basic assumption that
company managers make is that their companies should
(5) provide a high level of service to all their customers.
However, acting on this assumption can lead to a loss
of market share, less value for some customers, and
maintenance of an unwieldy structure for distributing
products which ensures high fixed costs.
What are the key sentences (10) According to Joseph Fuller, James O’Conor, and Richard
for this paragraph? Rawlinson, one area that needs particular scrutiny is
the way in which companies handle logistics, including
transportation costs, handling costs, management of
inventory, storage costs, and order processing. Different
(15) customers often have entirely different needs: designing
a logistical structure that provides every customer with
the same level of service is wasteful and inefficient. Fully
one-third of a company’s product may be stuck in the
“pipeline” between manufacturers and customers, where
(20) it only drains away money through transportation and
storage costs; if a particular customer does not need
certain products to be available immediately, a company
does not need to spend money to ensure that all its
merchandise is on hand. Another problem is traditional
(25) averaging, in which products that cost the manufacturer
relatively little to produce are given prices similar to those
of products that are expensive to produce. While this
means that the retailer is able to move more low-volume
products out of inventory, high volume products tend to
(30) be overpriced, and more specialized products are not
delivered speedily enough and may be underpriced.
What about this paragraph? Fuller et al. describe a soft drink company’s decision
to stop in-store promotions and special sales in favor
of standard pricing, because the inconsistent demand
(35) caused by the swings in price necessitated variability in
the manufacturing and distribution systems. Many retail
managers tend to overlook logistics out of a concern for
gross margin; that is, they are swayed by the gross profit
made by the sale of a specific item, instead of looking at
(40) the net profit that remains after logistics costs have been
subtracted. Low-volume, high-margin products may not
ultimately be as profitable as high-volume, low-margin
products that are easy to move around such as T-shirts or
calculators.

DO NOT DISTRIBUTE
114 |
Lesson 5

Passage Notes
Use this space to take notes on the passage.

55

Issue of the Passage:

Main Idea of the Passage:

How is the passage structured?

1. The primary purpose of the passage is to What do you notice about


the language of the incorrect
discuss the impact of logistics on profitability answer choices?
challenge the belief that retail and manufacturing businesses
must provide similar levels of service to all customers
question the adoption of standard pricing by retailers seeking
to sell high-margin products
fault retail managers who pay undue attention to gross profit
rather than focusing on net profit
argue that retail businesses should abandon the sale of
low-volume, high-margin products in favor of high-volume,
low-margin products

Issue: The topic of the passage.


Main Idea: What the author wants you to believe about the
issue the passage discusses.

DO NOT DISTRIBUTE
| 115
GMAT IN CLASS MANUAL

Finding the Main Idea: Follow the Author


For some passages, it’s very important to separate the issue under discussion from
the author’s opinion of that issue.
55
What’s the function of this The accession to power by the reform-minded Mikhail
paragraph? Gorbachev in 1985, the fall of the Berlin Wall, and the
eventual dissolution of the Soviet Union all created a
feeling of heady optimism and a sense of victory in the
(5) West in general and in the United States in particular.
Many in the Western World believed their side had
prevailed in a contest of ideologies. Two academics who
sought to give intellectual heft to this widespread attitude
were Francis Fukuyama and John Mueller.
What words indicate a shift in (10) Fukuyama argued in his essay, The End of History, that
the author’s tone?
liberal democracy and capitalism were the ultimate forms
of political and economic organization and that all history
led to the emergence of these structural arrangements.
Fukuyama essentially adopted Marx’s stage theory of
(15) history while supplanting Marx’s eventual outcome,
communism, with liberal democracy and capitalism. While
Fukuyama’s work can perhaps be excused as the musings
of an ideologue, Mueller’s work is more problematic.
Does the author agree or
In his book, Retreat from Doomsday: The Obsolescence
disagree with Mueller? (20) of Major War, Mueller makes the mistake of reasoning
How do you know? from a predetermined conclusion rather than pursuing
How does that affect the
a more authentic scholarship of weighing evidence to
main idea? arrive at a conclusion. Mueller’s central claim that major
war is obsolete rests on his categorization of war as an
(25) institution. Like other social institutions, such as slavery
and duelling, which are discussed, war can disappear.
However, the similarities between these institutions are
insufficiently described for the analogy to hold—assuming,
that is, that one accepts that duelling and slavery have
(30) truly vanished. While Mueller does present some
compelling historical evidence in the form of a rushed
analysis of the Cold War, he seems to have forgotten that
one cannot assume that a historical pattern will continue
to repeat.

DO NOT DISTRIBUTE
116 |
Lesson 5

Passage Notes
Use this space to take notes on the passage.

55

Issue of the Passage:

What’s the author’s opinion of each scholar?

Main Idea of the Passage:

How is the passage structured?

1. The primary purpose of the passage is to What misunderstanding of


the passage is assumed by
describe how the works of two scholars helped many of the wrong answers?
contribute to the disintegration of the Soviet Union
argue that Fukuyama’s explanation for the end of history
is more likely correct than Mueller’s
assess the relative strengths of two competing
hypotheses describing the decline of communism
offer a critique of two scholarly works that emerged out of
the exhilaration that surrounded the end of the Cold War
advance the view that liberal democracy and capitalism
are the highest forms of social organization

DO NOT DISTRIBUTE
| 117
GMAT IN CLASS MANUAL

Finding the Main Idea: Change or Problem


Sometimes, you can find the main idea by thinking about problems or changes
discussed in the passage.
55
Is this paragraph Until recently, corporate ideology in the United States
describing a problem or a has held that bigger is better. This traditional view of
change?
the primacy of big, centralized companies is now being
challenged as some of the giants of American business
(5) are being outperformed by a new generation of smaller,
streamlined businesses. If it was the industrial revolution
that spawned the era of massive industrialized companies,
then perhaps it is the information revolution of the 1990s
that is spawning the era of the small company.
(10) For most of this century, big companies dominated an
American business scene that seemed to thrive on its own
grandness of scale. The expansion westward, the growth
of the railroad and steel industries, an almost limitless
supply of cheap raw materials, plus a population boom
(15) that provided an ever-increasing demand for new products
(although not a cheap source of labor) all coincided to
encourage the growth of large companies.
But rapid developments in the marketplace have
begun to change the accepted rules of business and
(20) have underscored the need for fast reaction times.
Small companies, without huge overhead and inventory,
can respond quickly to a technologically advanced age
in which new products and technologies can become
outmoded within a year of their being brought to market.
(25) Of course, successful emerging small companies face
a potential dilemma in that their very success will tend
to turn them into copies of the large corporate dinosaurs
they are now supplanting. To avoid this trap, small
companies may look to the example of several CEOs of
(30) large corporations who have broken down their sprawling
organizations into small, semi-independent divisions
capable of making their way into the twenty-first century.

DO NOT DISTRIBUTE
118 |
Lesson 5

General Questions
General questions ask about the passage as a whole. Use your sense of the passage’s
main idea and structure to work these questions, and refer to the passage as needed.
55

Main Idea/Primary Purpose Questions

1. The primary purpose of the passage is to


present evidence that resolves a contradiction in
business theory
discuss reasons why an accepted business pattern is changing
describe a theoretical model and a method whereby that
model can be tested
argue that a traditional ideology deserves new attention
resolve two conflicting explanations for a phenomenon

Tone/Attitude Questions

2. The author’s attitude toward emerging small companies


is one of
disappointment
optimism
uncertainty
criticism
elation

Structure Questions

3. Which of the following best describes the organization of the


first paragraph of the passage?
A conventional model is described and an alternative is introduced.
Every part of the answer
An assertion is made and a general supporting example is given. must match the structure of
Two contradictory points of view are presented and evaluated. the paragraph.
A historical overview is given to explain a phenomenon.
A new theory is described and then qualified.

DO NOT DISTRIBUTE
| 119
GMAT IN CLASS MANUAL

POE for General Questions


Good answers to a general question fit with the scope of the passage as a whole.
Eliminate choices that:
55 • Use recycled language. Most of these answers will also be too narrow.
• Use extreme language. Be wary of answers that make sweeping claims.
These answers can be considered too broad.
• Make bad comparisons about items in the passage. The items may be from
the passage but is the comparison?
• Reverse the main idea.
• Make emotional appeals or call on outside knowledge.

Pace Yourself Wisely


Adjust your approach to reading comprehension passages and questions according to where
the passage appears in the Verbal section.

Passage #1
• Passages early in the test matter most. Make sure you spend enough time to
get the questions right.
• Always go back to the passage. Make sure you can support your answer
with proof or evidence. Spend as much time here as you need.
Passage #2
• The second passage counts less than the first in determining your final
score. However, it is still important to spend a good amount of time
backing up your answers with evidence from the passage.
• Do not get stuck on a killer question. Pay attention to your timing in
relation to the suggested time allotted for each part of the section.
Passage # 3
• In order to establish a plan for the third passage, evaluate your strengths
and weaknesses.
• If you excel at reading comprehension, invest the time to get these
questions right. They will be getting more difficult, so it may take you just
as long to get through these questions as it did to get through those in the
first two passages.
• If reading comprehension is your weakest area, pick up your pace on the
third passage. Push through these questions to give yourself more time to
spend on your stronger areas.
Passage #4
• The final passage affects your score the least. Spend more time here if this is
your strongest area of the test and less if it is your weakest.
• If time is running out, dispense with working the passage. Go directly to
the questions, and read only what you need to find the answers.
• If you only have a few minutes left in the section, eliminate answers with
extreme wording or those that make predictions, and pick an answer
choice.

DO NOT DISTRIBUTE
120 |
Lesson 5

Math 5
Even the toughest GMAT geometry problems test fairly basic ideas, but those
basic ideas can be combined in unusual ways.
55

Geometry: The Basic Approach


There are some standard things that you should always do when working a
geometry question.
N

O M

1. In the figure above, if MN = 2, then the area of the circle with


the center O is
π

4
π
For a 45°: 45°: 90° triangle,
2
the ratio is 1:1: 2 .
π

DO NOT DISTRIBUTE
| 121
GMAT IN CLASS MANUAL

For a 30°: 60°: 90° triangle, 2. If equilateral triangle XYZ has a perimeter of 12, what is the area
of triangle XYZ ?
the ratio is 1: 3 :2.

2 3
55
4 3

12

8 3

Geometry Basic Approach


• Redraw the figure that is provided on your noteboard.
• If no figure is provided, draw your own.
• Show any information from the problem on the figure.
• Write down any formulas that you need.
• Keep the marker moving!

DO NOT DISTRIBUTE
122 |
Lesson 5

Data Sufficiency Figures


Be careful in how you interpret the figures that accompany data sufficiency
problems.
55
B

D
O

A E C

1. In the figure above, the circle with center O is inscribed in


triangle ABC. What is the area of the circle with center O ?

(1) The area of the shaded region is 2π.

(2) The degree measure of ∠DOE is twice that of ∠DAE.

DO NOT DISTRIBUTE
| 123
GMAT IN CLASS MANUAL

The Basic Approach and Geometry Data Sufficiency


For data sufficiency questions, you can link the Basic Approach to the Pieces of the
Puzzle approach.
55
R
Q

What do you know? 1. What is the circumference of the circle above?


What do you need?
(1) The length of arc QRS is one-sixth of the circumference.

(2) The length of chord QS is 6.

C B

2. In the figure above, A and B are square regions. If the perimeter


of square region B is 20, what is the area of triangular region C ?

(1) The hypotenuse of triangular region C is 5 5.

(2) The area of square region A is four times the area of square region B.

DO NOT DISTRIBUTE
124 |
Lesson 5

Plugging In for Geometry


Don’t forget that Plugging In can be a useful tool—even on geometry problems!

55
y
x

1. The figure above shows the cross section of a water pipe


that has a diameter of x inches and a length of l inches. If the
walls of the water pipe are a uniform y inches thick, which of
the following is an expression for the maximum volume of
water that can be carried by this cross section of the pipe?
 x2 2 
πl − xy + y 
 4 
πx l
2

( 2
π l x − 2 xy + y
2
)
(
π l x − 4 xy + y
2 2
)
 x2 2 
πl  − 2 xy + 4 y 
 4 

DO NOT DISTRIBUTE
| 125
GMAT IN CLASS MANUAL

2. A rectangular horse corral is constructed with one of its long


sides against a barn. If the length of the corral’s long side is twice
that of its short side and the ratio of the corral’s area to the length
of fence required for the three fenced sides is 10 to 1, what is the
55 length of the long side?

20
40
60
80
90

x z
R T

3. In triangle RST in the figure above, does RS = ST ?

(1) y = 180 – 2x

(2) x + y = y + z

DO NOT DISTRIBUTE
126 |
Lesson 5

Combining Concepts
Many GMAT geometry questions combine concepts. Two of the most common
ways of doing so are overlapping figures and shaded regions.
55

Overlapping Figures
The key to solving problems with overlapping figures is to determine what the
two shapes have in common.

B C

A D
What do the square and
circle have in common?
1. Rectangle ABCD has length 8 and width 6. What is the area
of the circle with center O ?

10π
25π
50π
64π
100π

DO NOT DISTRIBUTE
| 127
GMAT IN CLASS MANUAL

N
M y

55
O
x
L

2. If the area of the circle above with center O is 64π, what is the
area of triangle LMN ?

(1) x = 2y

(2) OL = LM

DO NOT DISTRIBUTE
128 |
Lesson 5

Shaded Regions
The key to solving most shaded region questions is to focus on the unshaded part.

C 55

B D

A E
1. In the figure above, ABCDE is a regular pentagon and arc AC
is a section of the circumference of a circle with center at B
and radius 10. What is the perimeter of the shaded region?

 3π 
10  2 +

 5 
 3 
10  3 + π 

 5 
10 (2 + 3π)

10 (5 + 3π)

30 (1 + π)

DO NOT DISTRIBUTE
| 129
GMAT IN CLASS MANUAL

P Q

55
R
O

2. In the figure above, OPQR is a square. If O is the center of the


circle, and the distance between point P and point R is 4 2 ,
what is the area of the shaded region?

16 – 8π
16 – 4π
32 – 8π
32 – 2π
64 – 12π

Shaded Area = Total Area – Unshaded Area

DO NOT DISTRIBUTE
130 |
Lesson 5

55

O C


3. In the rectangular coordinate system shown above, the circle
is centered at the origin. If the coordinates of point A are
(1, 3 ) and the coordinates of point B are ( 4, 4 3 ), what is
the area of the shaded region?
π
8 3−
3

2

8 3− π
3

8 3 − 4π

2
16 3 − π
3

16 3 − 4 π

DO NOT DISTRIBUTE
| 131
GMAT IN CLASS MANUAL

Coordinate Geometry
Many coordinate geometry problems are really about right triangles. If you need to
determine the length of a line that’s not parallel to the x-axis or the y-axis, turn it into
55 a right triangle problem.

1. In the rectangular coordinate system, if the points D(–1,–1),


E(–1,1), F(a,1), and G(a,–1) are the vertices of a rectangle with a
diagonal length of 2 5 and a > 0, then what is the value of a ?

3 5

7 3

DO NOT DISTRIBUTE
132 |
Lesson 5

Rise y −y
Slope = = 2 1
Run x2 − x1
where (x1, y1) and (x2, y2) are points on the line. 55
It doesn’t matter which coordinate you call y2 or y1 as
long as you use the corresponding x2 and x1.

2. If line l passes through the origin and the point (a,b), what is
b
the value of ?
a
(1) (5,2) is on line l.
5
(2) Line m, which is defined by the equation y = − x + 4 is
2
perpendicular to line l.

3. Line m is tangent to the circle with center (0,0) at (−3,y) such


that its y-intercept is positive. If the radius of the circle is 5,
what is the slope of line m ?
−4

4

3
3

4
4

5

DO NOT DISTRIBUTE
| 133
GMAT IN CLASS MANUAL

y = mx + b
x, y = variables that stand for the coordinates of any point on
55 the line
m = slope of the line
b = y-intercept = y-coordinate of the point (0,b) where the line
crosses the y-axis

3
2
1
1 2 3 4 5 6
-1
-2
-3

4. In the rectangular coordinate system shown above, the shaded


region is bounded by straight lines. Which of the following is an
equation of one of the boundary lines?

y=1

y=2

x=0

2
y= x–2
3
3
y= x–2
2

DO NOT DISTRIBUTE
134 |
Lesson 5

5. What is the x-intercept of the line defined by the equation


y = 4x + 5 ?

(5,–4) 55

(4, 45 )
(– 45 ,0)
( 45,0)
(0,– 45 )

6. The points (p,q) and (p – 2,q + c) are both on the line


expressed by the equation y = 2x + 5. What is the value of c ?

–4
–1
1
2
4

DO NOT DISTRIBUTE
| 135
GMAT IN CLASS MANUAL

Homework Review
Use this chart to note any questions you have from the reading or examples in the homework.

Page # Question # What que stion do you have ?


55

DO NOT DISTRIBUTE
136 |
Lesson 5

Practice
B
105

A 30 C

Note: Figure not drawn to scale.

1. What is the perimeter of the triangle above? 3. In the triangle above, if BC = 4 2, then what is
the area of ∆ABC ?
3+ 3
64
2
3 16 +16 3

8+8 3
2+ 2
8+4 2
3+ 3
8 2
3+ 5

A (-3,5)
P

2
O
2

R Q B
2
2

2. In ∆PQR above, PQ = 4. In the figure above, AOB is the diameter of a


circle centered at O. If the coordinates of A are
1 (–3,5), then the coordinates of B are

2
(–3,–5)
2 (–3,5)

2 (3,–5)
(3,5)
1
(5,3)
5. A circle with center (0,0) and radius 8 will pass
2 through all of the following points EXCEPT
(–8,0)
2 (0,–8)
(0,8)
(8,0)
(8,8)

DO NOT DISTRIBUTE
| 137
GMAT IN CLASS MANUAL

6. The slope of a line containing points (2,–3) and 9. The length of an edge of cube A is 5% greater
(4,p) is –1. What is the value of p ? than the length of an edge of cube B. If the
volume of cube B is 27 cubic centimeters, then
–6 which of the following is nearest to the volume
–5 of cube A ?
1 23.1
5 25.65
12 27.125
28.35
B
31.25
x
y +2− b 2
10. For the line with equation = m + , m is
x x
not zero. If the line is rotated 90°, then the slope
60
A C of that line would be

7. In the figure above, if A, B, and C are points 1



on the circle, and if AB = AC, then what is the m
value of x ? 1

m
40
45 m
50
55 –m
60
m–2
8. A square is inscribed in a circle with area πr2.
What of the following is an expression for the
area of the square?

r2

2
2r 2

r2

2r2

4r2

DO NOT DISTRIBUTE
138 |
Lesson 5

11. A sphere with a radius of 5 is hollowed out at 13. In the rectangular coordinate system, the
coordinate of vertex A of triangle ABC is (0,56).
the center. The part removed from the sphere If vertex B lies at the origin, how many points on
line AC have integer values for both their x and y
has the same center, and a radius of 3. What
values?
fractional part of the original sphere remained? (1) Vertex C of triangle ABC lies on the x-axis and
(The formula for the volume of a sphere the triangle has an area of 196.
(2) The coordinates of vertex C are (x,0) and x >
4 3 0.
is V = πr )
3
2 B

5
16 C

25 4
27 A

125 12 D
98

125 14. In the figure above, if the area of triangle ABD is
3 30, what is the sum of AB and BC ?

5 5
8
12. A certain cube floating in a bucket of water has 12
between 80 and 85 percent of its volume below 16
the surface of the water. If between 12 and 19
16 cubic centimeters of the cube’s volume is
above the surface of the water, then the length B C
of a side of the cube is approximately
4
5 6
7
8
9

A D

15. In the figure above, what is the area of


square ABCD ?
4
10
12
18
24

DO NOT DISTRIBUTE
| 139
GMAT IN CLASS MANUAL

y D

R(3,10) C

A B

18. In the rectangular solid above, if AB = 5, what


is the surface area of the solid?
S(9,2)
(1) BC = 8
x (2) The volume of the solid is 80.

16. In the rectangular coordinate system shown r


above, what is the distance between point R
and point S ?
10
x
6 3
12
9 2
20 19. A rectangular label of width x has been
17. What is the greatest distance, in inches, wrapped around the cylinder above, encircling
between any two corners of a rectangular box the cylinder without overlap. If the radius of the
with dimensions of 6 inches, 8 inches, and cylinder is 6, and the label has the same area
10 inches? as the base of the cylinder, then what is the
value of x ?
10
12 3
10 2 5
10 3 6
24 6π

DO NOT DISTRIBUTE
140 |
Lesson 5

Answers and Explanations


1. D Apply the 30:60:90 triangle relationship to this 8. D Draw the diagram, and make sure the square is
triangle to find that the hypotenuse equals 2 and inside the circle. Now, plug in r = 4. The area of
the long leg (the base) equals 3 . Now, just add
the circle is 16π. The diameter of the circle is
up the three lengths to find the perimeter.
8, which is also the diagonal of the square. The
2. C Because the two non-hypotenuse sides of diagonal of the square creates two 45:45:90
this right triangle are the same length, the triangles, so use that relationship to find the
triangle is a 45:45:90 triangle. Use the 45:45:90
length of the side of the square. Each side
relationship to find that the hypotenuse equals
2 measures 4 2 , so the area of the square is 32
× 2 , or 1.
2 (target answer).
3. C When you drop a perpendicular from point 9. E This is a very work-intensive problem, but there
B to side AC (call that point D), you create a is no way around the work. First, find the side
for cube B, which is 3 since its volume is 27. The
30:60:90 triangle (∆ABD) and a 45:45:90
length of the edge of cube A is 5 percent greater
triangle (∆BCD). Since BC = 4 2 , BD = AC than the length of the edge of cube B. Thus, the
= 4. Now look at the 30:60:90 triangle: Since length of the edge of cube A is 105 percent of cube
B. We can find the length of the side of cube A
BD = 4, AD = 4 3 . The base AC = 4 + 4 3 by multiplying 3 times 1.05. To find the volume
and the height BD = 4. Use the area formula: of cube A, we must raise the side of cube A to the
1
2
)
(4 + 4 3 (4) = 8 + 8 3 . power of 3. (3.15)3 = 31.25.

4. C B is in the quadrant where x is positive and 10. B Since we are given the equation of a line in
y is negative. The only choice with a positive a strange form, it’s probably a good idea to
x-coordinate and a negative y-coordinate is (C). rewrite it in the more familiar form of y = mx + b.
Rewriting the given equation reveals that we have
been given the standard line equation. Now we
5. E Each of the points in the wrong answer choices
know the slope is m, and the only question is
is 8 units from the origin. However, the point what happens to it when it is rotated 90°. The
(8,8) is 8 2 units from the origin, which can be best way to determine that might be to draw the
picture of a line and then rotate it 90 degrees,
determined using either the Pythagorean which would reveal that the slope has become
Theorem or the 45:45:90 triangle relationship. negative, which means we can eliminate (A) and
(C). You can also eliminate (E) because for that
6. B Se t u p t h e s l o p e f o r m u l a , a n d s o l ve : choice to be correct the slope would not only
p − ( −3 ) rotate but would also change its value, and that
p+3
= −1 , which becomes = −1 does not happen here. The last two choices have
4−2 2 only one difference: whether rotating the line
making p = –5. makes the slope merely negative or the negative
7. E Since AB = AC, ∠B = ∠C. The measure of ∠A reciprocal. Plugging In should resolve that.
is 60°, so ∠B + ∠C = 120°. Thus, ∆ABC is
equilateral, and each angle measures 60°.

DO NOT DISTRIBUTE
| 141
GMAT IN CLASS MANUAL

11. D This one sounds weird, but don’t let yourself 12. A The best way to approach this problem is to Plug
In The Answers since the answers give us the side
be thrown off by the inclusion of the sphere. of the cube. If we start with the middle choice,
(C), then we have a cube with side 7. If the cube
Remember, any time GMAC asks you about has a side of 7, then it will have a volume of 343.
We are told that between 80% and 85% of the
volume is below the surface of the water, which
a strange figure, they will supply you with the means that between 15% and 20% of the volume
is above the surface. If the volume of the cube is
relevant formula, as is done for this problem. 343, then 20% is about 68 and 15% is about 51.
Neither of these numbers is between the 12 and
From here, it’s a lot like a shaded region question. 16 cubic centimeters that are supposed to be above
water, so clearly this can’t be the answer. Since the
numbers are too large, we need to try something
Start by figuring the area of the whole sphere,
smaller. Pick one of the smaller choices and try
500 again.
using the formula: It’s π . Now, figure out
3 13. A Start by translating the question to understand
the volume of the smaller sphere. It’s 36π . which pieces of the puzzle are given and which
pieces are needed. The question tells us where
two vertices are, point B at (0,0) and point A
Now, look carefully at the question: We need to
at (0,56). To answer the question, we need the
coordinates of the last point. Statement (1) tells
determine what fractional part of the original sphere us that the third vertex lies on the x-axis which
means that triangle ABC is a right triangle. We
remains. First, determine the volume of the remain- also know the area of the triangle so we can
calculate the length of the base of the triangle.
ing portion of the sphere: Remember that we already know that the height
π π π
of the triangle is 56. Knowing the length of the
base, we can find the coordinates of vertex C. We
To determine what fractional part remains, we now know two points on line AC which means
we can calculate the slope of line AC which
need to take the remaining portion and divide allows us to answer the question. So, AD. State-
392 ment (2) only tells us that vertex C lies on the
π
392 98
it by the original volume: 3 = = . x-axis but we have no way of determining the
500 500 125 length of the triangle’s base. So, statement 2 is
π
3 insufficient.

DO NOT DISTRIBUTE
142 |
Lesson 5

14. D Since you know that the base of triangle ABD is 17. C There’s a formula that can be used to find the
12 and that its area is 30, you can use the formula diagonal of a rectangular solid: d2 = a2 + b2 + c2,
for the area of a triangle to find that the height where a, b, and c are the dimensions of the
of ABD is 5. Be sure to add this information to rectangular solid. In this case,
the figure. For triangle ABD, you now know AD d2 = 62 + 82 + 102 = 200. Now, just take the square
is 12 and BD is 5. You can use the Pythagorean root of 200 and simplify to find the distance.
Theorem to find that the length of AB is 13.
(Remember, however, that GMAC likes to test the 18. C Start by applying the Pieces of the Puzzle
5:12:13 special right triangle so it’s best to have approach. In this case, you know one of the three
this relationship memorized.) Triangle BCD turns dimensions of the box. To find the surface area,
out to be a 3:4:5 special right triangle, another you need to find the area of each surface. So,
common relationship that GMAC likes to test. you’ll need the statements to provide the other
So, AB + BC = 13 + 3 = 16. two dimensions of the box. Statement (1) provides
only one additional dimension. So, statement (1)
is insufficient and the possible answers are B, C,
15. D Start by labeling everything that you know on
or E. To evaluate statement (2), the formula for
the figure. In this case, you’ll want to add that the volume of a rectangular solid is V = lwh. The
statement provides the value for the volume and
∠ADC = 90° and that ∠CAD = ∠ACD = 45°.
question stem provided one of the three dimen-
You also want to write down the formula for sions. Statement (2) is also insufficient. When the
the area of a square, A = s2. So, you need to find statements are combined, however, you can now
use the volume formula to find the value of the
the side of the square. Use the 45:45:90 triangle missing dimension.
relationship to find that the side of the square is
6 19. A Start by drawing the figure and labeling the radius
. Plugging that value into the formula gives as 6. Next, find the area of the base of the cylinder.
2
an area for the square of 18. Since the base of the cylinder is a circle, its area is
A = πr2 = 36π. The label is a rectangle, so the for-
16. A Finding the distance between two points in mula for its area is A = lw. The length of the label
a plane requires the construction of a right is the same as the circumference of the cylinder,
triangle. First, draw a line connecting points R and so substitute 2πr for the l in the formula for the
S. Next, draw a vertical line down from R and a area of the label. You can also substitute x for w in
horizontal line left from S. The point of the formula to get: A = 2πrx = 36π. Finally, just
intersection has coordinates (3,2). The put in the value of the radius, 6, and solve to find
vertical leg of the triangle has a length of 8 and the the x = 3.
horizontal leg has a length of 6. The hypotenuse
has a length of 10.

DO NOT DISTRIBUTE
| 143
DO NOT DISTRIBUTE
Lesson 6

Lesson 6
66

Reading Comprehension 2
The Basic Approach
Step 1: Work the passage.
Read for the passage’s main idea, and get a sense of its structure.

Step 2: Understand the Question Task.


Look for the question’s subject and the words that indicate its task.

Step 3: Find the Information that Addresses the Task.


Once you understand the question task, use the question’s subject to locate infor-
mation in the passage that addresses the question task.

Step 4: Use POE to find the answer.


Be skeptical of each answer choice. Try to find reasons to eliminate the answer.
Eliminate choices that:

• Use recycled language.


• Use extreme language. Be wary of answers that make sweeping claims.
• Make bad comparisons about items in the passage. The items may be
from the passage but is the comparison?
• Reverse the main idea.
• Make emotional appeals or call on outside knowledge.

DO NOT DISTRIBUTE
| 145
GMAT IN CLASS MANUAL

Use one of the passage


mapping strategies to The wealth of morphological, phonetic, and word
find the main idea of this similarities among certain languages has led linguists to
passage. recognize the unity of the well-defined family of languages
66 called the Aryan or Indo-European family. Yet even this
(5) latter term is largely a misnomer. This group of languages
spreads over an enormous range virtually without
interruption, reaching from Central Asia to the fringes of
westernmost Europe. The westernmost terminus of the
family is Celtic, while its easternmost representatives were
(10) the Tokharian languages, a pair of tongues once spoken
by the residents of the Tarim River Basin in Western China
and unearthed in documents written more than a thousand
years ago.
So remarkable and definite are the similarities among
(15) these languages that linguists are convinced they
all derived from an earlier language spoken by some
community in the prehistoric past. While we know that
Latin began as a rustic dialect in the province of Latium,
no one knows where proto-Aryan was first spoken. Some
(20) speculate that it was first used in Southern Russia, while
still others point to the Iranian plateau as a potential
cradle. Though some philologists believe that the Old
Indic and Persian of the Avesta contain the most archaic
features of Aryan found to date, this does not necessarily
(25) fix the habitat of these early Aryan-speaking peoples
closer to Asia than to Europe. Consider Icelandic. Though
this language has strayed far from its original source, it
preserves many of the characteristics discarded by those
who remained behind.
(30) From the existing evidence, only one thing seems
certain. By the time of Vedic hymns, the first recorded
instance of Aryan, those tribes speaking this early
language had already begun their widespread dispersal.

Issue of the Passage:

Main Idea of the Passage:

How is the passage structured?

DO NOT DISTRIBUTE
146 |
Lesson 6

Specific Questions and POE


Specific questions ask about a fact or detail from the passage.

Detail Questions
66
These questions ask you to find something the author said.

Question Subject:_____________________________

Task Word:___________________________________

1. The “misnomer” (line 5) refers to To learn about the POE tools,


think about why somebody
the fact that not all Indo-Europeans are of Aryan descent might pick each of the wrong
the fact that the family of languages goes beyond the answers.
boundaries implied by the name
the fact that the family of languages actually originated
in Western China
the fact that the languages are no longer spoken
the fact that proto-Aryan was spoken in the area known
as Latium

POE Notes:

Detail Purpose
These questions ask you why the author included a piece of information in the
passage.

Question Subject:_____________________________

Task Word:___________________________________
Is this question task “What” or
2. The author mentions Icelandic in order to “Why”?

provide support for the contention that early


Aryan-speaking peoples did not necessarily live in Asia
provide another example of the archaic features found in
the Avesta
provide evidence for the inception of proto-Aryan
counter the speculation that proto-Aryan was first
spoken in Russia
provide an alternative explanation for the similarities
found in early languages

POE Notes:

DO NOT DISTRIBUTE
| 147
GMAT IN CLASS MANUAL

Infer/Imply/Suggest Questions
These questions are really just another “what” task even though it sounds like you
are supposed to do more.

Question Subject:_____________________________
66
Task Word:___________________________________

3. Which of the following does the passage imply about the


Tokharian languages?
They date to a far earlier period than does Celtic.
They possess many of the same features as did the Old
Indic and Persian of the Avesta.
They were once spoken over a far greater range than they
are today.
They undermine the accuracy of the name “Indo-European”
for their particular family of languages.
They are remarkably similar to the languages spoken by
current residents of the Tarim River Basin.

POE Notes:

EXCEPT Questions
Here you have four questions for the price of one.

Question Subject:_____________________________

Task Word:___________________________________
How does the word ‘except’
change the process for this 4. According to the passage, all of the following characterize
question?
Indo-European languages EXCEPT
similarities in the structure of words and in the sounds employed
common origins in some prehistoric community
widespread, intercontinental dispersal
recorded instance of early use in ancient literature
preservation of characteristics discarded by earlier speakers

POE Notes:

DO NOT DISTRIBUTE
148 |
Lesson 6

Reading Comprehension POE: Learn to be suspicious of


answer choice characteristics that are likely to make the answer
wrong. Start with specific words in the answer choice and try
to place those words into a POE category. POE can lead to an
explanation why the answer is incorrect.
66
Explanation: The reason an answer choice is incorrect.
Coming up with explanations is often more time-consuming
than POE. Explanations start with POE.

Two Pass POE

Pass One

• Be suspicious of every answer.


• Look for characteristics that are likely to make an
answer wrong, such as recycled language. See if
you can use those characteristics to explain why
the answer is wrong.

Pass Two

• Down to two? Use POE aggressively.


• Eliminate the answer that sounds more like the
passage.
• Eliminate the answer that uses the stronger
language.
• Eliminate the answer that makes a comparison.

DO NOT DISTRIBUTE
| 149
GMAT IN CLASS MANUAL

The tropical leaf-cutter, or attine, ant provides a


remarkable example of complex symbiosis involving
several species. The ants cultivate a fungus in
underground caverns to serve as a source of food. DNA
(5) analysis reveals that the fungi in attine gardens around
the world are clones of a single source. The ants do not
66 allow their fungus crop to develop fruiting bodies, the
means by which plants engage in sexual reproduction.
Instead, a queen ant starting a new nest takes a sample
(10) of the fungus from the old nest to start the new garden,
spreading the fungus vegetatively, or asexually.
For many years, the phenomenon of sexual reproduction
puzzled biologists, as it passes only half of the parents’
genes to the succeeding generation and requires a more
(15) complex mechanism than does asexual reproduction.
What benefit of sexual reproduction would outweigh
these limitations? One likely answer is that it provides a
defense against parasitic attack. Simple parasites, such as
bacteria or molds, mutate rapidly and pose a challenge to
(20) other species developing defense mechanisms. The rapid
evolution of the attacking species provides an advantage
in this biological arms race. Sexual reproduction allows
the more complex species to shuffle its genes between
generations and, as a result, evolve quickly enough to
(25) match the parasites’ mutations.
A clonal monoculture, such as that of the attine, should
be highly susceptible to parasites, yet preliminary research
suggested no evidence of such a problem in the ants’
gardens. More comprehensive studies showed that the
(30) Escovopsis mold, a parasite related to the “green mold”
known to commercial mushroom farmers, is present in
the ants’ crops and poses a serious threat to the fungus.
However, the attine ants provide the defense mechanism
lacking in the fungus’s asexual reproduction by means
(35) of a bacterium that grows in patches on their skin. This
actinomycete bacterium produces an antibiotic used to
control the mold and limit its destructive effects on the
ants’ food source.

DO NOT DISTRIBUTE
150 |
Lesson 6

1. The phrase “clonal monoculture” (line 26) refers to which of


the following?
The Escovopsis mold
A species of leaf-cutter ant
Sexual reproduction
A fungus 66
An antibiotic bacterium

2. Which of the following can be most reasonably inferred about


reproductive strategies from the passage?
The fruiting bodies found on numerous fungi are a
means of vegetative reproduction.
Plants that rely on sexual reproduction are highly
susceptible to parasitic attack by bacteria and molds.
Sexual reproduction is an inefficient method for
transmitting a parent’s genes to its offspring.
The bacterium found on patches of green mold
produces an antibiotic substance used by commercial
mushroom farmers.
Parasitic bacteria use sexual reproduction as a means to
achieve rapid mutation.

3. The author describes the interaction between a parasite and


a sexually reproducing organism as an “arms race” (line 22) in
order to
emphasize the aggressive nature of the parasitic organism
warn of the dangers of biological weapons
underscore the need for the attine ants to defend their gardens
point out the devastating effects of the Escovopsis mold
depict the result of rapid evolution by both organisms

4. The passage suggests which of the following about the


fungus grown by the attine ants?
By shuffling its genes, it is able to mutate rapidly in
response to parasitic threats.
Due to its asexual reproduction, it is susceptible to
attack by the actinomycete bacterium.
It could potentially engage in sexual reproduction.
It is closely related to the crops grown by commercial
mushroom farmers.
Without the care of the attine ants, it would quickly
become extinct.

DO NOT DISTRIBUTE
| 151
GMAT IN CLASS MANUAL

Critical Reasoning 2
There are a few types of critical reasoning questions that do not require you to identify
the conclusion, premise, and gap.

66
Inference Questions
Inference questions ask you to infer or conclude something based on the passage.

Step 1: Read and identify the question.


Inference questions typically ask:

• Which of the following can be inferred from the information above?


• Which of the following conclusions is best supported by the passage?
• Which of the following conclusions could most properly be drawn from
the information above?
• Which of the following must be true on the basis of the statements
above?

Step 2: Work the argument.


Read the passage. There’s no need to look for the conclusion and premise.

Step 3: Predict what the answer should do.


Most of the time, you will not be able to come up with an answer in your own words.
Simply keep in mind that you want the answer best supported by the facts.

Step 4: Use POE to find the answer.


Look for an answer that must be true. Avoid answers that:

• Are out of scope


• Use extreme language

DO NOT DISTRIBUTE
152 |
Lesson 6

1. According to a recent study, fifteen corporations


in the United States that follow a credo of social
responsibility are also very profitable. Because
of their credos, these fifteen corporations
give generously to charity, follow stringent
environmental protection policies, and have
vigorous affirmative action programs.
66
Which of the following can be correctly inferred
from the statements above?
Following a credo of social responsibility
helps to make a corporation very profitable.
It is possible for a corporation that follows a credo
of social responsibility to be very profitable.
A corporation that gives generously to
charity must be doing so because of its
credo of social responsibility.
Corporations that are very profitable tend to
give generously to charity.
Corporations that have vigorous affirmative
action programs also tend to follow
stringent environmental protection policies.

DO NOT DISTRIBUTE
| 153
GMAT IN CLASS MANUAL

2. The two divisions of a high-tech company have


performed quite consistently over the past five years. In
each year, the telecommunications equipment division
accounted for 35 percent of profits and 15 percent of
revenues, and the high-speed internet division made up
the balance.
66
Which of the following can properly be inferred regarding
the past five years from the information above?
The telecommunications equipment division
has made higher profits per dollar than the
high-speed internet division.
Sales for both divisions have remained flat over
the five years.
The high-speed internet market involved tougher
competition than the telecommunications
equipment market during the past five years.
Management devoted a greater number of
company resources to the telecommunications
equipment division than to the high-speed
internet division over the past five years.
More profitable products made up a higher
percentage of the products offered by the
telecommunications division.

DO NOT DISTRIBUTE
154 |
Lesson 6

3. A combination of anxiety and external pressure


leads to nausea. All the auditioners for the new
reality show Wanna Be’s suffer from external
pressure. Some of the auditioners feel anxiety
about performing well for the producers, but others
do not feel anxious. The producers of Wanna Be’s
select only those contestants who feel anxiety. 66
Which of the following conclusions is most strongly
supported by the passage above?
The auditions for Wanna Be’s cause more
performers to feel external pressure than
do auditions for other shows.
Most of the people who audition become
contestants on Wanna Be’s.
There is more nausea among auditioners
for Wanna Be’s than among producers of
Wanna Be’s.
No auditioner who doesn’t feel anxiety has nausea.
All of the auditioners for Wanna Be’s who
become contestants have nausea.

DO NOT DISTRIBUTE
| 155
GMAT IN CLASS MANUAL

4. In broad thermodynamic terms, the distinction between


solar energy and energy derived from fossil fuel is
artificial. Fossil fuel molecules represent the decayed
remains of plants. All of the energy these fossil fuels
contain once resided in the sun and was, so to speak,
trapped by plants here on Earth through the process of
66 photosynthesis, whereupon it was housed, principally,
within the carbohydrate molecules of which the plants
were composed. The process of burning unleashes
that energy, and when we run our lights, factories, and
automobiles by burning fossil fuels _______________.

Which of the following is the most logical completion of


the passage above?
we deplete our stores of an ever more
precious resource
we use energy that is, in fact, derived from the sun
we spend the legacy left to us by our prehistoric ancestors
we mimic the process of energy generation that exists in
the sun
we return to the sun that which originally resided there

DO NOT DISTRIBUTE
156 |
Lesson 6

Integrated Reasoning 2
To answer some Integrated Reasoning questions, you use the same skills that you
use when answering Critical Reasoning questions.

66
Integrated Reasoning and Inferences
Many types of Integrated Reasoning questions ask you to make an inference based
on the data presented.

The table displays performance statistics for six different luxury cars.

Model Curb Weight Horsepower 0–60 Highway Fuel


(lbs) acceleration Economy
(sec) (mpg)
Series 3 3,609 230 7.1 26
250 IS AWD 3,651 204 8.3 27
4A 3,704 211 6.3 29
Kappa Class 3,737 228 7.1 25
Gamma x37 3,822 328 5.6 25
LT 3,968 305 5.3 26

For each of the following statements, select Yes if the statement


can be shown to be true based on the information in the table.
Otherwise, select No.

Yes No
Additional sorts of the table
The car with the least horsepower also takes the
data are provided on the next
most time to accelerate from 0 to 60. page.

If the curb weight of the 4A were reduced, its


highway fuel economy would increase.

For each of the cars listed, as horsepower


increases, 0 to 60 acceleration gets faster.

DO NOT DISTRIBUTE
| 157
GMAT IN CLASS MANUAL

Sort By: Horsepower

Model Curb Weight Horsepower 0–60 Highway Fuel


(lbs) acceleration Economy
(sec) (mpg)
66 250 IS AWD 3,651 204 8.3 27
4A 3,704 211 6.3 29
Kappa Class 3,737 228 7.1 25
Series 3 3,609 230 7.1 26
LT 3,968 305 5.3 26
Gamma x37 3,822 328 5.6 25

Sort By: 0–60 acceleration (sec)

Model Curb Weight Horsepower 0–60 Highway Fuel


(lbs) acceleration Economy
(sec) (mpg)
LT 3,968 305 5.3 26
Gamma x37 3,822 328 5.6 25
4A 3,704 211 6.3 29
Kappa Class 3,737 228 7.1 25
Series 3 3,609 230 7.1 26
250 IS AWD 3,651 204 8.3 27

Sort By: Highway Fuel Economy (mpg)

Model Curb Weight Horsepower 0–60 Highway Fuel


(lbs) acceleration Economy
(sec) (mpg)
Gamma x37 3,822 328 5.6 25
Kappa Class 3,737 228 7.1 25
Series 3 3,609 230 7.1 26
LT 3,968 305 5.3 26
250 IS AWD 3,651 204 8.3 27
4A 3,704 211 6.3 29

DO NOT DISTRIBUTE
158 |
Lesson 6

Integrated Reasoning and Arguments


Some Integrated Reasoning questions are really just Critical Reasoning
questions asked in a different format.
1. Allison: Childhood obesity is rampant in the public school
system. In order to combat this dangerous trend, we must 66
make a move to remove any and all non-nutritious food items
from school campuses. This must start with the removal of
soft drink vending machines, the most egregious offender.

Evodio: While I agree that childhood obesity is a serious threat


to the health of our nation’s children, I do not believe the
removal of soft drink vending machines solves this problem.
The root of the pandemic lies in a combination of uninformed
parenting and limited opportunities for physical activity.

Allison: You are correct that parental involvement is key to the


solution. However, the school system is an influential model
of nutrition and can therefore help guide parents as to what is
appropriate.

Evodio: That assumes the parents of the obese children pay close
attention to their children’s school diet. On the contrary, those
parents are the least likely to be aware of the school meal plans.

In the table below, select the statement that best strengthens


Allison’s argument. Then, select the statement that best
strengthens Evodio’s argument. Make only two selections, one
for each column.

Allison Evodio
Students who attend schools without soft
drink vending machines, in general, consume
far fewer soft drinks outside of school than do
other students.
Studies have shown that students who do
poorly in school by comparison are more likely
to eat non-nutritious foods.
Childhood obesity rates have remained
constant over the past 5 years despite a
concerted effort to introduce healthier meal
items to school cafeterias.
Students who ride the bus tend to be more
obese than other students.
Parents often base their meal planning around
their children’s preferences, which have been
shown to be influenced by food available at
school.
The kinds of physical activity typically found
in a school environment are not particularly
effective at weight reduction.

DO NOT DISTRIBUTE
| 159
GMAT IN CLASS MANUAL

Definitions Case Study

A sampling bias occurs in which a sample is collected in a


particular manner as to cause a portion of the targeted
population to be more or less likely to be selected than others.
66 In such a case, the bias produces what is called a biased
sample: a non-random cross section of the targeted population
in which not all individuals were equally likely to have been
chosen. The presence of a sample bias can adversely affect the
study for which the sample was gathered. The bias often
causes the results to indicate a phenomenon that may actually
not be present. The following lists several common kinds of
sample biases.

Self-selection: This bias occurs when the group of people under


study has control over whether to take part in the study. The
willingness of the participants to participate may align with the
characteristics of the study itself. (For example, a voluntary
customer satisfaction survey may only draw responses from
those participants with strong opinions.)

Pre-screening: This bias occurs when participants from a


specific group are asked to volunteer for the study. The
participants may be incidentally drawn from groups that may
have a certain interest in the study. (For example, a study
designed to test the effects of health food on overall health may
only advertise at a fitness center.)

Exclusion: This bias occurs when a portion of the targeted


population is not included in the sample. (For example, a
political internet survey will exclude those who cannot gain
regular access to the internet, whether because of economic or
regional limitations.)

Healthy User: This bias occurs when the method of sampling


selects participants who are more likely to be healthy than
the average population. (For example, a study using manual
laborers would include participants who are more likely to be in
good health.)

DO NOT DISTRIBUTE
160 |
Lesson 6

Definitions Case Study

Survey Professionals conducted a survey on college students


about their interests in Zapp Cola products. The intention of
the survey was to gauge demand on targeted college
campuses so that Zapp Cola could determine how many 66
vending machines to field. The survey offered several samples
of different Zapp Cola products and then asked the students to
rate each product’s quality of taste on a scale of 1 to 5. The
students were then asked what prices they felt were fair to pay
for each product. In order to generate interest in the survey,
Survey Professionals offered a free bottle of any Zapp Cola
product upon completion of the survey. The advertisements
for the survey were only posted in the various cafeterias and
eateries on and surrounding the targeted college campuses.

1. Which of the following actions, if taken by Survey


Professionals, would have most effectively
strengthened the results of the survey by reducing
the bias present in the survey?
Instead of posting in the cafeterias on the campuses,
Survey Professionals should have posted the
advertisements for the survey only in the eateries
surrounding the campuses.
Survey Professionals should have offered an
alternative incentive in place of the choice
of a bottle of a Zapp Cola Product.
The survey given should not have asked how
much the participants were willing to pay
for the Zapp Cola products.
The advertisements for the survey should
have encouraged anyone from the general
population to participate in the survey
rather than just college students.
The scale used in the survey should have
been changed so that the participants
could write in their own impressions of
each product.

DO NOT DISTRIBUTE
| 161
GMAT IN CLASS MANUAL

Plugging In for Two-Part Analysis


For some Two-Part Analysis questions, you’ll be able to plug in!
1. A jar contains r red marbles and 2 times as many black marbles
as red marbles. Two marbles are drawn from the jar at random
66 without replacement.

In the table below, select an expression that represents the


probability of choosing two red marbles from the jar. Then, select
an expression that represents the probability of choosing at least
one red marble from the jar. Make only two selections, one for each
column.

Probability Probability of
of two red at least one red
marbles marble

How can you identify this


r
problem as a plug in? 3r − 1

r −1
3r − 1

r −1
9r − 3

3r − 1
9r − 3

5r − 1
9r − 3

7r − 1
9r − 3

DO NOT DISTRIBUTE
162 |
Lesson 6

2. A machine completes x percent of a certain job working at a


constant rate of 2 parts per hour and the remainder of the job
working at a constant rate of 8 parts per hour.

In the table below, select one expression that represents, in


terms of x, the average (arithmetic mean) number of parts per 66
hour made by the machine for the entire job. Then, select an
expression that represents the total amount of time required by
the machine, working as described above, to complete a job
consisting of 80 parts. Make only two selections, one for each
column.

Average parts Total time to


per hour complete 80
parts
3 x + 100
800

800
3 x + 100

x+8
2

3 x + 100
10

3 x + 100
8

3x + 4
8

DO NOT DISTRIBUTE
| 163
GMAT IN CLASS MANUAL

Plugging In The Answers for Two-Part Analysis


For some Two-Part Analysis questions, you’ll also be able to Plug In The Answers.
3. The town of Clarkesville currently employs police officers and
firefighters. Clarkesville wishes to increase the size of its police
66 force by 25% while reducing the number of firefighters by
37.5% so that it employs an equal number of police officers and
firefighters.

In the table below, select a value for the number of police officers
currently employed by Clarkesville and a value for the number of
firefighters currently employed by Clarkesville that is consistent
with the information provided above. Make only two selections, one
for each column.

police officers firefighters


9
16
20
24
32
56

DO NOT DISTRIBUTE
164 |
Lesson 6

PITA for Two-Part Analysis

1. Write down the answer choices on your


noteboards. Make two columns, leaving some
space between.
66
2. Decide which column is easier to work with.
3. Label both columns.
4. Starting with an answer in the middle of the
easier column of numbers, work the steps of the
problem.
5. Check for a match between the two columns that
makes a condition in the problem true.

4. Kevin is ten years older than Scott. In six years, Kevin will be
twice as old as Scott.

In the table below, select Kevin’s age in six years. Then, select
Scott’s current age. Make only two selections, one in each
column.

Kevin’s age in Scott’s current


six years age
4
6
10
14
16
20

DO NOT DISTRIBUTE
| 165
GMAT IN CLASS MANUAL

5. For the first 5 hours of a trip, a car drives at a constant rate of 70


miles per hour. For the last h hours of the trip, the car drives at a
constant speed of m miles per hour. The car averages 55 miles
per hour for the entire trip.

66 In the table below, select a value of h, the number of hours for the
second part of the trip and a value for m, the rate of speed in miles
per hour for the second part of the trip, that is consistent with the
information above. Make only two selections, one in each column.

h (hours) m (mph)
3
8
10
25
30
35

DO NOT DISTRIBUTE
166 |
Lesson 7

Lesson 7
77

Critical Reasoning 3

Common Argument Patterns


Many GMAT arguments involve either causality or plans. These arguments have
common assumptions.

1. A harmful algal bloom (HAB) occurred in the Gulf


of Mexico three months after unusually heavy farm
runoff was recorded along the Gulf Coast in July
of 2005. Thus, it is evident that farm runoff causes
harmful algal blooms.

Which of the following statements, if true, most What’s different about scope
seriously weakens the argument above? for this argument?

The harmful algal bloom triggered by the


runoff along the Gulf Coast subsided after
a few months.
The growth of phytoplankton in the ocean
is generally limited by the amount of
nutrients available in ocean waters.
No harmful algal bloom occurred within one
year of the unusually heavy farm runoff
recorded off the coast of California in June
of 2004.
Farm runoff often contains key nutrients on
which algae feed.
Harmful algal blooms are often indicated by red
discolorations on the surface of the ocean.

DO NOT DISTRIBUTE
| 167
GMAT IN CLASS MANUAL

2. In the past five years, there has been a marked decline


in the consumption of soft drinks sweetened with
sugar. During the same time, there has been increasing
publicity about the adverse health effects of weight
gain caused by the consumption of sugar-laden drinks.
Clearly, the public’s awareness of the harmful health
effects of weight gained by consuming sugar-laden
77 drinks caused the decline in the consumption of such
drinks.
How did the correct answer Which of the following statements, if true, most
strengthen this argument? strengthens the argument above?
Many consumers have switched to drinking diet
soft drinks which are not sweetened with sugar.
The average person’s consumption of soft drinks
sweetened with sugar has decreased by 8.2
ounces per day.
Consuming large quantities of sugar-laden
drinks has also been linked to mood swings
throughout the day.
The consumption of packaged cookies
which feature sugar as a key ingredient has
decreased during the time period in question.
The price of sugar-laden soft drinks has
remained stable for the past five years.

DO NOT DISTRIBUTE
168 |
Lesson 7

3. If tourists continue to visit Bridges National Park


at the current rate, the fragile desert environment
that the park seeks to protect will be irreparably
harmed. The park’s Director has put forth a
proposal to charge visitors to the park a fee based
on the number of hours spent in the park. This fee
will induce tourists to limit their time in the park and
thus protect the fragile desert environment. 77
Which of the following is an assumption made in
drawing the conclusion above?
Tourists will limit the amount of time that
they spend in the park by hiking only the
shorter trails in the park.
Most tourists will easily be able to pay the
fee even if they do not reduce the number
of hours that they spend in the park.
The fee will not induce tourists to enter the
park early in the morning before the fee
collection station has opened and leave
only after the fee collection station has
closed for the night.
Protecting the fragile desert environment is an
important issue for most visitors to the park.
Tourists who do not wish to pay the fee can
visit nearby Chasmlands National Park.

DO NOT DISTRIBUTE
| 169
GMAT IN CLASS MANUAL

4. Excessive government regulation, not the current


high price of oil, is responsible for the poor industrial
production in Country A since its new government
came to power. Country B pays the same price for oil,
but while industrial output in Country A has been falling,
Which patterns do you it has been rising in Country B.
recognize in this argument?

77 Which of the following, if true, would cast the most


doubt on the argument above?
Agricultural production is also declining in
Country A.
Whereas Country B must import most minerals,
Country A exports minerals.
In both Country A and Country B, there has been
little credit available through the credit markets.
The industrial goods produced in Country A
are different from those that are produced in
Country B.
Country A’s new government increased regulation of its
industry to promote a cleaner environment.

DO NOT DISTRIBUTE
170 |
Lesson 7

5. The renewed surge in fear over the virulence of the


Ebola virus is irrational and unfounded. While in 2008
only 54 deaths worldwide were directly attributed to
Ebola, the US Centers for Disease Control estimates
that over 30,000 deaths were caused by the influenza
virus in the United States alone in that same year. Yet
no such hysteria has surrounded influenza, despite
the significantly higher number of fatalities. 77
Which of the following, if true, would most
undermine the argument above?
Deaths due to Ebola tend to be concentrated
in remote geographical locations.
Ebola has a shorter incubation period than
many other tropical diseases.
Transmission of the influenza virus among the
US population is aided by the mobility of the
US population.
The average fatality rate for the most common
form of the Ebola virus is significantly higher
than the average fatality rate for common
forms of the influenza virus.
Worldwide, over 30,000 people die from
Yellow Fever each year.

Pattern Assumption Weaken Strengthen

Causality

Planning

Analogy

Sampling

DO NOT DISTRIBUTE
| 171
GMAT IN CLASS MANUAL

Math 6

More Problem Solving Plugging In


For some plugging in questions, you may need to look a little harder to determine
how to plug in!
77

Is it easier to pick a value for 1. A hotel has a total of n rooms and suites. If the number of
n or for something else in this
5 3
problem? rooms is the number of suites, and of the suites have two
4 5
bedrooms, how many two bedroom suites, in terms of n, does

the hotel have?

1
n
4
4
n
15
13
n
20
3
n
4
37
n
20

DO NOT DISTRIBUTE
172 |
Lesson 7

2. Mike and Dave are each assigned to produce x identical widgets.


Based on their respective rates, Dave calculates that it will take
him 5 hours to produce the widgets while Mike calculates that
it will take him 2 hours longer than Dave. Dave agrees to make y
of Mike’s widgets so that they can each complete the task in the
same amount of time. What is the number of widgets, in terms of
y, that Mike was assigned to make?
77
2y
3y
4y
5y
6y

2
3. At a bakery, each small loaf of sourdough bread requires as What’s a good way to
3 organize the information as
much flour to make as each large loaf of sourdough bread. If the you plug in on this problem?

3
store sells as many small loaves of sourdough bread as large
5
loaves, what fraction of all the flour used by the store to make

loaves of sourdough bread was used to make large loaves of

sourdough bread?

1

4
2

5
3

5
2

3
5

7

DO NOT DISTRIBUTE
| 173
GMAT IN CLASS MANUAL

More Data Sufficiency Plugging In


In some cases, you may want to either plug in or list values that satisfy statements
for value data sufficiency questions.

1. What is the sum of the units digit and tens digit of positive
integer x ?
77
(1) x = 5k where integer k ≥ 2

(2) x divided by 20 has a remainder of 5.

2. If x is a two digit prime integer, what is the value of x ?

(1) x is 3 more than a multiple of 4.

(2) When x is divided by 7, the remainder is 2.

DO NOT DISTRIBUTE
174 |
Lesson 7

Number Theory Topics


Many GMAT questions involve number theory topics.

1. If a, b, c, and d are integers such that 1 < a < b < c < d and


the product of a, b, c, and d is 330, what is the value of cb ?
32 77
121
125
243
2,048

Unique Prime Factorization Theorem

Every integer n, n > 1, can be factored uniquely into prime


factors.

2. If m is a multiple of 10 and n is a multiple of 15, is mn a What must be true for one


integer to be a multiple of
multiple of 360 ? another?

(1) m is a multiple of 8.

(2) n is a multiple of 9.

DO NOT DISTRIBUTE
| 175
GMAT IN CLASS MANUAL

3. If x, y, and z are positive integers, and x is a multiple of 3, for


which of the following must z and 18 have a common factor
greater than 1 ?
x y z
I. + =
9 6 18

x y z
77 II. + =
6 9 18

x y z
III. + =
6 5 30

I only
II only
I and II only
I and III only
I, II, and III

What number theory rule is 4. If m is the product of positive integers a and b, does there exist
this question testing? a positive integer k, which is a multiple of both a and b, such that
k < m ?

(1) The only factor that a and b have in common is 1.

(2) Integer a is prime.

DO NOT DISTRIBUTE
176 |
Lesson 7

5. If x is an integer, is 2x a factor of 12! ?

(1) x is the sum of two distinct single-digit prime numbers.

(2) 0 < x < 11

77

6. If Set Q contains only positive integers, is the average


(arithmetic mean) of the numbers in Set Q equal to the median
of the set?

(1) The numbers in Set Q are consecutive and odd.

(2) Set Q contains an odd number of numbers.

DO NOT DISTRIBUTE
| 177
GMAT IN CLASS MANUAL

Homework Review
Use this chart to note any questions you have from the reading or examples in the homework.

Page # Question # What que stion do you have ?

77

DO NOT DISTRIBUTE
178 |
Lesson 7

Practice
1. What is the remainder when integer n is divided 8. If set N contains only consecutive positive
by 10 ? integers, what is the sum of the numbers in
set N ?
(1) When n is divided by 110 the remainder is
(1) Nineteen times the sum of the first number in
75.
the set and the last number in the set is 1,729.
(2) When n is divided by 100 the remainder is
(2) There are 38 numbers in the set.
25.
9. At the Massachusetts Academy this year, the
2. If 6 is a factor of a and 21 is a factor of b, is ab a
boys are all 2, 3, 5, or 7 years of age. If the
multiple of 70 ?
product of the ages of the boys in a given class
(1) a
is a multiple of 4. is 10,500, then how many 5-year-olds are in
(2) b is a multiple of 15. that class?
0
3. If n is an integer greater than 0, what is the 3
remainder when 912n+3 is divided by 10 ? 5
0 125
1 2,100
2
7 10. If x is an integer greater than zero but less than
integer n, is x a factor of n ?
9
(1) n is divisible by all positive integers less
4. If n is an integer greater than 5.3, then n! must than 10.
be divisible by which of the following numbers? (2) x is not a multiple of a prime number.
7
11. The product of all prime numbers less than 29 is
11 approximately equal to which of the following?
12
13 2 × 104
14 2 × 106
2 × 108
11! 2 × 109
5. The fraction is equivalent to which of the
following? 77 2 × 1010
27×33×52
29×34×52 12. If a is a positive integer, is a + b an even
integer?
28×34×52
28×34×52×72 (1) xaxb = 1
28×34×52×11 (2) x ≠ 1
15!
6. Is a prime? 13. If is an integer, what is the greatest possible
3m
(1) x ! = a value of m ?
(2) x > 2
4
7. If x and n are positive integers, is n! + x divisible 5
by x ? 6
7
(1) n > x
8
(2) n is not a prime number.
14. If k is a multiple of 11, is kl a multiple of 154 ?
(1) Every factor of 42 is also a factor of l.
(2) k is divisible by 21.

DO NOT DISTRIBUTE
| 179
GMAT IN CLASS MANUAL

Answers and Explanations


1. D Plug in. To find a number that satisfies statement
so, the answer to the question is ‘yes’. Statement
(1), remember that you can add the divisor and (1) is not sufficient and the possible answers are
the remainder. So, n  =  185 satisfies the state- B, C, or E. You can follow a similar procedure for
ment. In this case, the answer to the question is statement (2). You could make a = 6 and b = 21 ×
5, the remainder when 185 is divided by 10. To 15. In this case, the answer to the question is ‘yes’
find the next number that satisfies statement (1), because all the required prime factors are present.
start with a multiple of the divisor, then add the Note that it won’t be possible to get a ‘no’ because
remainder. So, for example n = 295 = 220 + 75 any additional values of b that satisfy statement
satisfies the statement. In this case, the answer to (2) will always include 5 and 7 among its prime
the question is also 5. You can probably see the factors. So, the second statement is sufficient.
pattern that is emerging at this point. Because 3. E When something looks like an insane amount
any multiple of 110 has a units digit of 0, you’ll of work, start looking for a shortcut. In this
always get a number for n that ends in 5 once you case, the shortcut is a pattern: 91 = 9; 92 = 81;
add 75. Since the remainder of an integer divided 93 = 729; 94 = 729 × 9 = 6,561. Multiply that by
by 10 is always the units digit of the integer, the another 9? You’ll get a number ending in 9. Then,
answer to the question is always 5. So, statement a number ending in 1. And so forth and so on.
(1) is sufficient and the possible answers are A or The bottom line is that whenever 9 is raised to an
D. You can follow a similar procedure to test out odd power, the units digit is 9. When it’s raised
statement (2). Numbers that satisfy statement (2) to an even power, the units digit is 1. When you
are always the sum of a multiple of 100 and 25. divide a number by 10, its remainder will always
These numbers also always have a units digit of be its units digit. Now, you just need to know if
5. So, the answer to the question is again always you are raising 9 to an even or an odd power. The
5, which makes statement (2) sufficient, as well. exponent for 9 is the expression 12n + 3. Note
2. B This question is all about factoring. We need that the first part of the expression (12n) always
to determine whether 70 is a factor of ab, and produces an even number because the product
the easiest way to do that is to break 70 down of an even integer and an even integer or an odd
into its prime factors. 7 × 5 × 2 = 70. So if ab is integer is even. When the odd integer 3 is added to
divisible by 7, 5, and 2, then it’s divisible by an even integer, the sum is odd. So, the exponent
70. The question itself lets us know that ab is for 9 is always odd no matter what the value of n
divisible by 2 (since 6 is a factor of a) and by 7 and the result will be an integer that has a units
(since 21 is a factor of b), so all we need is proof digit of 9. The remainder when that integer is
that it is divisible by 5. At this point, it’s worth- divided by 10 is 9.
while to plug in. For statement (1), make a = 12 4. C If n is greater than 5.3, then the smallest n! can be
and b  =  21. So, ab  =  12  × 21, which does not is 6!. Since 6! = 6 × 5 × 4 × 3 × 2 × 1, it is defi-
include 5 as one of its prime factors. The answer nitely divisible by 12, because any n! greater than
to the question is ‘no’ in this case. However, if 6 will include both a 6 and a 2 in its expansion,
a = 60 and b = 21, then ab includes all the required thus making it a multiple of 12. Also, n! does not
primes (2, 5, and 7) among its prime factors and have to be divisible by anything greater than 6, so
7, 11, and 13 are eliminated as are any multiples
of those numbers, such as 14.

DO NOT DISTRIBUTE
180 |
Lesson 7

5. C Write out the factorial and then cancel everything Alternatively, Plug In values for x and n and you
that you can. Since 11! includes both a 7 and an will discover these facts. Just keep the numbers
11 we can cancel those numbers with the 77 in small and easy to work with. For example, when
the denominator leaving us with 10 × 9 × 8 × 6 testing Statement (1), start with something like
× 5 × 4 × 3 × 2 and making answers (D) and (E) n = 3 and x = 2. In this case, the question be-
incorrect. Next, a glance at the answers shows comes is “3! + 2 divisible by 2”? and the answer
that we don’t need to calculate the number, but is ‘yes’. Try that with a few other values such
rather just put it in an exponential form, so the as n = 5 and x = 3 and you will quickly find
next step is to express the remaining numbers as that Statement (1) is sufficient. Remember that
products of 2’s, 3’s, and 5’s. So, that’s (2 × 5) × (3 × part of the point of plugging in is that it helps
3) × (2 × 2 × 2) × (2 × 3) × 5 × (2 × 2) × 3 × 2. you to determine which rules are tested by the
Next, remember that you add the exponents when question. The possible answers are (A) or (D).
multiplying expressions with like bases. Note Statement (2) tells us nothing about x nor its
that there are eight 2’s, so the correct expression relationship to n. Stating that n is NOT prime
should start with 28. Of the answers that remain, means it could be a vast number of values. Thus
only (C) works. Statement (2) is not sufficient.
8. C Statement (1) just lets you know that the sum of
6. C Statement (1) does not tell us anything about the
the first and last numbers in the set is 91, but that
value of x or a. We can’t say whether it’s prime
allows the first number to be 1 and the last to be
or not, because if x is 2, then a is prime, but if
90 or the first to be 45 and the last to be 46, so
x is anything other than 2, then a is not prime.
it’s not sufficient. The possible answers are B, C,
Statement (1) alone is insufficient. The possible
or E. Statement (2) tells you how many numbers
answers are BCE. Statement (2) says nothing
are in the set, but gives you no notion of the
about a, thus Statement (2) is insufficient.
values of any of those numbers. Together, we know
Eliminate (B) and keep CE. Taken together, we
that there are 38 consecutive numbers, and that
know that x is greater than 2, and so a is the
the sum of the smallest of the numbers and the
product of at least 3 integers (3!). Since a prime
largest is 91. This information is sufficient,
number has only 2 factors, a cannot be prime,
since we now can determine exactly which two
and the correct answer is (C).
numbers are the smallest and largest in the set.
7. A This one is tough. To understand the relevance
9. B The phrase “product of the ages . . . is 10,500”
of Statement (1), you have to recognize the
tells us that we need to factor 10,500 to see what
following:
its prime factors are. Remember that the best way
• A factorial is divisible by all positive to find prime factors is to use a factor tree. If we
integers less than or equal to the integer you break 10,500 down to its prime factorization, we
are taking a factorial of. For example, 5! is get 2 × 2 × 3 × 5 × 5 × 7. There are three five-
divisible by all positive integers less than or year-olds.
equal to 5.
• If b is a multiple of y, then if you add y to
b, the result will still be divisible by y. For
example, 12 is divisible by 3. If you add
12 + 3, the result is a number divisible by 3.

DO NOT DISTRIBUTE
| 181
GMAT IN CLASS MANUAL

10. B Statement (1) tells us that n is a multiple of all 0. Since xaxb = xa + b = 1, we know that a + b must
the integers 1–9, inclusive. The first number be equal to 0, an even interger.
that satisfies Statement (1) is the least common
multiple of the integers from 1–9. To find this 13. C In order for the result when 15! is divided by 3m
number, start by breaking each of the integers 1–9 to be an integer, then for every 3 on the bottom
into its prime factors. Of course, 2, 3, 5, and 7 of the fraction, there must be a 3 on the top of the
are already prime. Then, 4 = 22; 6 = 2 × 3; 8 = 23 fraction. In the expansion of 15!, the following
and 9 = 32. So, the least common multiple must numbers are multiples of 3: 3, 6, 9, 12, and 15.
include three 2’s and two 3’s as well as a 5 and Next, write out the prime factorizations of these
7 among its prime factors. Therefore, the least numbers: 3 = 31; 6 = 2 × 3; 9 = 3 × 3; 12 = 22 × 3;
common multiple of the integers from 1–9 is 15 = 3 × 5. There are a total of six 3’s in the prime
23 × 32 × 5 × 7 = 2,520. Now, it’s time to think factorizations of these numbers.
about numbers for x. If x = 5, then the answer to
14. A Start by assessing the question. If kl is to be a
the question is ‘yes’. However, if x = 11, which is
multiple of 154, then kl divided by 154 must be
also less than 2,520, then the answer to the ques-
an integer. That means that every prime factor
tion is ‘no’ since 11 is not a factor of 2,520. The
in the prime factorization of 154 = 2 × 7 × 11
possible answers are B, C, or E. Statement (2) tells
must be a prime factor of either k or l. Since k is
us that x is not a multiple of a prime number, but
a multiple of 11, the statements need to provide
all integers greater than 0 except for 1 are mul-
a way to determine if both 2 and 7 are factors of
tiples of prime numbers, so what Statement (2)
either k or l. Since the question is phrased as a
really tells us is that x is 1. And 1 is a factor of all
‘yes-no’ question, plug in. For Statement (1), the
integers, so x must be a factor of n.
largest factor of 42 is 42, so l must be at least 42.
1 1. C First, write out all primes less than 29 (2, 3, 5, 7, Pick l = 42 and k = 11 for the first plug in. For
11, 13, 17, 19, 23), then begin grouping them as these numbers, the answer to the question is ‘yes’
numbers the products of which are multiples of because both 2 and 7 are prime factors of 42. It
10 (because the answers are expressed as won’t be possible to get an answer of ‘no’ to the
powers of 10). You get (2 × 5)(3 × 7)(11)(13)(17) question because only multiples of 42 will satisfy
(19)(23) which could be expressed as 10 × 21 × 11 the first statement’s restrictions on l. So, write
× 13 × 17 × 19 × 23. Then round these products down AD. For Statement (2), let k = 11 × 21 and
as close to multiples of 10 as possible, which gives l = 2. In this case, the answer to the question is
us 10 × 20 × 10 × 10 × 20 × 20 × 20. Each 20 ‘yes’ because all the required prime factors (2, 7,
can be expressed as 2 × 10, yielding 10 × 2 × 10 and 11) are present. However, if k = 11 × 21 and
× 10 × 10 × 2 × 10 × 2 × 10 × 2 × 10. Expressing l = 3, then the answer to the question is ‘no’. The
the products as exponents yields 24 × 107. Finally, second statement is insufficient and the answer
24 = 16, which is approximately 20, or 2 × 10. So to the problem is A.
we have 2 × 10 × 107, or 2 × 108.

12. C Statement (1) doesn’t give enough information


to figure out whether x = 1 or whether a + b = 0.
Statement (1) is thus insufficient and the possible
answers are B, C, or E. Statement (2) alone tells
nothing about a or b, so it’s not sufficient. Elimi-
nate (B), and keep CE. Considered together, we
know that x is not 1, and thus a + b must equal

DO NOT DISTRIBUTE
182 |
Lesson 8

Lesson 8
88

Critical Reasoning 4

Identify the Reasoning Questions


Identify the reasoning questions ask about the method used by the author to reach
the conclusion or what role a phrase plays in an argument.

Step 1: Identify the question.


Look for questions that ask what technique, strategy, or method the author
employs or what role a bolded phrase plays. Identify-the-Reasoning questions
typically ask:

• The argument uses which one of the following methods of


reasoning?
• The bolded phrases play which of the following roles in the
argument above?

Step 2: Work the argument.


Break the argument into its parts. Look for:

• Conclusion
• Premise
Step 3: Predict what the answer should do.
The correct answer describes the structure of the argument.

Step 4: Use POE to find the answer.


Eliminate answers that:

• Are only partial matches.


• Describe something the argument did not do.

DO NOT DISTRIBUTE
| 183
GMAT IN CLASS MANUAL

1. Poppy: High taxes have a chilling effect on the


economy. When individuals and corporations are taxed,
they have less money to spend. Demand for products
and services is reduced, causing unemployment to
increase. Taxes must be lowered.
Lilly: But you must also consider that taxes generate
funds for the government. If taxes are lowered, the
government will be forced to borrow more money,
88 thus reducing the amount of credit available. Unable to
borrow money easily, businesses and individuals will be
forced to limit their purchases.
Lilly objects to Poppy’s argument by
claiming that Poppy has exaggerated the
adverse effects of high taxes
indicating that Poppy has based his argument on
insufficient evidence about the effects of taxes
on the economy
noting that Poppy has failed to adequately define
the term taxes
demonstrating that the danger of reducing
taxes is far more severe than the threat of
maintaining them at current levels
suggesting that the economic benefits of easy
credit outweigh the danger of unemployment

DO NOT DISTRIBUTE
184 |
Lesson 8

2. Although computer manufacturer X has


experienced decreasing sales in the last quarter,
stockholders should not sell their shares of the
company. The stock price of a financially troubled
company can rise dramatically once those problems
are solved. Last year, the stock price of steel
manufacturer Y rebounded after the company
reduced its accounts receivable backlog.
The bolded phrases play which of the following 88
roles in the argument above?
The first phrase contains the author’s
conclusion and the second phrase contains
unrelated information.
The first phrase states a position and the
second phrase provides evidence to
undermine that position.
The first phrase states a premise on which
the conclusion is based and the second
phrase provides the conclusion.
The first phrase states the conclusion and
the second phrase supports the conclusion
with an analogy.
The first phrase offers advice and the
second phrase draws a contrast between
two companies.

DO NOT DISTRIBUTE
| 185
GMAT IN CLASS MANUAL

Resolve/Explain Questions
Some questions ask you to resolve an apparent paradox or explain a discrepancy.

Step 1: Identify the question.


Look for key words like resolve and explain.

Step 2: Work the argument.


88 Read the argument and state the conflict. Ask “Why X but also Y?”

Step 3: Predict what the answer should do.


The correct answer provides a resolution that allows both statements to be true.

Step 4: Use POE to find the answer.


Eliminate answers that:

• Are out of scope.


• Make the conflict worse.
• Address only one side of the conflict.

DO NOT DISTRIBUTE
186 |
Lesson 8

1. Although the mathematical validity of the laws of


probability is indisputable, most people do not trust
the dictates of these laws. Even among people
who claim to have studied probability theory,
for instance, a majority express a greater fear of
flying on commercial airlines than of driving an
automobile on our nation’s highways, despite the
fact that the probability that one would suffer an
automobile-related death or injury by choosing to
drive is more than twenty times the probability of
88
an airline-related death or injury if one chooses to
fly.

Which one of the following, if true, provides the


best explanation for people’s mistrust of the laws
of probability in the case described above?
A complete understanding of the laws of
probability requires a thorough knowledge
of advanced statistical analysis techniques.
People who studied probability theory in an academic
environment may be ill equipped to apply that
knowledge to real-world situations.
People tend to suspend their belief in
probability when they feel somewhat in
control of their own fate.
The probability of automobile-related injury
or death is not significant enough to
dissuade many people from driving.
The greatest risk to the individual driver in
terms of automobile-related injuries or
fatalities are the actions of the other drivers
on the road.

DO NOT DISTRIBUTE
| 187
GMAT IN CLASS MANUAL

Minor Question Types


A few other question types occasionally appear on the GMAT. To answer these
questions, you’ll need to break down the arguments and understand their structures.

Evaluate-the-Argument Questions
88 To evaluate an argument, you need information that allows you to determine the truth
of the assumption.

Step 1: Read and identify the question.


Evaluate-the-argument questions often use words such as evaluate or assess. Also, the
answer choices are frequently questions.

Step 2: Work the argument.


Break the argument into its parts. Look for:

• Conclusion
• Premise
• Gap, Assumption, or Pattern

Step 3: Predict what the answer should do.


The correct answer will give you a way to evaluate the truth of the assumption.

Step 4: Use POE to find the answer.


Eliminate answers that:

• Are out of scope.

DO NOT DISTRIBUTE
188 |
Lesson 8

1. Ergonomically designed computer keyboards tend


to lose their “play”—the responsiveness of the
keys—more quickly than do traditional keyboards.
A software designer has suggested that it is in fact
the curvature of the key rows and not increased
typing speed that is to blame. Due to the bent
shape of the board, it is more difficult for the
average user to clean between the keys, resulting in
a gradual deadening of the spring mechanisms.
88
The answer to which of the following questions
will most likely yield significant information that
would help to evaluate the software designer’s
hypothesis?
Do traditional keyboards and ergonomically
designed keyboards utilize the same plastics?
Does sprinkling a keyboard with dust impede the
spring action beneath the keys?
Does a keyboard with deadened play make typing
more difficult?
Do computer manufacturers receive more
complaints about ergonomically designed
keyboards than about traditional keyboards?
Are software designers more likely than other
users to utilize their keyboards when working?

DO NOT DISTRIBUTE
| 189
GMAT IN CLASS MANUAL

Flaw Questions
Flaw questions are similar to weaken questions.

Step 1: Read and identify the question.


Look for the words flaw or vulnerable to criticism.

Step 2: Work the argument.


88 Break the argument into its parts. Look for:

• Conclusion
• Premise
• Gap, Assumption, or Pattern

Step 3: Predict what the answer should do.


The answer needs to describe a problem with the structure of the argument.

Step 4: Use POE to find the answer.


Eliminate answers that:

• Cannot be matched to the facts of the argument.


• Contain irrelevant information.

1. Assembly Line Sandwiches uses only Lean Cuts brand


of deli meats, which are lower in saturated fat than any
other deli meats, to make all its sandwiches. Since diets
high in saturated fat are linked to heart disease, it can
be concluded that the sandwiches served by Assembly
Line are nutritious.

Which of the following is a flaw in the argument above?


No information is offered about the unsaturated
fat content of Lean Cuts deli meats.
Low in saturated fat is not the same thing as nutritious.
Different people have different nutritional needs.
Many customers add cheese and mayonnaise to
their sandwich which increases the fat content.
Toastie’s Sandwiches, Assembly Line’s biggest
competitor, has recently started using Lean
Cuts deli meats to make its sandwiches.

DO NOT DISTRIBUTE
190 |
Lesson 8

Sentence correction 3

Redundancy
Don’t be repetitive. Don’t be redundant. Don’t be repetitive. The reason for this
is because you shouldn’t say the same thing more than once.

1.
If the depletion of the ozone in the upper portion 88
of the Earth’s atmosphere were to continue at its
present rate, by the year 2000 the hole in the ozone
layer would be at least one thousand miles wide or
wider.
If the depletion of the ozone in the upper
portion of the Earth’s atmosphere were
to continue at its present rate, by the year
2000 the hole in the ozone layer would be
at least one thousand miles wide or wider.
Were the depletion of ozone in the upper
portion of the Earth’s atmosphere to
continue at its present rate, by the year
2000 the hole in the ozone layer would be
at least one thousand miles wide.
Was the depletion of ozone in the upper
portion of the Earth’s atmosphere to
continue at its present rate, by the year
2000 the hole in the ozone layer would be
at least one thousand miles wide or wider.
If the depletion of ozone in the upper portion
of the Earth’s atmosphere were continuing
at its present rate, by the year 2000 the
hole in the ozone layer would be at least
one thousand miles wide.
Should the depletion of ozone in the upper
portion of the Earth’s atmosphere continue
at its present rate, by the year 2000 the
hole in the ozone layer would be at least
one thousand miles wide or wider.

DO NOT DISTRIBUTE
| 191
GMAT IN CLASS MANUAL

Clauses and Connectors


When conjunctions and punctuation change in the answer choices, make sure clauses
are joined correctly.

2. By the late 1940s, ENIAC, the first electronic digital


computer was able to compute a ballistics trajectory
88 in about 30 seconds, and it was much less time than
the 20 hours required to perform the same calculations
with a desk calculator.
computer was able to compute a ballistics
trajectory in about 30 seconds, and it was
computer, which was able to compute a
ballistics trajectory in about 30 seconds, was
computer, was able to compute a ballistics
trajectory in about 30 seconds,
computer was able to compute a ballistics
trajectory in about 30 seconds, being
computer, and was able to compute a ballistics
trajectory in about 30 seconds, and was

Grammar and Meaning


GMAC often changes the intended meaning of a sentence by misplacing modifiers or
using ambiguous pronouns. You can eliminate such answers either because they break
a grammar rule or change the meaning.

3. Some marmot populations of Central Asia are


infected more frequently by plague than rats, which
are established in medical histories as the primary
mammalian vector of the Black Death.
are infected more frequently by plague than
are more frequently infectious with plague than
are infectious with plague more frequently than are
are infected more frequently by plague than are
are frequently infected with more plague than

DO NOT DISTRIBUTE
192 |
Lesson 8

4.
Some artifacts that were defaced and carried away
undamaged in the looting of the museum last April
were designated to be part of a traveling exhibit
next year.
Some artifacts that were defaced and
carried away undamaged in the looting of
the museum last April were
Some artifacts that were defaced or carried
away undamaged in the looting of the 88
museum last April had been
Some artifacts that the looting of the
museum last April defaced and carried
away undamaged have been
Last April the looting of the museum
defaced or carried away undamaged some
artifacts that have been
Last April some of the artifacts that were
defaced or carried away undamaged in the
looting of the museum had been

5.
The bolas spider, in its sticky globules, hanging
from silken threads dangled from its front legs, The order of the phrases
affects the meaning of the
stores a pheromone-like secretion that lures sentence.
male moths, which become stuck on the spider’s
globules.
The bolas spider, in its sticky globules,
hanging from silken threads dangled from
its front legs,
Hanging from silken threads dangled from
its front legs, the bolas spider, in its sticky
globules,
In its sticky globules, which it hangs from
silken threads dangled from its front legs,
the bolas spider
In its sticky globules, hanging from silken
threads dangled from its front legs, the
bolas spider
The bolas spider, in its sticky globules, hangs
from silken threads dangled from its front
legs and

DO NOT DISTRIBUTE
| 193
GMAT IN CLASS MANUAL

6. In the early Thirteenth Century, resulting from the


Mongol conquest of the Chinese capital Zhongdu,
which later became known as Beijing, and numerous
other military victories, Genghis Khan added the
northern half of China to his rapidly expanding empire.
resulting from the Mongol conquest of the Chinese
capital Zhongdu, which later became known as
Beijing, and numerous other military victories,
88 the Mongol conquering of the Chinese capital which
later became known as Beijing, Zhongdu, and
numerous other military victories resulted, and
because of the result of the Mongol conquest of the
Chinese capital Zhongdu, which later became known
as Beijing, and numerous other military victories,
as a result of the Mongol conquest of the Chinese
capital Zhongdu, which later became known as
Beijing, and numerous other military victories,
Zhongdu was conquered, later became known as
Beijing, and after numerous other military victories,

DO NOT DISTRIBUTE
194 |
Lesson 8

7.
The site of funeral services for three Presidents
of the United States, eighty-three years after the
foundation stone was laid in the presence of
President Theodore Roosevelt and a crowd of ten
thousand onlookers in 1990, the completion of the
Washington National Cathedral took place.
The site of funeral services for three
Presidents of the United States, eighty-
three years after the foundation stone was 88
laid in the presence of President Theodore
Roosevelt and a crowd of ten thousand
onlookers in 1990, the completion of the
Washington National Cathedral took place.
The site of funeral services for three
Presidents of the United States, the
completion of the Washington National
Cathedral, eighty-three years after the
foundation stone was laid in the presence
of President Theodore Roosevelt and a
crowd of ten thousand onlookers in 1990.
Completed eighty-three years after the
foundation stone was laid in the presence
of President Theodore Roosevelt and a
crowd of ten thousand onlookers in 1990,
the Washington National Cathedral was the
site of funeral services for three Presidents
of the United States.
Completed in 1990, eighty-three years
after the foundation stone was laid in the
presence of President Theodore Roosevelt
and a crowd of ten thousand onlookers,
the Washington National Cathedral was the
site of funeral services for three Presidents
of the United States.
Completed in 1990, eighty-three years
after the foundation stone was laid in the
presence of President Theodore Roosevelt
and a crowd of ten thousand onlookers in
1990, funeral services for three Presidents
of the United States were held at the
Washington National Cathedral.

DO NOT DISTRIBUTE
| 195
GMAT IN CLASS MANUAL

Math 7

Simultaneous Equations
Simultaneous equations are usually tested in data sufficiency format. Remember to
use the Pieces of the Puzzle approach.

88 1. What is the value of x ?

(1) 5x + 10y = 65

(2) x + y = 8

Translate the words into 2. Jinhee spends $65 on hats. If she buys only derbies and fedoras,
equations. Be sure to write how many derbies does she buy?
the equations down.

(1) Jinhee buys derbies for $5 each and fedoras for $10 each.

(2) Jinhee buys a total of 8 hats.

3. If 5x + 10y = 65, what is the value of x ?


What do you know?
What do you need?
(1) x + y = 8

(2) x is an integer.

4. Elizabeth spends a total of 65¢ on postage stamps. She buys only


stamps with face values of 5¢ and 10¢. How many 5¢ stamps
does she buy?

(1) She spends 50¢ on envelopes.

(2) She buys a total of 8 stamps.

DO NOT DISTRIBUTE
196 |
Lesson 8

Simultaneous Equations Nuances


Be wary of these traps when dealing with simultaneous equations.

5. What is the value of x ?

(1) 5x + 10y = 65

88
(2) 130 – 20y = 10x

6. If Jinhee buys only derbies and fedoras, what is the cost of


a derby?

(1) Jinhee purchases three derbies and five fedoras for a total
cost of $65.

(2) Jinhee spends $130 to purchase ten fedoras and six derbies.

7. A certain test has two types of questions. Multiple choice


questions are worth 2 points each and fill in questions are What must be true of the
worth 5 points each. How many multiple choice questions did variables for this question?
Vikram answer correctly?

(1) Vikram scored a total of 28 points on the test.

(2) Vikram answered more than two of the five point


questions correctly.

DO NOT DISTRIBUTE
| 197
GMAT IN CLASS MANUAL

8. A theater charges $13 for an adult ticket and $5 for a child’s


ticket. If Ruchi took a group of her friends and their children to a
show at this theater, how many children were in the group?

(1) Ruchi spends a total of $72 dollars on the tickets.

(2) Ruchi buys a total of 8 tickets.


88

What’s the trap answer? 9. What is the average (arithmetic mean) of 8a and 4b ?

(1) a + b = 5

(2) 2a + b = 11

DO NOT DISTRIBUTE
198 |
Lesson 8

Sets
1. The set of prime factors of integer y has one more element What’s the definition of
than the set of prime factors of 84. What is the least possible a set?
value of y ?
168
210
252
420
88
462

Groups
For group problems, decide whether to use the group equation or the group grid.

1. A group of fourth-graders and fifth-graders is going on a field


trip to the zoo. Only 20% of the students remembered to
bring written permission from their parents. If 40% of the
students are fourth-graders and 25% of the fourth-graders
remembered to bring written permission, what percentage of
the students are fifth-graders who forgot to bring
written permission?
10%
30%
50%
60%
80%

DO NOT DISTRIBUTE
| 199
GMAT IN CLASS MANUAL

2. Of the 600 residents of Clermontville, 35 percent watch the


television show Island Survival, 40 percent watch Lovelost
Lawyers,and 50 percent watch Medical Emergency. If all
residents watch at least one of these three shows and 18 percent
watch exactly 2 of these shows, then how many Clermontville
residents watch all three of these shows?
240
150
88 108
42
21

Venn Diagrams
Venn diagrams are another way to represent a group problem.

Arches Yosemite

x y x + 10

1. The pictograph above shows a representation of how many of


the 200 members of a hiking club have visited Arches National
Park, Yosemite National Park, or both. If every member has visited
at least one of the two parks, what is the value of x ?

(1) 150 members have visited Yosemite National Park.

(2) The number of members who have visited both parks is


50% greater than the number of members who have visited
only Yosemite.

DO NOT DISTRIBUTE
200 |
Lesson 8

Permutations and Combinations


Some GMAT problems deal with calculating the number or ways to choose or
arrange a group of objects. GMAC doesn’t use the math terms so don’t worry
about them.

Choosing from Different Sources


88
In these problems, you are choosing a number of items to fill specific spots, and
each spot is filled from a different source.

1. Each night before he goes to bed, Jordan likes to pick out


an outfit to wear the next day. He has 12 different shirts, 10
different pairs of jeans, and 8 pairs of sneakers. If an outfit
consists of 1 shirt, 1 pair of jeans, and 1 pair of sneakers, how
many different outfits does Jordan have?
30
90
240
480
960

Same Source—Order Matters


For some problems from which you choose items from the same source, the order
matters.
Which words indicate that the
2. Five people are running in a race. The first one to finish wins order matters?
a gold medal, the second wins a silver medal, and the third
wins a bronze medal. How many different arrangements of
medal winners, in order from first to third, are possible?
5
10
60
120
125

× × =
choices for gold choicesleft for silver choicesleft for bronze different arrangements

DO NOT DISTRIBUTE
| 201
GMAT IN CLASS MANUAL

Same Source—Order Doesn’t Matter


For other problems from which you choose items from the same source, the order
doesn’t matter.

Which words indicate that


3. Five people are running in a race. The first three to finish win gift
the order doesn’t matter?
certificates. How many different groups of people could win the
gift certificates?
88 5
10
60
120
125

Order Does Matter Order Doesn’t Matter


• (a, b, c) ≠ (b, a, c) • (a, b, c) = (b, a, c)
• different arrangements • different groups
• Don’t divide • Do divide

There are two key questions to ask when doing one of these problems:
1) Is there a single source, or are there multiple
sources?
2) If there is a single source, do you want to count
distinct groups or different arrangements/orders?

DO NOT DISTRIBUTE
202 |
Lesson 8

Combining Concepts
Some problems combine the concepts of same source and different source.

4. Kris is purchasing gear to ride his bike this winter. He wants to


buy 2 different pairs of gloves, 1 parka, 2 different hats, and 3
different pairs of boots. If the catalog from which he will order
offers 5 types of gloves, 3 different parkas, 4 hats, and 6 pairs
of boots, how many different orders could he place? 88
360
720
3,600
7,200
36,000

5. Ben needs to form a committee of 3 from a group of 8


engineers to study design improvements for a product. If two
of the engineers are too inexperienced to serve together on the
committee, how many different committees can Ben form?
20
30
50
56
336

DO NOT DISTRIBUTE
| 203
GMAT IN CLASS MANUAL

Homework Review
Use this chart to note any questions you have from the reading or examples in the homework.

Page # Question # What que stion do you have ?

88

DO NOT DISTRIBUTE
204 |
Lesson 8

Practice
Mixed Topics
1. Of the 150 students at Hunter High, 45 are in 4. On March 15th, the population of the city of
the glee club and 72 are in the key club. If the Madrigoon was 0.15 billion people. On May 1st,
number who are in neither group is twice the an earthquake struck Madrigoon and destroyed
number who are in both groups, how many are 0.01% of the 30 million homes. If an equal
in both groups? number of people lived in each home and 50%
of the people whose homes were destroyed
22 moved to another city, then how many people
33 moved to another city?
44
55 7.50 × 104
66 1.50 × 104
7.50 × 103
2. How many integers between 0 and 1570 have a 0.15 × 104
prime tens digit and a prime units digit? 0.15 × 103
295
5. An ice cube is floating in a glass of water with
252
1 1
236 between and of its mass above water and
96 6 7
the rest submerged below the water’s surface.
76
The ratio of the part of the mass above water to
3. In March, Kurt ran an average of 1.5 miles
per hour. If he had increased his pace by 10 the part of the mass below water is between
seconds per mile by June, then which of the
following expresses the number of hours it 1 1
would take Kurt to complete one mile in June? and
5 6
1 1
3590 and
6 7
602
5 5
2410 and
6 7
602
2390 6 and 7

2
60
6 7
3585 and
7 6
60
602
6. Is (9x)3 – 2x= 1 ?
3590
(1) The product of x and positive integer y is not x.
(2) x is an integer.

DO NOT DISTRIBUTE
| 205
GMAT IN CLASS MANUAL

−1
 45−1 + 5−1  11. If each marble in a set of 50 marbles is
7.   =
  either red, black, or white, how many
 10 
are red?
1

45 (1) 25 marbles are not black and 40
1 marbles are not white.
(2) There are 15 more black marbles
40 than white marbles, and 5 more red
2 marbles than white marbles.

9

5 Permutations and Combinations


1. Flippy’s Flowers is designing a special prom
45
corsage that consists of one rose, one orchid,
8. On this year’s Westchester basketball team, the and one gardenia. If Flippy carries four types
players are all either 5, 7, or 11 years of age. If of roses, three types of orchids, and five types
the product of the ages of the players on the of gardenias, how many different corsages can
team is 18,865, then what is the probability that Flippy design?
a randomly selected team member will NOT
12
be 7 ?
24
3 30

7 60
2 120

5 2. At a prestigious dog show, six dogs of different
16 breeds are to be displayed on six adjacent
podiums. If the Springer spaniel must be
37
displayed on the leftmost podium, how
3 many display arrangements of the six dogs

5 are possible?
49 5

55 6
30
9. If a = (23)(43)(59) and b = (46)(56)(69), then which 120
3
of the following values is less than ab ? 240
3. On Random Omelet Monday, a chef creates
(27)(55)(63)
omelets by randomly choosing three out of
(2)(43)(55)(63) a possible six fillings. How many different
(210)(33)(55) omelets can the chef possibly make?
(212)(55)(6)
9
(26)(53)(67)
18
10. What is the ratio of x to y ? 20
(1) 0.8x + 0.5y = 40 120
(2) 0.625y = 50 − x 720

DO NOT DISTRIBUTE
206 |
Lesson 8

4. The co-op board of a certain residential 8. Alfred, ever hungry, decides to order 4 desserts
building must consist of two men and three after his meal. If there are 7 types of pie and 8
women. If there are six men and seven women types of ice cream from which to choose, and
available for the committee, how many Alfred will have at most two types of ice cream,
different committees are possible? how many distinct groups of desserts could he
consume in his postprandial frenzy?
65
525 588
1,050 868
1,287 903
100,800 1,806
2,010
5. Eight Alaskan Huskies are split into pairs to pull
one of four sleds in a race. How many different 9. Ten telegenic contestants with a variety of
assignments of Huskies to sleds are possible? personality disorders are to be divided into two
“tribes” of five members each, tribe A and tribe
32
B, for a competition. How many distinct
64
groupings of two tribes are possible?
420
1,680 120
2,520 126
252
6. In a group of 8 semifinalists, all but 2 will 1,200
advance to the final round. If in the final round
1,260
only the top 3 will be awarded medals, then
how many groups of medal winners are 10. How many 4-digit numbers begin with a digit
possible? that is prime and end with a digit that is prime?
20 16
56 80
120 800
560 1,440
720 1,600
7. Alan has a flock of sheep from which he will 11. Chef Gundy is making a new “style” of salad
choose 4 to take with him to the livestock show which will contain two kinds of lettuce, one
in Houston. If Alan has 15 distinct possible kind of tomato, one kind of pepper, and two
groups of sheep he could take to the show, kinds of squash. If Chef Gundy has 8 kinds of
then which of the following is the number of lettuce, 4 kinds of tomatoes, 5 types of pep-
sheep in his flock? pers, and 4 kinds of squash from which to
choose, then how many different “styles” of
30 salad can he make?
15
640
7
1,120
6
2,240
5
3,360
13,440

DO NOT DISTRIBUTE
| 205
GMAT IN CLASS MANUAL

12. A teacher is assigning 6 students to one of 14. A baseball team consists of 20 players, 5 of
three tasks. She will assign students in teams whom are pitchers and 15 of whom are position
of at least one student, and all students will be players. If the batting order consists of 8
assigned to teams. If each task will have exactly different position players and 1 pitcher, and if
one team assigned to it, then which of the the pitcher always bats last in the order, then
following are possible combinations of teams which of the following expressions gives the
to tasks? number of possible different batting orders for
I. 90 this baseball team?
II. 60
III. 45 (15!)(5)

I only 8!
I and II only (15!)(5)

I and III only 7!
II and III only
(15!)(5!)
I, II, and III
7!
13. Entries in a particular lottery game are made
up of three digits each, 0 through 9. If the order (15!)(5)
of digits in the entries matters, how many
different possible entries exist in which all three 20!
digits are not equal?
516
720
989
990
1,321

DO NOT DISTRIBUTE
208 |
Lesson 8

Answers and Explanations

Mixed Topics
1. B Plug In The Answers. Of course, first step is to convert 2,390 seconds to hours. To do
you need to set up the group formula: so we must divide 2,390 by 60 to get minutes
Total = Group 1 + Group 2 – Both + Neither, and then divide it again by 60 to convert minutes
so 150 = 45 + 72 – B + N. Start with (C). If 44 into hours.
students are in both groups, 88 would be in
Neither. Does 150 = 45 + 72 – 44 + 88? No. 4. C You need to start by figuring out how many
That comes out to 150 = 161, clearly an incorrect people are in each home; we have 150 million
formulation. You need a smaller number. Try 33, people in 30 million homes. Just ignore all
which would mean 66 would be in Neither group. those extra zeros, and you’ll realize you need to
Does 150 = 45 + 72 – 33 + 66? Absolutely. divide 15 by 3, which means there are 5 people
per home. Next, we need to figure out how
2. B First, there are only 4 prime digits: 2, 3, 5, and 7. many homes were destroyed; 0.01 percent of 30
Next, if you start writing down the numbers that million is 3,000. Now, half of the inhabitants
meet the question’s criteria, you will see a pattern of the destroyed homes decided to move away;
emerge. Between 0 and 99, the only numbers that if there are 5 people per home, then there were
will work are: 22, 23, 25, 27, 32, 33, 35, 37, 52, 15,000 people in the destroyed homes. Half of
53, 55, 57, 72, 73, 75, and 77, for a total of 16 them left the city, so 7,500 left. Now translate
numbers. Between 100 and 199, the only numbers into scientific notation: 7,500 = 7.5 × 103.
that will work are: 122, 123, 125, 127, 132, 133,
135, 137, 152, 153, 155, 157, 172, 173, 175, and 5. A First, express the ratio of mass above water to the
177; a total of 16 numbers. The pattern becomes
clear; in every hundred, there are 16 numbers that
you want. Since there are 15 hundreds between 0 mass below water when 1 of the mass is above
6
and 1,500, so far you have 15 × 16 = 240 numbers. 1
Lastly, you need to count the numbers from 1501
to 1570 that meet the question’s requirement. the water: 6 = 1 . Now, repeat the process for
5 5
Those are: 1522, 1523, 1525, 1527, 1532, 1533, 1
6
1535, 1537, 1552, 1553, 1555, and 1557 for an 1 7 1
additional 12 numbers. 240 + 12 = 252. when of the mass is above the water: = .
7 6 6
7
3. C The problem has two conversions to watch out 6. C Start by translating the question and understand-
for; first, it gives 1.5 miles in March but 1 mile
ing the pieces of the puzzle given and the pieces
in June, second, it adds 10 seconds to his mile per
hour rate. The order in which you deal with these needed. The only way to make the equation equal
are up to you, but they must be dealt with. First 1 is for 9 to be raised to the power of 0. For that
let’s deal with the 1.5 mile to 1 mile problem. 3
to happen, x must either be 0 or . Statement
Initially, he runs 1.5 miles per hour, which is 2
the same as saying that he does 3 halves of a (1) tells us that x is not 0, but it doesn’t tell us
mile in 60 minutes, thus each half must take 20 3
whether x is an integer or whether it could be  .
minutes. Now, we know that in March it took 2
him 40 minutes to run a mile. Let’s now convert So, BCE. Statement (2) says that x is an integer, so
3
those minutes to seconds, 40 minutes = 2,400 it can’t be , but it could still be 0 or some other
2
seconds. If by June he increased his pace by 10 integer; thus this statement alone is not sufficient.
seconds, that means it would take him less time to
complete the mile, so in June a mile would take Eliminate choice (B). Together, we know that x is
him 2,390 seconds. Now we have the time it neither a fraction nor 0, so there’s no way that the
would take him to do a mile in June, so the last
equation can equal 1.

DO NOT DISTRIBUTE
| 205
GMAT IN CLASS MANUAL

7. E This question tests basic math in a somewhat 10. E Statement (1) provides one equation with two
variables. Since there’s no way to find either the
complex manner, combining exponent and
values of x and y or to rearrange this equation
x
fraction rules. First, we should probably rewrite to find the value of (the ratio of x to y), this
y
statement is insufficient. The possible answers are
the numbers with negative exponents as fractions: B, C, or E. Statement (2) is insufficient for the
1 same reasons. Eliminate choice (B). When the
45−1 = and 5−1 = 1 . Add the fractions together,
45 5 infomation is combined, it may look as though
and you get 10 or 2 . Next, remember that there are two equations. However, if the equation
45 9
in statement (1) is multiplied by 1.25, the result
dividing by 10 is the same as multiplying by 1 .
10 is x + 0.625y = 50, which is the equation from
2 1 1 Statement (2). Since one equation is a multiple
× = . Now, we have to deal with the final
9 10 45
−1 of the other, there’s really just one equation and
negative exponent:  1  = 1 = 45 .
 45  1 the combined information is insufficient.
45
8. B Before the probability can be found, you need 11. D Remember to use the Pieces of the Puzzle
to know how many 7-year-olds are on the team approach for value data sufficiency questions.
and how many total members the team has. The From the question stem, you know that
phrase “the product of the ages of the players . . .” r + b + w = 50, which provides one equation in
gives us the hint that we will need to factor 18,865 three variables. To answer the question, you’ll
to find the age distribution of the team members. need two more equations with those variables.
Also, since the ages given are all prime numbers, (Remember that you always need as many
a factor tree will help a lot here. The prime
equations as there are variables.) Translate
factorization of 18,865 is 5 × 7 × 7 × 7 × 11, so
Statement (1) to find that r + w = 25 and r + b = 40.
there must be five children on the team whose
ages are 5, 7, 7, 7, and 11. So the probability Since the statement provides two more equations,
of selecting a child that is not age 7 will be two it is sufficient. The possible answers are A or D.
(because two of the children are not age 7) out Statement (2) can be translated to find that
of five (because there are 5 total children from b  =  w  +  15 and r  =  w  +  5. Again, since the
whom to choose). statement provides two additional equations, it
is sufficient.
9. D This question is all about simplification. First,
translate both a and b into their simplest forms:
a = (23)(4 3)(5 9) = (23)(2 6)(5 9) = (29)(5 9) and Permutations and Combinations
b = (46)(56)(69) = (212)(56)(29 × 39) = (221)(56)(39).
So ab = (230)(515)(39) and 3 ab = (210)(55)(33). 1. D If Flippy chooses one of each flower, he has
4 × 3 × 5, or 60, different corsages.
Now just compare that number with your answer
choices. Remember that 6 is the same as 2 × 3, so 2. D The Springer spaniel must be on the far left. After
63, for example, is the same as (23 × 33). that, 5 dogs could be on the second podium, 4
on the third, and so forth. The total number of
arrangements is 1 × 5 × 4 × 3 × 2 × 1, which
equals 120.

DO NOT DISTRIBUTE
210 |
Lesson 8

3. C For this problem, the order doesn’t matter, so you’ll 7. D This problem, for which the order doesn’t matter,
need to divide. To choose 3 out of 6 fillings, just may seem somewhat tougher because we aren’t
6 × 5 × 4 120
compute = = 20 . given the number of sheep that were initially in
3× 2×1 6
the flock. Or, so it seems. Plug In The Answers! In
4. B First, find the number of ways you can choose 3 the answers we are given choices for the number
of 7 women:  7 × 6 × 5  = 35, and the number of of sheep in the flock; all we have to do is try them
 3× 2×1 
 6×5  out until one gives us the right number of distinct
ways you can choose 2 of 6 men:   = 15 .
 2×1 
groups when choosing 4 out of the flock. Start
Multiply the numbers together: 35 × 15 = 525.
with (C). If there are 7 sheep in the flock, then we

5. E Here, the order doesn’t matter. You must find out need to find out how many ways we can choose 4
of 7 sheep when order does not matter. The math
how many ways you can create the four teams. For
would look like: 7 × 6 × 5 × 4 . Since this yields 35,
the first team, you have 8 × 7 = 28 possibilities. 4 × 3 × 2 ×1
2×1 there must be fewer than 7 sheep. Try a smaller
For the second team, you have 6 × 5 = 15
2×1 number. Six works.
possibilities (you have only 6 options because 2
8. C Figure all the distinct combinations: PPPP (35),
dogs were assigned to the first sled). For the third PPPI (280), and PPII (588). Add them together,
4×3 and the total is 903.
sled, you have = 6 possible groups. For the
2×1 9. C You have to select five out of ten for tribe A,
2×1
final team, you have = 1 group. To arrive yielding 252 possible tribe A’s. Tribe B must
2×1
consist of the remaining contestants, so there is
at your final answer, just multiply the numbers just one possible tribe B.
together. 1 0. E To find out how many different numbers will
have a prime as the first digit and a prime as the
6. B The entire discussion of rounds is a red herring. last digit, we only to need find out how many
different choices there are for each digit in the
The question is asking for possible combinations four digits. For the first and last digits, we have 4
of the final three, and it is possible for any of the different possible numbers (prime digits 2, 3, 5,
and 7). For the second and third digits, we have
original 8 contestants to have advanced to the final 10 possibilities (0–9, inclusive). If we multiply the
possibilities for each digit (4 × 10 × 10 × 4), then
round, thus we need to pick 3 out of 8, and order
we get the total number of combinations possible
doesn’t matter. 8 × 7 × 6 = 56 . for the four-digit number.
3× 2×1

DO NOT DISTRIBUTE
| 205
GMAT IN CLASS MANUAL

11. D Find the number of ways you can choose 2 out of 13. D For each entry, order matters and we have a
choice from among 10 possibilities for each
8 lettuces, then 1 out of 4 tomatoes, then 1 out of digit, so the total number of possible entries is
103 = 1,000. For each integer, there is exactly one
5 peppers, and lastly 2 out of 4 squash. Multiply entry for which all the digits are the same. So the
total number of entries with the digits not all the
them all together. The math would look like this:
same is 1,000 – 10 = 990.
 8×7   4×3 
  ( 4 )( 5 )   = 3 , 360.
 2×1   2×1 
14. B We are choosing 8 of the 15 position players, and
12. B The one twist in the problem is that you have
not been told how many members are on each order matters. So the number of possible orders is
team, thus allowing you to get several different 15 ! 15 !
= = . We are choosing any 1 of the 5
answers. The best way to approach this problem (15 − 8 ) ! 7 !
is to try out the different possible ways of pitchers, so we multiply the result we got in the
arranging the team members: You could have
teams with equal numbers (2 on a team), you prior step by 5 to get our answer.
could have 3-, 2- and 1-member teams, or you
could have 4-, 1- and 1- member teams. Now
just figure out the possibilities for each of these
options, and eliminate answers as appropriate.

DO NOT DISTRIBUTE
212 |
Lesson 9

Lesson 9

99

Verbal Review

Sentence Correction
Remember to always use the Basic Approach when tackling any Sentence
Correction question.

Can you ID one of the common errors in the underlined portion?


Yes No
Eliminate Choice (A) Can you ID an error from
the answer choices?
• 2/3 split
Eliminate any answers that • Differences in answers
obviously repeat the error. Yes No

POE with the remaining Starting at (B), use POE.


answer choices. • New errors.
• New errors. • Changes to the meaning
• Changes to the meaning of the sentence.
of the sentence.

Two left? Choose the more concise answer.

DO NOT DISTRIBUTE
| 213
GMAT IN CLASS MANUAL

Now, use the Basic Approach to work these questions. Remember to look for
commonly tested errors.

1.
Having left no historical record of their own, the
histories of their enemies provide modernity the only
record of the Huns, who conquered numerous
Germanic tribes in Europe and assaulted the Eastern
Roman Empire.
Having left no historical record of their own, the
histories of their enemies provide modernity
99 the only record of the Huns, who conquered
numerous Germanic tribes in Europe and
assaulted the Eastern Roman Empire.
Having left no historical record of their own,
modernity knows the Huns, who conquered
numerous Germanic tribes in Europe and
assaulted the Eastern Roman Empire, entirely
through the histories of their enemies.
Although they have left no historical record of
their own, conquered numerous Germanic tribes
in Europe and assaulted the Eastern Roman
Empire, the Huns are known to modernity
entirely through the histories of their enemies.
Having left no historical record of their own, the
Huns, who conquered numerous Germanic
tribes in Europe and assaulted the Eastern
Roman Empire, are known to modernity
entirely through the histories of their enemies.
Leaving no historical record of their own,
modernity knows the Huns through the
histories of their enemies, who conquered
numerous Germanic tribes in Europe and
assaulted the Eastern Roman Empire.

DO NOT DISTRIBUTE
214 |
Lesson 9

2. The demand for rare earth minerals like dysprosium


and similar elements remain significant, prompting
widespread illegal mining of the minerals, which are
used extensively in high-tech manufacturing.
minerals like dysprosium and similar
elements remain
minerals like those of dysprosium and
similar elements are remaining
minerals such as dysprosium and similar
elements remains
99
minerals, such as dysprosium and similar
elements, remain
minerals, like the minerals of dysprosium
and similar elements, remains

Unlike most of Norway, where residents can rely on


3.
a sunrise every day of the year, in Finnmark the sun
rises only from late January through November, and
during the other two months the county
experiences polar night and polar twilight, in which
the sun never appears above the horizon.
Unlike most of Norway, where residents can
rely on a sunrise every day of the year, in
Finnmark the sun
Unlike most of Norway’s residents who can
rely on a sunrise every day of the year, the
sun in Finnmark
Unlike those of most of Norway, where
residents can rely on a sunrise every day of
the year, Finnmark’s sun
In comparison with most of Norway, whose
residents can rely on a sunrise every day of
the year, the sun in Finnmark
In most of Norway, residents can rely on a
sunrise every day of the year, but in
Finnmark the sun

DO NOT DISTRIBUTE
| 215
GMAT IN CLASS MANUAL

4.
On August 15, 1974, President Nixon unilaterally ended
the convertibility between the dollar and gold, which
was intended to prevent a foreign government from
precipitating an exchange crisis.
On August 15, 1974, President Nixon unilaterally
ended the convertibility between the dollar and
gold, which was
President Nixon unilaterally ended the
convertibility between the dollar and gold on
August 15, 1974, and it was
9
President Nixon unilaterally ended the
convertibility between the dollar and gold on
August 15, 1974, which was
On August 15, 1974, President Nixon unilaterally
ended the convertibility between the dollar and
gold and it had
On August 15, 1974, President Nixon unilaterally
ended the convertibility between the dollar and
gold, an action

5. The Great Blizzard of 1888 cut off many telegraph


and railroad links in the northeastern United States,
and is still remembered for its striking features: the
unseasonably warm and rainy weather that preceded it,
the sustained winds that created snowdrifts more than
50 feet tall, and prodigious snowfall was measured at
over 50 inches in New England.
the sustained winds that created snowdrifts
more than 50 feet tall, and prodigious snowfall
was measured at over 50 inches in New
England
the sustained winds created snowdrifts more
than 50 feet tall, and in New England the
prodigious snowfall was measured at over 50
inches
the sustained winds that created snowdrifts
more than 50 feet tall, and the prodigious
snowfall measured at over 50 inches in New
England
snowdrifts were created more than 50 feet
tall by the sustained winds, in New England
the prodigious snowfall measured at over 50
inches
snowdrifts which were created more than 50
feet tall by the sustained winds, and prodigious
snowfall that was measured at over 50 inches
in New England

DO NOT DISTRIBUTE
216 | © Princeton Review Management, L. L. C.

17 GMAT InClass Manual Lesson 9.indd 216 7/2/18 6:46 PM


Lesson 9

Critical Reasoning
By now you should be a pro at applying the Basic Approach to different types of
arguments.

Step 1: Identify the question.


Begin by reading the question stem. Identify the question type before you read
the argument.

Step 2: Work the argument.


For most question types, you will start working the argument by finding the
99
conclusion and premises. Next, you’ll want to find the gap, assumption, or pattern.

Step 3: Predict what the answer should do.


Think about what the credited response needs to do.

Step 4: Use POE to find the answer.


It’s easier to find wrong answers than right answers. Use POE aggressively.

Use the Basic Approach to work these questions:

1. In 2000, the number of new students admitted to Ph.D.


programs each year averaged 1,250 students per university,
while the total number of Ph.D. students enrolled averaged
7,500 students per university. By 2010, the number of new
students admitted to Ph.D. programs each year had fallen
to an average of 900 students per university, while the total
number of Ph.D. students enrolled averaged 8,100 students
per university.
Which of the following conclusions is most strongly
supported by the statements above?
The total number of students enrolled in Ph.D. programs
increased from 2000 to 2010.
The average length of time a student remained enrolled
in a Ph.D. program increased between 2000 and 2010.
The percentage of applicants accepted by Ph.D.
programs declined from 2000 to 2010.
The number of universities remained constant from
2000 to 2010.
The demand for Ph.D. degrees declined from 2000 to 2010.

DO NOT DISTRIBUTE
| 217
GMAT IN CLASS MANUAL

2. A recent study suggests that regular exercise improves the health of a per-
son’s heart and cardiovascular system. Five years ago, people under the age
of 60 accounted for 50 percent of the people who had suffered one or more
heart attacks. Today, people under the age of 60 account for only 40 percent
of the people who have suffered one or more heart attacks. The same study
shows that people under the age of 60 exercise more regularly today than they
did 5 years ago, while the exercise habits of people aged 60 and over have
remained the same.

Which one of the following most strengthens the argument?


99 Some people over the age of 60 exercise as much or more
than do people under the age of 60.
The proportion of the population aged 60 and over has
remained constant over the last five years.
The use of cholesterol-lowering drugs has reduced the
frequency of heart attacks among all age groups.
People aged 60 and over are generally less capable of
strenuous exercise than are people under the age of 60.
A number of factors, such as nutrition and stress levels,
affect the incidence of heart attacks.

DO NOT DISTRIBUTE
218 |
Lesson 9

3. Every driver in the United States is legally required to


purchase liability insurance that protects other individuals
in the event that the driver causes property damage or
bodily injury. Some politicians argue that this insurance is
partly responsible for the high rate of automobile collisions,
because it reduces the drivers’ financial incentives to operate
their automobiles in a safe and responsible manner. If drivers
were required to pay directly for any damage they cause, they
would drive more carefully.

The politicians’ argument makes which of the following 99


assumptions?
If drivers were not required to carry liability insurance,
individuals would be unprotected from uninsured drivers
with little money to pay for damages they cause.
Drivers who cause bodily injury to another feel little or
no regret for their actions.
Responsible drivers and reckless drivers pay similar
premiums for liability insurance.
The cost of liability insurance is more than some drivers
can afford.
Most drivers would purchase liability insurance even if
they were not required to do so by law.

DO NOT DISTRIBUTE
| 219
GMAT IN CLASS MANUAL

Reading Comprehension

The Basic Approach


Step 1: Work the passage.
Read for the passage’s main idea, and get a sense of its structure.

Step 2: Understand the Question Task.


Look for the question’s subject and the words that indicate its task.
99
Step 3: Find the Information that Addresses the Task.
Once you understand the question task, use the question’s subject to locate
information in the passage that addresses the question task.

Step 4: Use POE to find the answer.


Be skeptical of each answer choice. Try to find reasons to eliminate the answer.
Eliminate answers that:

• Use recycled language.


• Use extreme language.
• Make comparisons that are not in the passage.
• Reverse an idea from the passage.
• Make emotional appeals or use outside knowledge.

DO NOT DISTRIBUTE
220 |
Lesson 9

Until recently, multinational corporations (MNCs) were


able to dominate markets by sheer force of their size,
affecting consumer tastes and flow of capital on a scale
undreamed of since the days of the British Empire.
(5) Today, however, the marketplace that MNCs helped to
create no longer yields the steep profits that it once
did, although tariffs and other obstacles to international
trade have grown less severe. Many observers trace the
slowdown to the MNCs failure to adopt management
(10) structures appropriate to the computer age.
Following World War II, MNCs employed a 99
“command-and-control” management structure. Under
this system, upper management determined company-
wide policy and dictated it to battalions of middle
(15) managers worldwide, who would then execute these
orders at various outposts, making few adjustments
for local circumstances. This technique worked only so
long as competition was limited to a few leviathans. But
newly available communication technologies and an
(20) emphasis on service render command-and-control
techniques inefficient, at best. Shrinking overhead for
even the smallest companies (due to dramatic
technological advances) has revealed the greatest
drawbacks of command-and-control: smaller enterprises
(25) can respond to consumer needs more quickly than can
the lumbering giants.

1. The primary purpose of the passage is to


explain the “command-and-control” management style
utilized by MNCs
argue that the reduced profits of the MNCs result from
their failure to adapt their management frameworks to
technological innovations
justify the management policies that MNCs have used
since the days of the British Empire
compare the MNCs’ “command-and-control”
management style to the more flexible management
styles of smaller companies
dismiss a possible explanation for the shrinking profits
of MNCs

DO NOT DISTRIBUTE
| 221
GMAT IN CLASS MANUAL

2. The author mentions the British Empire in order to


provide an illustration of how failure to adapt to
technological advances can cause economic decline and
shrinking profits
give an example of a leviathan
show how the recent expansion of smaller enterprises is
similar to the growth of nationalism that resulted from the
dissolution of the British Empire
point out the contrast between the British Empire’s tactics
of coercion and intimidation and strategies that work
99 better for a free world market
show that MNCs were able to dominate world markets in
much the same way as the British Empire was able to at
its peak

3. The passage suggests that all of the following are reasons for the
decline of MNCs EXCEPT
the reduced overhead costs of smaller businesses
MNCs’ slower than average response to changes in
consumer preferences
the elimination of middle managers by smaller businesses
MNCs’ failure to update their management structures
smaller businesses’ exploitation of new technologies

DO NOT DISTRIBUTE
222 |
Lesson 9

Math 8

POE Review
When you are stuck on a question or running out of time, look for good reasons
to eliminate answers. Then, guess and move on.

Common Trap Answers Include 99


• Too Obvious
• Numbers in the Problem
• Simple Manipulations
• Partial Answers

You have 1 minute to eliminate as many incorrect answers as possible for each of
the following four questions.

1. What is the product of the average (arithmetic mean) and the median
of the set composed of the distinct prime factors of 4,095 ?
25
28
31
35
42

DO NOT DISTRIBUTE
| 223
GMAT IN CLASS MANUAL

A D C

99

2. In the figure above, ABC is an equilateral triangle with an area of 9 3 .


B is a point on the circle with center D, and D is the midpoint of line
segment AC. What is the area of the shaded region?

18 π − 9 3

9 3π

27π − 9 3

36π − 9 3

243π

DO NOT DISTRIBUTE
224 |
Lesson 9

3. Al can complete a particular job in 8 hours. Boris can complete


the same job in 5 hours. Cody can complete a second job, which
requires twice as much work as the first, in 8 hours. If all three
people work together on the larger job for 2 hours, how long, in
hours, would it take Al, working alone, to finish the job?
0.8
3.0
6.8
8.0 99
8.8

4. If 66 × 103 × 152 = 2a × 3b × 5c, then which of the following is the value


2

of a × b 3 × c−2 ?

–120

2

3
12

25
72

5

360

DO NOT DISTRIBUTE
| 225
GMAT IN CLASS MANUAL

Data Sufficiency Review


1. Are v and w integers?
What should you do when
you see a Yes/No question?
(1) v + w is an integer.

v
(2) is an integer.
w
99

What should you do when 2. A farmer has 4 henhouses, each containing 6 egg-laying hens.
you see a value data
sufficiency? Each henhouse is painted a different color: red, blue, green, and
yellow. What was the average (arithmetic mean) egg yield per hen
last week?

(1) The hens in the red and blue henhouses yielded a total of 36 eggs
last week.
(2) The hens in the blue and yellow henhouses yielded a total of 48 eggs
last week.

DO NOT DISTRIBUTE
226 |
Lesson 9

3. In City Y last year, what was the ratio of the number of


commuters who rode buses to the number of commuters
who rode trains?

(1) In City Y last year, 50,000 commuters rode trains and What’s the trap answer?
48,000 commuters rode buses.

(2) In City Y last year, if the number of commuters who rode


trains had been 20 percent greater, the number of
commuters who rode trains would have been 125
percent of the number of commuters who rode buses. 99

DO NOT DISTRIBUTE
| 227
DO NOT DISTRIBUTE

You might also like